You are on page 1of 120

!

"!#$%&$'
"!#$%&$' )**
)**+,'%
+,'%

%-. $'/ 011234 5-. 6.76289:5;2<


6.7
6.76289:5;2<
6289:5;2< 2= 5-;4
5-;4 760:5;:. >22?1.5 ;< 0<@ A0<<.6
A0<<
A0< <.6
.6 49;50>1. 52
5-. "-;,B#%
"-;,B#% 0771;
0771;:0<5C
0771;:0<5
0771;:0<5C
:0<5CC *5-.6
*5-.6 94.4 ;<:198;<
*5-.6 ;<:198;<DD 4.11;<D
4.11;<D 26 1;:.<
1;:.<4;<D
4;<D :27;.4E
:27;
:27;.4E
.4E 26 7245;
7245;<D
7245;<D
<D

52 7.642<01E
7.642<01E ;<45;595;2<01
;<45;595;2
;<45;595;2<01
<01 26 5-;68 7065@
7065
7065@
@ 3.>4;5.
3.>4;5. 06. 762-;>;5.8C
762-;>;5.8C
GENERAL DIRECTIONS

The Philip
Philippin
pinee Law Scho
Schoolol Admis
Admissio
sion
n Test
Test consis
consists
ts of four
four subte
subtests
sts.. Each
Each subtes
subtestt
contains multiple-choice items.

For each item, select


select your answer
answer from the
the options
options given.
given. On your
your answ
answerer sheet,
sheet, shade
shade
the circle
circle marke
marked
d with
with the letter
letter of your
your chosen
chosen answ
answer.
er. For
For examp
example,
le, if your
your answ
answerer to

an
sureitem
sureitem
yourismark
your opti
option
markon
onC, then
thcircle
the encle
cir comp
complelete
tely
ly shad
is dark.
dark. sh
Avade
e the
Avoid
oidthe orrec
circ
circle
incorr
inc let mark
ect ma rked
shadied
ng C
shading ofas
cirshow
shown
circle
cles ans belo
s as be
thlow
eywmay
they . Make
Ma ket
not
no
  be recog
recogniz
nized
ed as an answ
answer.
er.

Make
Make sure
sure you
you are markin
markingg the answ
answerer colum
columns
ns corre
correspo
spond
nding
ing to the item
item numbe
numberr you
you
are
are on.
on. Mark
Mark only
only one
one answ
answerer for
for each
each item.
item. If you
you want
want to chan
change
ge your
your answ
answer
er,, eras
erasee
the first answer
answer comp
completely
letely.. Incomple
Incomplete te erasures
erasures will be
be interprete
interpreted
d as another
another answer 
answer 
thereb
therebyy produ
producin
cingg “multip
multiple
le answe
answers.
rs.”” Items
Items with
with multip
multiple
le answ
answers
ers are autom
automati
atical
cally
ly
considered wrong.

Do not
not write
write anythi
anything
ng on this
this test
test book
booklet
let.. Use
Use the blank
blank pages
pages of
of your
your answer
answer shee
sheett for 
your scratch work.

Follo
Follow
w care
carefu
full
lly
y the
the spec
specif
ific
ic dire
directctio
ions
ns for
for each
each subt
subtes
estt or sect
sectio
ion.
n. When
When you
you fini
finish
sh a
subtes
subtest,
t, proce
proceed
ed to the next
next unti
untill you
you have
have compl
complete
eted
d the entire
entire test.
test.

DO NOT TURN THIS PAGE UNTIL YOU ARE TOLD TO DO SO.

PRACTICE BOOKLET
3

TEST
TEST
TEST A. COMMUNICATIO
COMMUNIC
COMMUNICA
COMMUNICATION
ATION
TION
N AND LANGU
LANGUAGE
AGE PROFICIE
PROF
PROFICIENCY
NCY

Section
Sectio
Section
n 1. Identifyin
Ident
Identi
Identifyi
ifying
fying
ng
g Sentence
Sente
Sentence
nce Errors
Errors
Errors

Direct
Direction
ions:
s: Read
Read each
each senten
sentence
ce carefu
carefully
lly but
but quick
quickly,
ly, paying
paying attent
attention
ion to the unde
underlin
rlined
ed word
word or 
  phrase
phrase.. Each
Each senten
sentence
ce conta
contains
ins eith
either
er a single
single error
error or
or no error
error at
at all.
all. If the
the senten
sentence
ce conta
contains
ins an
error,
error, select
select the under
underlin
lined
ed word
word or phras
phrasee that
that must
must be chang
changed
ed to make
make the senten
sentence
ce corre
correct.
ct. If                 
the sentence
sentence is correct,
correct, select
select choice
choice D.

In choosin
choosing
g answers,
answers, follow
follow the requireme
requirements
nts of standard
standard written
written English.
English.

1. Anyb
Anybod
ody
y who
who is ready
ready with
with theanswer
theanswer may
may raise
raise their
their right
right hand
hand.. No error 
error 
A B C D

2. The chair
chairma
man
n of the board
board had spend
spend nearly
nearly an hour listen
listening
ing to the member
memberss of
of the
the press
press
A B C

before
before issuin
issuing
g the order.
order. No error 
error 
D

3. Some
Some youn
young
g peop
people
le want
want succes
successs badly
badly but
but they
they are not
not willing
willing to work
work for it. No error 
error 
A B C D

4. Some universities has


has set up small colleges for
for closer relationships
relationships between
between professors and
and
A B C

stud
studen
ents
ts.. No erro
error r 
D

5. The
The admi
admini
nist
stra
rati
tion
on offi
office
cers
rs have
have expr
expres
esse
sed
d they’r
they’ree gratitu
gratitude
de to the rank and file
A B

for their loyalty


loyalty and support.
support. No error 
C D

6. It is imperativ
imperativee that the matter
matter be kept unknown
unknown from the organize
organizers.
rs. No error 
error 
A B C D

7. Ramire
Ramirez,
z, accept
accepting
ing his defeat
defeat like
like a true
true sport
sport,, cong
congrat
ratula
ulated
ted the newly
newly procl
proclaim
aimed
ed
A B C

champion
champion.. No error 
D

PRACTICE BOOKLET
BKL PhiLSAT_AA_0317 GO ON TO
TO THE NEX
NEXT
NEXT
XT PAGE
PA
PAGE
AG
GE
E
4

8. The const
construc
ructio
tion
n worke
workers
rs were
were tired
tired but
but the aftern
afternoo
oon
n nap rest
rest them.
them. No error 
error 
A B C D

9. Did you
you notic
noticee the way
way she look
look when
when she
she caugh
caughtt sigh
sightt of
of the
the accide
accident
nt victim
victims?
s? No error 
error 
A B C D

10.
10. Smoke
Smoke billow
billowed
ed abov
abovee the roofto
rooftops
ps as fireme
firemen
n battled
battled their
their way
way throu
through
gh the noisy
noisy crowd
crowd..
A B C

  No error 
error 
D

11.
11. Food
Food at the
the cant
cantee
een
n is expe
expens
nsiv
ive,
e, so I use
use to brin
bring
g a pack
packed
ed lunc
lunch.
h. No erro
error r 
A B C D

12.
12. Althou
Although
gh tired
tired and hung
hungry,
ry, the gover
governo
norr reluct
reluctant
antly
ly agreed
agreed to a twenty-
twenty-min
minute
utess press
press
A B C

conferen
conference.
ce. No error 
error 
D

13.
13. The compu
computer
ter,, a leadin
leading
g techn
technolo
ologic
gical
al dev
device
ice,, it make
makess human
human tasks
tasks easier
easier to do.
do.
A B C

  No error 
error 
D

14. At the
Atime
time of the revolu
revolutio
tion,
n, violen
violence
ce durin
during
g the
B vigil
vigil and momen
moments
ts of prayer 
prayer 

minimized
minimized by the leaders
leaders of differe
different
nt religiou
religiouss sectors.
sectors. No error 
error 
C D

15.
15. That
That nine
ninety
ty perce
percent
nt of the body
body consis
consists
ts of fluids
fluids,, parti
particul
cularl
arly
y blood
blood and
and water
water.. No error 
error 
A B C D

16.
16. Mayo
Mayon
n Volc
Volcan
ano
o is an acti
active
ve stra
strato
tovo
volc
lcan
ano
o in the
the Bico
Bicoll Regi
Region
on reno
renown
wned
ed for
for its
its
A

most
most perfect
perfect cone
cone and
and nomin
nominate
ated
d as one of the wond
wonders
ers of the
the world
world.. No error 
error 
B C D

17.
17. If the Filipi
Filipino
noss had
had not
not gon
gonee to the street
streetss to
to demon
demonstr
strate
ate their
their resist
resistanc
ancee agains
againstt
A B

the dictat
dictator,
or, we would
would not
not have
have regain
regained
ed our
our freedo
freedom.
m. No error 
error 
C D

PRACTICE BOOKLET
BKL PhiLSAT_AA_0317 GO ON TO
TO THE NEX
NEXT
NEXT
XT PAGE
PA
PAGE
AG
GE
E
5

18.
18. The Sena
Senate
te has unti
untill Novemb
November
er to cond
conduct
uct inter
intervie
views
ws for
for the confi
confirma
rmatio
tion
n of                 
A

the
the President’s appointees,  but the senators decided to do this after the budget session.
B C

  No error 
error 
D

19. If the
the phon
hone rin
rings and
and I’m not
not aroun
around,
d, please
please   answer
answer it and
and make sure
A

to get the caller’s


caller’s name,
name, his contact
contact number
number and what
what his
his messa
message
ge is. No error 
error 
B C D

20.
20. Most
Most of
of the
the athlet
athletic
ic equip
equipme
ments
nts used
used this
this year
year by the PE Depa
Departm
rtmen
entt are
are newly
newly acquir
acquired
ed..
A B C

  No error 
error 
D

21. Our history


history teacher
teacher believes
believes that twelve
twelve meetings
meetings are insufficie
insufficient
nt for us to master 
master 
A B C

the history
history of Japan
Japan,, so he assigned
assigned additiona
additionall readings
readings and
and projects
projects for suppo
support.
rt.

  NoD
error 
error 

22.
22. Despi
Despite
te the bad weath
weather
er last
last Sund
Sunday,
ay, the family
family fun-ru
fun-run
n spons
sponsore
ored
d by the paren
parents
ts were
were
A B C

so successf
successful
ul that they were able to raise sufficien
sufficientt funds
funds to buy school
school materials
materials for 

the
the Aeta
Aetas.
s. No erro
error r 
D

PRACTICE BOOKLET
BKL PhiLSAT_AA_0317 GO ON TO
TO THE NEX
NEXT
NEXT
XT PAGE
PA
PAGE
AG
GE
E
6

Section
Sectio
Section
n 2. Sentence
Sente
Sent
Senten
ence
nce
ce Completion
Completio
Completion
n

Direct
Direction
ions:
s: Choo
Choose
se the word
word or phras
phrasee that,
that, when
when insert
inserted
ed in the senten
sentence,
ce, best
best fits
fits the meanin
meaning
g
of the
the senten
sentence
ce as a whole
whole..

23. The class condoled


condoled with their teacher
teacher when
when her mother
mother _______
__________
______
___ last week.
week.

(A) passed up
(B) passed off                  

(C)
(D)  p
paassed
ssed aout
way

24.
24. The corne
cornered
red kidna
kidnappe
ppers
rs were
were force
forced
d to ____
______
____
_____
_____
__ their
their white
white flag
flag becau
because
se they
they
feared for their
their lives.
lives.

(A) rise
(B) race
(C) raze
(D) raise

25.
25. ____
______
____
____
____
___
_ his
his close
close rela
relati
tion
on with
with the
the vict
victim
im,, no one
one was
was allow
allowed
ed by the
the poli
police
ce to
enter
enter the scene
scene of the crime
crime..

(A) Regarding
(B)
(B) Irre
Irrega
gard
rdle
lesssly
sly of         

(C)
(D)
(D) Regardless
Irre
Irrega
gardlesssof 
rdle of                          

26.
26. _____
_______
____
____
____
__ the burgl
burglar
ar was,
was, he must
must have
have cased
cased our
our house
house for days
days befo
before
re actua
actually
lly
entering
entering our house
house that night.
night.

(A) Whoever  
(B) Whomever  
(C) Whosoever  
(D) Whomsoever 

27.
27. Mothe
Motherr tells
tells Sarah
Sarah that
that her small
small cat
cat named
named Elsie
Elsie ____
________
______
____
___
_ under
under the
the swing
swing on
the front
front porch.

(A) lays
(B) lying
(C) is lying
(D) is la
laying

PRACTICE BOOKLET
BKL PhiLSAT_AA_0317 GO ON TO
TO THE NEX
NEXT
NEXT
XT PAGE
PA
PAGE
AG
GE
E
7

28.
28. I _____
_______
____
____
____
__ the assign
assigned
ed task
task ahead
ahead of schedu
schedule.
le.

(A) done
(B) am do
d one
(C) was done
(D) have done

29.
29. The Presid
President
ent,, who
who is known
known to be a chain
chain-sm
-smok
oker,
er, might
might just
just ____
_______
_____
____
____
__ stop
stop
smokin
smokingg in his own
own time
time witho
without
ut being
being pressu
pressured
red by anyon
anyone.
e.

(A) quit

(B)
(C) quite
quitely
(D) quietly

30.
30. The employ
employees
ees are eager
eagerly
ly await
awaiting
ing for the date
date of ___
______
_____
____
____
___
_ of thei
theirr new
new
salary
salary scheme.
scheme.

(A) affectiv
tivity
(B) effectiv
tivity
(C)
(C) effe
effecctiv
tiveness
ess
(D)
(D) effe
effect
ctiv
ivel
elin
ines
esss

31.
31. Many
Many electi
election
on losers
losers susp
suspect
ect that
that some
some winne
winners
rs _____
_______
____
____
____
__ their
their way to
the victor
victory
y line.
line.

(A) cheat

(B)
(C) will
havecheat
cheated
(D) has cheated

32.
32. The invest
investors
ors ______
________
____
____
___
_ consid
considera
erable
ble capita
capitall when
when the tsuna
tsunami
mi devas
devastat
tated
ed
the province.
province.

(A) lose
(B) loss
(C) lost
(D) loose

33.
33. The rebels
rebels believ
believee that
that they
they are safe
safe in the mounta
mountains
ins where
where _____
_______
____
____
____
__ famili
familiar 
ar 
with
with the terrai
terrain.
n.

(A) there
(B) their  
(C) there are
(D) they a re
re

PRACTICE BOOKLET
BKL PhiLSAT_AA_0317 GO ON TO
TO THE NEX
NEXT
NEXT
XT PAGE
PA
PAGE
AG
GE
E
8

34. The propertie


propertiess of Alex’s rich unmarrie
unmarried
d aunt were
were divided
divided _______
__________
______ 
___  all
the nephews
nephews and nieces
nieces in accordan
accordance
ce with her last
last will
will and
and testament
testament..

(A) to
(B) with
(C) among
(D) between

35. Mother
Mother told us to _______
__________
______
___ our little
little brother’s
brother’s tantru
tantrums
ms since
since he’s a special
special child
child
and we all love
love him.
him.

(A)
(B)  p
puutt o
on
ff                  
(C) put acacross
(D) put up up with

36.
36. The
The snat
snatch
cher
er surp
surpri
rise
sed
d me from
from behi
behind
nd,, grab
grabbe
bed
d my hand
hand,, and
and ____
______
____
____
____
___
_ with
with
my handbag.
handbag.

(A) made do
(B) made out
(C) made off                  
(D) made away

37.
37. By Decemb
December
er next
next year,
year, Mr.
Mr. and Mrs.
Mrs. Ocamp
Ocampo
o ____
______
____
_____
_____
__ for forty
forty years
years..

(A) were married

(B)
(C) had
h avebeen
bee
ben
enmarrie
mar
marieddd
rried
ie
(D)
(D) will
ill ha
have beenmar
enmarrieried

38.
38. The Moun
Mountt Pinatu
Pinatubo
bo volca
volcanic
nic erupti
eruption
on is consid
considere
ered
d the ____
______
_____
_____
____
__ in
Philippine history.

(A) worse
(B) worst
(C) worsest
(D) most wo
worse

39.
39. _____
_______
____
____
____
__ Musli
Muslimm laws
laws are
are based
based on Islam,
Islam, specia
speciall shari
shariah
ah court
courtss have
have been
been
created
created to decide
decide on family
family and personal
personal conflicts
conflicts of Muslim
Muslimss in autonom
autonomousous regions.
regions.

(A) Since
(B) Although
(C) When
(D) Even though

PRACTICE BOOKLET
BKL PhiLSAT_AA_0317 GO ON TO
TO THE NEX
NEXT
NEXT
XT PAGE
PA
PAGE
AG
GE
E
9

Sectio
Section
n 3. Improv
Impro
Improving
rovin
ving
ing
g Senten
Sentence
tences
ces
s

Direct
Direction
ions:
s: Read
Read each
each senten
sentence
ce carefu
carefully
lly but
but quick
quickly
ly and
and determ
determine
ine wheth
whether
er the unde
underlin
rlined
ed
  port
portion
ion is cor
correc
rectt or whet
whether
her it need
needss to be revise
revised.
d. If the senten
sentence
ce needs
needs no
no revisi
revision
on,, choo
choose
se
optio
option
n D.

In choo
choosi
sing
ng answ
answer
ers,
s, follo
follow
w the
the requ
requir
irem
emenents
ts of stan
standa
dard
rd writt
written
en Engl
Englis
ish.
h. Pay
Pay atte
attent
ntio
ion
n to
gramm
grammar,
ar, choic
choicee of words
words,, senten
sentence
ce constr
construc
ucttion,
ion, and
and punct
punctuat
uation
ion.. Choo
Choose
se among
among the optio
options
ns
which
which revision
revision results
results in a sentence
sentence that is clear and precise.
precise.

40. Ally’s cat was finall


finally
y found
found shive
shiverin
ring
g under
under the
the stairs
stairs of the
the house
house covere
covered
d with
with mud
mud and
ravenously hungry.

(A) finall
finally
y found
found unde
underr thestairs
thestairs of the house
house,, shive
shiverin
ring,
g, cover
covered
ed with
with mud and
and
ravenously
ravenously hungry.
(B) found
found finall
finally
y shive
shiverin
ring
g covere
coveredd with
with mud
mud and raveno
ravenoususly
ly hung
hungry
ry unde
underr the stairs
stairs
of the house.
house.
(C) found
found shive
shiverin
ring,
g, and
and covere
covered
d with
with mud
mud and raven
ravenous
ously
ly hungry
hungry unde
underr the stairs
stairs of                 
the house
house finall
finally.
y.
(D) No
No change
change

41.
41. If the gover
governme
nmentnt decid
decides.
es. To amend
amend the Const
Constitu
itutio
tion.
n. One
One of the chang
changes
es should
should be the
abolition
abolition of the unneces
unnecessary
sary and expensi
expensive
ve party-list
party-list system
system..

(A) If the gover


governme
nment
nt decid
decides;
es; to amend
amend the Consti
Constitut
tution
ion,, one of the chang
changes
es shoul
should
d
be the abolition
abolition of the unneces
unnecessary
sary and expensi
expensive
ve party-list
party-list system.
system.
(B) If the gover
governme
nment
nt decid
decides
es to amend
amend the Const
Constitu
itutio
tion,
n, one
one of the chang
changes
es shou
should
ld be
the abolition
abolition of the unneces
unnecessary
sary and expensiv
expensivee party-list
party-list system.
system.
(C)
(C) If the
the gove
govern
rnme
ment
nt deci
decide
des,
s, to amen
amend d the
the Cons
Consti
titu
tuti
tion
on,, one
one of the
the chan
change
gess shoul
should
d be
the abolition
abolition of the unneces
unnecessary
sary and expensiv
expensivee party-lis
party-listt system.
system.
(D) No change

42. “Death was


was close
close that
that we were
were alread
already
y crying
crying and prayin
praying
g the rosary
osary,”
,” one
one of the ship
ship
disaster victims narrated.

(A)
(A) was
was too clos
closee that
that we were
were alre
alread
adyy cryi
crying
ng and
and pray
prayin
ing
g
(B)
(B) was
was so clos
closee than
than we were
were alre
alread
ady
y cryi
crying
ng and
and pray
prayin
ingg
(C)
(C) was
was so clos
closee that
that we
we were
were alre
alread
ady
y cryi
crying
ng and
and pray
prayin
ingg
(D) No change

43.
43. Driv
Drivin
ing
g alon
along
g the
the Imus
Imus Hig
Highw
hway
ay,, two
two cows
cows cro
cross
ssed
ed in fron
frontt of me.
me.

(A)
(A) Cros
Crossi
sing
ng in fron
frontt of
of me,
me, I saw
saw two
two cows
cows,, while
while driv
drivin
ing
g alon
along
g the
the Imus
Imus High
Highwaway.
y.
(B)
(B) As I was
was dri
drivi
ving
ng alon
alongg the
the Imus
Imus Hig
Highw
hway ay,, two
two cows
cows cro
cross
ssed
ed in fron
frontt of me.
me.
(C)
(C) Two
Two cows
cows cros
crosse
sedd in fron
frontt of me whil
whilee driv
drivining
g alon
along
g the
the Imus
Imus High
Highwaway.
y.
(D) No change

PRACTICE BOOKLET
BKL PhiLSAT_AA_0317 GO ON TO
TO THE NEX
NEXT
NEXT
XT PAGE
PA
PAGE
AG
GE
E
10

44. Most compress


compressors
ors operate
operate rather
rather simply
simply it cranks
cranks up energy
energy to cool your
your refrigerato
refrigerator r 
using
using the same
same amou
amountnt of pow
power
er all
all day long
long but
but that
that is not the
the case
case with
with ABC
ABC’s
Inverter Compressor.

(A) Most
Most com
compre
presso
ssors
rs oper
operate
ate rather
rather simply
simply.. It cran
cranks
ks up
up energy
energy to cool
cool your 
refrigerato
refrigeratorr using
using the same amount
amount of power
power all all day long but that
that is not
the
the case
case with
with ABC
ABC’s Inverter Compressor.
(B) Most
Most com
compre
presso
ssors
rs oper
operate
ate rather
rather simply
simply.. It cran
cranks
ks up energy
energy to cool
cool your 
your 
refrigerato
refrigeratorr using
using the same amount
amount of power
power all all day long,
long, but that
that is not
the
the case
case with
with ABC
ABC’s Inverter Compressor.
Compressor.
(C) Most
Most com
compre
presso
ssors
rs oper
operate
ate rather
rather simply
simply,, it cran
cranks
ks up energ
energyy to cool
cool your 
your 

refrigerato
refrig
the caerator
the case wirthusing
se with usin
ABgCthe
ABC same amount
amou
’s Inverter  nt of power
power all
Compressor. all day long but that
that is not
(D) No change

45.
45. My moth
mother
er ask
asked
ed me why
why I did
did not tak
takee more
more pict
pictur
ures
es of the
the chil
childr
dren
en play
playin
ing
g in
the park?
park?

(A) “why
“why I did
did not
not take
take more
more pictur
pictures
es of
of the
the child
children
ren playin
playing
g in the park.
park.””
(B) “why
“why I did
did not
not take
take more
more pictur
pictures
es of
of the
the child
children
ren playin
playing
g in the park?
park?””
(C)
(C) why
why I did
did not
not take
take more
more pict
pictur
ures
es of the
the chil
childr
dren
en play
playin
ing
g in the
the park
park..
(D) No change

46.
46. Actu
Actual
ally
ly,, flyi
flyinglem
nglemur
urss don’t
don’t fly they
they just
just glid
glidee from
from tree
tree to tree.

(A) don’t fly and


and glid
glidee from
from tree
tree to tree
tree
(B) don’t
don’t fly
fly but
but glid
gliding
ing from
from tree
tree to tree
tree

(C)
(D) don’t
No chfly;
angethey   just glide from tree to tree

47.
47. Drivin
Driving
g too slowly
slowly along
along SLEX
SLEX last
last Satur
Saturday
day.. A highwa
highway
y patrol
patrol office
officerr stopp
stopped
ed
Miss Perez.

(A)
(A) Miss
Miss Pere
Perezz was
was stopp
stopped
ed by a high
highway
way patr
patrol
ol offi
office
cerr drivi
driving
ng too
too slow
slowly
ly alon
along
g
SLEX last Saturday.
(B)
(B) Driv
Drivin
ing
g too
too slow
slowly
ly alon
along
g SLEX
SLEX last
last Satu
Saturd
rday
ay,, Miss
Miss Pere
Perezz was
was sto
stopp
pped
ed by a
highway patrol officer.
(C)
(C) Driv
Drivin
ing
g too
too slow
slowly
ly alon
along
g SLEX
SLEX last
last Satur
Saturda
day,
y, a high
highwawayy patr
patrol
ol offic
officer
er stopp
stopped
ed
Miss Perez.
(D) No change

48.
48. When
Whenev
ever
er he arri
arrive
vess from
from work
work,, Alex
Alex who
who lives
lives alon
alonee in a cond
condom
omin
iniu
ium
m unit,
unit, heat
heatss his
precooked dinner.

(A)
(A) Alex
Alex,, who
who live
livess alone
alone in a cond
condom
omin
iniu
ium
m unit,
unit, heat
heatss his pre
preco
cook
oked
ed dinn
dinner
er..
(B)
(B) Alex
Alex who
who live
livess alone
alone in a cond
condom
omin
iniu
ium
m unit
unit heat
heatss his pre
preco
cook
oked
ed dinn
dinner
er..
(C)
(C) Alex
Alex,, who
who live
livess alone
alone in a cond
condom
omin
iniu
ium
m unit
unit heat
heatss his prec
precoo
ooke
ked
d dinn
dinner
er..
(D) No change

PRACTICE BOOKLET
BKL PhiLSAT_AA_0317 GO ON TO
TO THE NEX
NEXT
NEXT
XT PAGE
PA
PAGE
AG
GE
E
11

49.
49. My mothe
motherr is a woman
woman of many
many talen
talents.
ts. She
She plays
plays the harp,
harp, bakes
bakes the best
best choco
chocolat
latee
cakes,
cakes, and you
you should
should see her
her orchid
orchidss and roses.
roses.

(A) There’s
There’s her talent
talent for
for playin
playing
g the harp,
harp, and for baking
baking the best chocola
chocolate
te cakes,
cakes,
and then
then there
there are the orchid
orchidss and
and roses.
roses.
(B) She
She plays
plays the harp,
harp, bakes
bakes the best
best choco
chocolat
latee cakes,
cakes, and grows
grows orchid
orchidss and roses.
roses.
(C)
(C) She
She is pla
playi
ying
ng the
the harp
harp,, baki
baking
ng the
the best
best cho
chococola
late
te cake
cakes,
s, and
and for
for grow
growin
ing
g orch
orchid
idss
and roses.
roses.
(D) No change

50.
50. After
After Anton
Antonius
ius Strad
Stradiva
ivariu
riuss learne
learned
d to play
play the violin
violin,, he experi
experime
mente
nted
d making
making a violin
violin

that
that had the soun
sound
d of a human
human voice
voice,, and succe
success
ss was
was achiev
achieved.
ed.
(A) he succeed
(B) he succeeded
(C)
(C) hewas
hewas suc
success
cessfu
full
(D) No change

PRACTICE BOOKLET
BKL PhiLSAT_AA_0317 GO ON TO
TO THE NEX
NEXT
NEXT
XT PAGE
PA
PAGE
AG
GE
E
12

TEST
TEST B. CRITICAL
CRITICAL THINKING
THINKING

Section
Section 1. Logical
Logical Reason
Reasoning
ing

Directions
Directions:: In this section,
section, brief conversa
conversations
tions or short
short passages
passages are given
given followed
followed by one or 
two
two ques
questi
tion
ons.
s. Thes
Thesee ques
questi
tion
onss ask
ask for
for the anal
analysysis
is and
and eval
evalua
uati
tion
on of the
the reas
reason
onin
ing
g in the
conversa
conversation
tion or passag
passage.e. It is conceiv
conceivable
able that all
all of the four choic
choices
es given
given for each
each question
question may
  be correc
correctt answers
answers.. The task
task is to sel
select the
the best
best answer
answer for
for each
each quest
question
ion.. The best
best answer
answer is the
the
one which
which does not
not require
require assumptio
assumptions ns that are irrelevant
irrelevant or inconsiste
inconsistent
nt with the statement
statements.
s.

1. About 90% of the world’s


orld’s populati
population
on growth
growth is occurring
occurring in the underde
underdevelo
veloped
ped and
devel
develop
oping
ing nation
nationss of the Third
Third World
World.. The
The popula
populatio
tion
n growth
growth rates
rates in these
these poor 
poor 
countries
countries make it diffic
difficult
ult to provide
provide the bare necessitie
necessitiess of
of housi
housing,
ng, fuel,
fuel, food,
food, and
medical
medical attention
attention.. Ironicall
Ironically,
y, there is a relations
relationship
hip between
between poverty
poverty and fertility:
fertility:
The greater
greater the proportio
proportion n of a given
given populati
population
on living
living in poverty,
poverty,___
______
______
______
_____.
__.

(A)
(A) the
the high
higher
er is the
the unem
unempl ploy
oyme
ment
nt rate
rate in the
the coun
countr
try
y
(B) the highe
higherr is the fertil
fertility
ity rate
rate of that
that countr
country y
(C) the great
greater
er is the tende
tendency
ncy toward
towardss disside
dissident
nt activi
activitie
tiess
(D) the great
greater
er is the
the propo
proportio
rtion
n of illega
illegall and crimin
criminal
al incid
inciden
ents
ts
  ______________________________________________________________________________ 

Jason: You
You should
shouldn’t
n’t smoke
smoke so much,
much, Honey
Honey.. Smok
Smoking
ing is not really
really good for your
your lungs.
lungs.
Kathy: Darlin
Darling,
g, I don’t
don’t agree
agree with
with you.
you. I have
have been
been smokin
smoking g for twenty
twenty years
years now
now,, and
and I
have not developed
developed emphysem
emphysema.a.

2. Whic
Which
h of the foll
follow
owin
ing
g resp
respon
onse
sess would
would best
best stren
strengt
gthe
hen
n and
and expla
explain
in Jaso
Jason
n’s argument?

(A) If you
you keep
keep smokin
smoking,g, you
you may find
find that
that you
yourr cigare
cigarette
tte consu
consump
mptio
tion
n will
will keep
keep
increa
increasin
singg in the future
future..
(B) Smok
Smoking
ing does
does not
not always
always devel
develop
op emphy
emphysemsema.a.
(C) Deve
Develop
loping
ing emphy
emphysemsemaa is not
not the only reaso
reason n why
why smoki
smoking
ng is not
not good
good for 
a person.
person.
(D) Many
Many perso
persons
ns who
who smoke
smoke as much
much as Kath
Kathyy does
does deve
develop
lop emphy
emphysem
sema.
a.
  ______________________________________________________________________________ 

For
For items
items 3 and
and 4, refer
refer to the follow
following
ing passa
passage
ge..

Today’s
Today’s methods
methods of discip
discipline
line are a far cry from those
those usually
usually employed
employed just twenty
twenty (20)
year
yearss ago.
ago. In the
the past
past,, the
the maxi
maxim m “spa
“spare
re the
the rod,
rod, and
and spoi
spoill the
the chil
child”
d” has
has been
been take
taken
n lite
litera
rall
lly
y
that
that corp
corpora
orall pun
punish
ishmen
mentt was
was the norm
norm of the day.
day. Span
Spankin
king,g, kneel
kneeling
ing on mongo
mongo seeds,
seeds, or 
stand
standing
ing unde
underr the heat
heat of the sun werewere some
some measu
measures
res taken
taken to ensur
ensuree that
that the child
child would
would
grow upright.
upright. This behavio
behaviorist
rist assum
assumption
ption was the
the norm of the day until research
research has exposed
exposed

the
internegative
nega
internatiotive
nal psychol
national psychologic
laws banogical
thealsaid
effects
effect s of such
practices
prac tices kinds
kind
and s ofprescribe
now corrective
correcti
pres cribeve
an behavio
beh avior.
r. e,
alternativ
alter Current
Curr
native, enthnational
which
whic natio nal andy
is popularl
pop ularly
know
known n as posi
positi
tive
ve disc
discip
ipli
line
ne.. In this
this styl
stylee of disc
discip
ipli
line
ne,, chil
childr
dren
en are
are made
made to real
realiz
izee
the conseque
consequencesnces of
of their wrong
wrongdoin
doing.g. Counselli
Counselling
ng and withdraw
withdrawinging of privilege
privilegess have now
replac
replaced
ed the ire of the beatin
beatingg stick
stick..

PRACTICE BOOKLET
BKL PhiLSAT_AA_0317 GO ON TO
TO THE NEX
NEXT
NEXT
XT PAGE
PA
PAGE
AG
GE
E
13

3. Whic
Which
h of the
the foll
follow
owin
ing
g assu
assump
mptio
tions
ns can
can be safe
safely
ly assu
assume
med
d as true
true??

(A)
(A) Chil
Childr
dren
en born
born at the time
time of the writi
writing
ng of the artic
article
le are
are most
mostly
ly undi
undisc
scip
ipli
line
ned.
d.
(B) There
There were
were no forms
forms of corpo
corporal
ral punis
punishme
hment
nt employ
employed ed from
from twent
twenty
y years
years ago until
until
the time
time of the articl
articlee’s writing.
(C)
(C) Thos
Thosee who
who were
were born
born twen
twentyty year
yearss and
and prio
priorr to
to that
that have
have expe
experie
rienc
nceded some
some form
form
of corporal punishment.
(D) Most
Most of those
those who
who havehave unde
undergo
rgone
ne corpo
corporal
ral pun
punish
ishmen
mentt exhibi
exhibitt psych
psycholo
ologic
gical
al
disorders.

4. Whic
Which
h of the
the defi
defini
niti
tion
onss belo
below
w may
may be said
said to be stra
strayi
ying
ng from
from the
the idea
ideass comm
commun
unic
icat
ated
ed

in the passag
passage?
e?
(A)
(A) Posit
ositiv
ivee disc
discipipli
line
ne is anyany form
form of dis
disci
cipl
plin
inee that
that yie
yield
ldss posi
positi
tive
ve resu
result
ltss on a
child’s behaviour.
(B)
(B) Beha
Behavi
viou
ouri
rist
stss bel
belie
ieve
ve that
that good
good deed
deedss are
are to be rewa
reward
rded
ed whil
whilee bad
bad deed
deedss are
are to
  be punishe
punished.
d.
(C) Corpora
Corporall punishm
punishment ent is a form of disciplin
disciplinee that involves
involves physical
physicallyly hurting
hurting a child
to correct behaviour.
behaviour.
(D)
(D) A maxi
maximm is a popu
populalarr sayin
sayingg whic
which
h also
also serv
serves
es as a guid
guidin
ing
g prin
princi
cipl
ple.
e.
  ______________________________________________________________________________ 

Since
Since all the
the floors
floors in
in the buildi
building
ng I have
have inspec
inspected
ted have
have marbl
marblee tiles,
tiles, it follow
followss that
that
the other
other floors
floors whic
whichh I have
have not
not seen
seen will
will also have
have marble
marble tiles.
tiles.

5. Whic
Which
h of the
the foll
follow
owin
ing
g have
have a para
parall
llel
el rea
reaso
soni
ning
ng to the
the stat
statem
emen
entt abov
above?
e?

(A) Ever
Ev ery
chur y town
church
chto
fawn
faci I have
cing
ng havpleaza
a plazvisi
viasite
ted
dsthas
must
mu has aa chur
be ch urch
town
town.ch
. faci
facing
ng a plaz
plaza;a; so a comm
commun unity
ity with
with a
(B) Some
Some newspa
newspaperperss are not
not selling
selling very
very well;
well; there
therefor
fore,
e, tabloi
tabloids
ds are
are not
not sellin
selling
g
very well.
(C) Since
Since the car deale
dealerr does
does not
not buy
buy cars
cars olde
olderr than
than five
five years
years old,
old, his new purcha
purchase
se
will
will proba
probably
bly be no older
older than
than five
five years
years old.
old.
(D)
(D) Sinc
Sincee all
all of the chil
childr
dren
en I know
know fear
fear monk
monkeyeys,s, ther
theree must
must be some
someththin
ing
g in the
monkeys that scare children.
  ______________________________________________________________________________ 

6. If a boy
boy grew
grew up in Mani
Manila
la,, then
then he is stre
street
et smar
smart.
t. This
This stat
statem
emen
entt can
can be deduc
deduced
ed
logically
logically from which
which of the following
following statement
statements?
s?

(A)
(A) Ever
Everyy stre
street
et smar
smartt boy
boy grew
grew up in Mani
Manila la or anoth
another
er city
city..
(B) The
The boy
boy in the statem
statemen
entt is either
either stree
streett smart
smart or app
appear
earss remar
remarka
kably
bly to be
street smart.
(C)
(C) Ther
Theree is no boy
boy who
who grew
grew up in Mani
Manilala who
who is not
not str
stree
eett smar
smart.
t.
(D)
(D) The
The majo
majoririty
ty of boys
boys who
who grew
grew up in Mani
Manila la are
are stre
street
et smar
smart.
t.

PRACTICE BOOKLET
BKL PhiLSAT_AA_0317 GO ON TO
TO THE NEX
NEXT
NEXT
XT PAGE
PA
PAGE
AG
GE
E
14

7. Food
Food prod
producucti
tion
on in the
the Thir
Third
d Worl
World d nati
nation
onss is
is mor
moree adve
advers
rsel
ely
y affe
affect
cted
ed by natu
natura
rall
disasters
disasters,, like floods
floods and
and droughts
droughts,, than it is in more affluent
affluent nations
nations becaus
becausee affluent
affluent
nations
nations are likely to have ______
__________
_____________
____._.

(A) mode
modernrn instru
instrume
ments
nts for accurat
accuratee foreca
forecasti
sting
ng of weathe
weather r 
(B) adequ
adequate
ate flood
flood contr
control,
ol, irriga
irrigatio
tion
n system
systems,s, and storag
storagee facili
facilitie
tiess
(C) enou
enough
gh agric
agricult
ultura
urall scient
scientist
istss and
and techn
technolo
ologis
gists
ts in food
food resear
research
ch and produ
producti
ction
on
(D) the neces
necessar
sary
y aircra
aircraft,
ft, pilots
pilots,, chemi
chemical
cals,
s, and equipm
equipmentent for
for undert
undertaki
aking
ng cloud
cloud
seedi
seeding
ng to induce
induce rain
rain

8. Doct
Doctor
orss and
and nurs
nurses
es are,
are, for
for the
the most
most part
part,, tota
totally
lly comm
committ
itted
ed to savi
saving
ng lives
lives.. A life
life lost
lost

is, fornasia
Eutha
Euthanasthem,
them
ia ,asalmost
almo stctice
a personal
a practi
pra pers
ce onal
ht failure,
might
mig failu
wellre, anthis.
alter
alter insult
insu
this. lt to their
It could
coul skills
skill
d have
hav e as corrup
and
andruptin
cor knowled
kno wledge.
ting infge.
g influe
luence
nce,, so
that
that in severe
severe cases,
cases, docto
doctors
rs and
and nurse
nursess might
might not
not try hard
hard enoug
enough h to save
save the patie
patient.
nt.
They
They might
might decid
decidee that
that the patient
patient would
would simply
simply be “bett
“better
er off
off dead”
dead” and take
take the steps
steps
neces
necessar
sary
y to make
make that
that come about
about.. This
This attitud
attitudee could
could then
then carry
carry over
over to their deali
dealing
ngss
with patients
patients less seriousl
seriouslyy ill. This would
would probably
probably result
result to _______
________________
_________
______..

(A) loss
loss of confid
confidenc
encee in the medica
medicall profe
professi
ssion
on
(B) nume
numerou
rouss cases
cases filed again
against
st doctor
doctorss and nurse
nursess
(C)
(C) an over
overal
alll decl
declin
inee in the
the qual
quality
ity of med
medic
ical
al car
caree
(D)
(D) a publ
public
ic outc
outcry
ry agai
agains
nstt euth
euthan
anas
asia
ia
  ______________________________________________________________________________ 

For
For items
items 9 and
and 10,
10, refer
refer to the follow
following
ing situa
situatio
tion.
n.

A conversa
conversation
tion between
between parents
parents of schoolch
schoolchildre
ildren:
n:

Sam:
Sam: Have
Have you
you heard
heard the news?
news? TheThe Princi
Principa
pall told
told me they
they are raisin
raising
g school
chool fees
fees nex
nextt year
year..
Pat: They’re heartless!
Sam: Well,
Well, accord
according
ing to him,
him, they’r
they’ree planni
planning
ng to put
put up a newscience
newscience labora
laborator
tory.
y. I alsoheard
they’re
they’re hiring
hiring more teachers
teachers next
next year.
year.
Pat: But why
why now?
now? Don’t
Don’t they
they realize
realize that prices
prices of goo
goods contin
continue
ue to inflat
inflate?
e? Do they
they want
want
us to suffer?
suffer?
Sam: I don
don’t
’t think
think it’s that,
that, altho
althougugh
h I hav
havee to admit
admit sud
sudde
denn increa
increases
ses can be worri
worriso
some.
me.
I was thinking
thinking of writing
writing a letter
letter asking for increment
incremental al increases.
increases.
Pat:
Pat: I am not sure
sure about
about that,
that, thoug
though.h. You
You see,
see, I got
got to talk
talk to one
one of the teach
teachers
ers.. His salary
salary
didn’t
didn’t increas
increasee in years!
years!

9. Whic
Which
h of the
the foll
follow
owin
ing
g is the
the main
main caus
causee of tens
tensio
ion
n in the
the conv
conver
ersa
sati
tion
on abov
above?
e?

(A) The teachers


teachers demand
demand higher
higher salaries
salaries in exchange
exchange for better
better teaching
teaching practices
practices..
(B) The
The impen
impendin
dingg increa
increase
se in tuitio
tuition
n fees
fees create
createss anxiet
anxiety
y among
among the paren
parents.
ts.
(C) Parents
Parents feel that the school
school is
is indiffer
indifferent
ent to their
their needs.
needs.
(D) The
The expert
expertise
ise of schoo
schooll employ
employees
ees is being
being quest
question
ioned
ed..

PRACTICE BOOKLET
BKL PhiLSAT_AA_0317 GO ON TO THE NEXT PAGE
15

10. Which
Which of the follow
following
ing argum
argument
entss is the LEAS
LEAST
T relev
relevant
ant to the issue
issue discus
discussed
sed in
the dialogue?
dialogue?

(A) The
The increa
increase
se in schoo
schooll fees
fees is
is justi
justifie
fied
d by the estab
establis
lishme
hment
nt of
of a new
new labora
laborator
tory.
y.
(B) An increa
increase
se in schoo
schooll fees
fees should
should trans
translat
latee to better
better salari
salaries
es for
for teach
teachers
ers and
and staff.
staff.
(C) Scho
School
ol fees
fees shou
should
ld not incre
increase
ase becaus
becausee paren
parents ts are burde
burdene
nedd by inflat
inflation
ion..
(D) Gradual
Gradual increa
increases
ses in school
school fees can
can help ease the parents’
parents’ burde
burden.n.
 _____________
___________________
__________________
_________________
_____________
_____________
__________________
__________________
____________
__________ 
____ 

Rizza:
Rizza: All
All the gradu
graduate
atess from
from Schoo
Schooll A go to Colle
College
ge Z.
Gary: I’m not
not sure.
sure. Some
Some of the
the stude
students
nts at Colle
College
ge Z come
come from
from Scho
School
ol B.

11. Gary’s
Gary’s response
response means
means that he has interpret
interpreted
ed Rizza’s
Rizza’s statemen
statementt to mean that 
that  _______.

(A) School
School A is better
better than School
School B
(B) most
most of the students
students from School
School A attend
attend College
College Z
(C) some
some School
School B graduate
graduatess do not attend
attend college
college
(D) only students
students from School
School A attend
attend College
College Z
  ______________________________________________________________________________ 

The deman
demandd that
that the death
death penalt
penalty
y be prove
provenn more
more deter
deterren
rentt than
than altern
alternati
ative
vess cannot
cannot be
satisf
satisfied
ied any
any more
more than
than the deman
demand d that
that six
six years
years in prisonbe
prisonbe prove
proven n to be more
more deterr
deterrent
ent than
than
three.
three. But the uncertain
uncertainty
ty which
which confron
confronts
ts us favors
favors the death
death penalty
penalty as long as, by imposing
imposing
it, we might
might save future
future victims
victims of heino
heinous
us crimes
crimes..

12.
12. Whic
Which
h of the
the foll
follow
owin
ing
g is the
the main
main poin
pointt of the pass
passag
age?
e?

(A) Imposing
Impo sing the
of heinous death
death penalty
crimes. penalty as punishm
punishmentent has been instrumen
instrumental
tal in the reduction
reduction
(B) It is difficult
difficult to
to prove
prove whether
whether or not
not the death
death penalty
penalty is a better
better deterren
deterrentt to
heinous crimes
crimes than other modes of penalty.
penalty.
(C) It has
has been
been satisf
satisfact
actori
orily
ly prove
provenn that
that impo
imposit
sition
ion of the death
death penal
penalty
ty is a bette
better r 
deterrent to heinous crimes than imprisonment.
imprisonment.
(D) There
There is no alternative
alternative to capital
capital punishme
punishment nt as
as a deterrent
deterrent to heinous
heinous crimes.
crimes.
  ______________________________________________________________________________ 

For
For items
items 13
13 and
and 14,
14, refer
refer to the follow
following
ing speec
speech.
h.

My dear
dear gra
gradu
duatates
es,, I impl
implor
oree you
you at thi
thiss mome
moment nt to
to use
use your
your tal
talen
entsts and
and skil
skills
ls in
improv
improving
ing our
our mothe
motherlarland.
nd. I beg
beg of you
you:: if you
you really
really do love
love this
this coun
country
try,, then
then you will
will not
not
work
work or do business
business elsewhere
elsewhere.. Don’t
Don’t you
you know
know that there are millions
millions of
of starvin
starving g mouths
mouths toto
feed
feed in our
our borde
borders
rs?? Take
Take a look
look at Cuba
Cuba.. For
For years
years,, her
her doors
doors were
were clos
closed
ed from
from the
the outs
outsid
idee
worl
world,
d, and
and yet,
yet, she
she is abl
ablee to prov
provid
idee free
free medi
medicacall serv
servic
ices
es to
to her
her citiz
citizen
ens.
s. If Cub
Cubaa made
made it,
then
then so can
can we.

I am
am sure that if you use your
your creati
creativity
vity and passion
passion in our country,
country, things
things will surely
surely get
better.
better. Whether
Whether you
you admit
admit it or not,
not, working
working elsewhere
elsewhere equals
equals abandonmen
abandonmentt of the land that
fed you for the longest
longest time.

PRACTICE BOOKLET
BKL PhiLSAT_AA_0317 GO ON TO
TO THE NEX
NEXT
NEXT
XT PAGE
PA
PAGE
AG
GE
E
16

13. Which
Which of the follow
following
ing statem
statement
entss accurat
accurately
ely depic
depicts
ts the bias
bias of the gradu
graduati
ation
on speake
speaker?
r?

(A) Inves
Investin
ting
g locall
locally
y is the best
best way to help
help the poor
poor of the speake
speaker’s
r’s coun
country
try..
(B) The
The gradu
graduate
atess will have
have to study
study and employ
employ Cuba
Cuba’s ’s examp
examplele for their
their count
country
ry to
improve.
(C) Leavi
Leaving
ng the count
countryry is fine
fine as long
long as one
one return
returnss to rende
renderr serv
service
ice or goods
goods..
(D) Love
Love for countr
countryy canno
cannott be expres
expressed
sed if one
one goes
goes abroa
abroadd to work.
work.

14. Which
Which of the statem
statemen
ents
ts below
below depic
depicts
ts a logica
logicall flaw
flaw in the speech
speech??

(A) The
The ones
ones who
who are gradu
graduati
ating
ng posses
possesss talent
talentss and
and skills
skills..

(B) Cuba
(C) Those
Those
Cu who
who
ba has leave
leaessful
succ ve fully
success thelycountr
country
made
mad eyavail
toailabl
av work
wo rke are
able not
notalpatrio
medic
medical pat riotic
tic.
servic
ser . to her
vices
es her citizens
citizens..
(D)
(D) Ther
Theree are
are seve
severa
rall poor
poor peopl
peoplee in the
the coun
countrtry
y wher
wheree the
the spea
speake
kerr and
graduates belong.
  ______________________________________________________________________________ 

Most philosoph
philosophersers agree that punishmen
punishment, t, in general,
general, is a morally
morally justified
justified social
social practice.
practice.
However
However uneasy
uneasy we might
might feel
feel about
about inflictin
inflictingg harm on another
another person,
person, it is hard to visualize
visualize
a _____
_______
________
____
__ manag
managinging to surviv
survivee witho
withoutut an establ
establish
ished
ed legal
legal syste
systemm of punis
punishm
hment
ent..
On the other
other hand
hand,, dissen
dissenter
terss argue
argue that
that it is possibl
possiblee to struct
structure
ure socie
society
ty in ways
ways that
that would
would
not necess
necessitate
itate commitmen
commitmentt to a legal
legal system
system of punishm
punishment.
ent. For examp
example,le, might
might it not be that
undesirab
undesirable
le social
social behavior
behavior could
could be adequate
adequately ly kept in check
check by therapeut
therapeuticic treatment
treatment rather 
rather 
than by tradition
traditional
al kinds
kinds of punishm
punishmentent such as imprisonm
imprisonment ent or the death
death penalty?
penalty? Such
Such a
system
system would
would certa
certainl
inly
y have
have the advan
advantag
tagee of being
being more
more human
humane. e.

15.
15. Whic
Which
h of the
the foll
follow
owin
ing
g best
best fit
fitss in the
the blan
blank?
k?

(A) despotic
despotic government
government
(B) multiracial population
(C) developing
developing country
(D) complex
complex society
society
  ______________________________________________________________________________ 

If you
you believ
believee that
that all peop
people
le are basica
basically
lly good
good and have
have poten
potentia
tial,
l, you
you can be led down
a lot
lot of path
pathss to the
the orga
organiniza
zati
tion
onal
al aby
abyss
ss.. You
You assu
assume
me that
that a bett
better
er env
envir
iron
onme
ment
nt,, more
more
empowerm
empowerment,ent, and juicier
juicier assig
assignmen
nments ts will
will do
do the trick.
trick. These
These are wonderfu
wonderfull concep
concepts,
ts, but if                 
the perso
personn and the job are misma
mismatchtched,
ed, you
you are desti
destined
ned for disapp
disappoin
ointme
tment.
nt.

16.
16. Whic
Which
h of the foll
follow
owin
ing
g woul
would
d be the
the best
best compl
complet
etio
ion
n of the pass
passag
agee abov
above?
e?

(A) No one can be success


successful
ful unless
unless that person
person is reasona
reasonably
bly matched
matched with what
what
he or she is expec
expectedted to do.
do.
(B) A person
person’s’s total
total dedica
dedication
tion is a sufficien
sufficientt indicati
indication
on that there
there is a match
match between
between
that
that perso
personn and
and the job.
job.
(C) Lack
Lack of intere
interest
st is the best
best indica
indicatio
tion
n of a mismat
mismatch ch betwe
between
en a perso
person
n and
and his
or her job.
(D) Bias in promotio
promotion n is a very sensit
sensitive
ive indicator
indicator of mismat
mismatches
ches between
between employee
employeess
and their jobs.
jobs.

PRACTICE BOOKLET
BKL PhiLSAT_AA_0317 GO ON TO THE NEXT PAGE
17

For
For items 17
17 and
and 18,
18, refer
refer to the follow
following
ing passag
passage.
e.

One of my
my biggest
biggest qualm
qualmss is the futility
futility of conv
convincin
incingg my students
students to read.
read. Today’s
Today’s kids
seem
seem to convu
convulse
lse at the idea
idea of going
going throu
through
gh a book
book until
until its end.
end. What’
What’ss wors
worse,
e, in my thirty
thirty
years
years of
of teach
teaching
ing English,
English, it seems
seems students
students get more and more repulsed
repulsed to the idea of reading
reading
as time goes on.
on.

I now
now appre
apprecia
ciate
te the fact
fact that
that my
my family
family didn’t
didn’t own
own a TV set when
when I was
was young
young.. I had
had to
  be imagin
imaginati
ative
ve to pass
pass time
time   – that’s
that’s why
why I read.
read. Readin
Readingg has
has taug
taught
ht me to imagin
imaginee and think
think..
I have
have always
always marve
marvelle
lled
d at how
how words
words in a page
page can take
take me to place
places,
s, even
even world
worldss I had
had never 
never 
fancie
fanciedd in my wilde
wildest
st drea
dreams
ms.. Too bad
bad many
many kids
kids nowa
nowaday
dayss are
are hook
hookeded into
into digita
digitall devic
devices
es

that
that can
can show
show them
them ever
everyt
ythi
hing
ng in one
one clic
click.
k. Yup,
Yup, no more
more time
time to read
read..
17.
17. Base
Based
d on the
the pass
passag
agee abov
above,
e, whic
which
h of the
the foll
follow
owin
ing
g assu
assump
mptio
tions
ns may
may be consi
conside
dere
red
d as
absolutel
absolutely
y true?

(A) Televisio
Televisionn has not
not yet
yet been
been invented
invented at the time of the narrator’
narrator’ss childho
childhood.
od.
(B)
(B) All
All kids
kids born
born rece
recent
ntly
ly have
have no natu
natura
rall prope
propens
nsity
ity to read
read..
(C) Moder
Modern-n-day
day device
devicess preven
preventt kids from
from readin
reading.
g.
(D) Engli
English
sh has been
been taugh
taughtt as a staple
staple schoo
schooll subje
subject.
ct.

18. Which
Which of the following
following statement
statementss best encapsulate
encapsulatess the author’s
author’s main assumption?

(A) Reading
Reading is the best way to increase
increase a person’s
person’s intelligen
intelligence.
ce.
(B) Modern-day techn
technolo
ologie
giess contri
contribu
bute
te to the decli
decline
ne in a child’
child’ss intere
interest
st in readin
reading.
g.
(C) Informati
Information
on literacy
literacy is necessary
necessary to survive
survive in today’s
today’s world.
world.
(D) Read
Reading
ing habit
habitss are
are diffic
difficult
ult to estab
establis
lish
h once
once a child
child reach
reaches
es ado
adoles
lescen
cence.
ce.

  ______________________________________________________________________________ 
19. Judgin
Judging
g from
from the conte
content
nt of the
the statem
statement
ent and
and the autho
authorit
ritati
ativen
veness
ess of
of the speak
speaker,
er,
which
which of the following
following is the most reasonab
reasonable
le and trustwort
trustworthy?
hy?

(A) City
City Mayo
Mayor: r: The
The peso’s
peso’s recov
recovery
ery in the past
past few days shou
should
ld be a boom
boom to
indus
industry
try,, becau
because se a stron
strongg peso
peso makes
makes the Philip
Philippin
pineses an attrac
attractiv
tivee destin
destinati
ation
on for 
international investors.
(B) Policema
Policeman: n: Sex
Sex educatio
education n is the best means
means of addressi
addressingng sexual
sexual ignoranc
ignorancee and
will greatly
greatly help minimize,
minimize, if not totally
totally eradicate
eradicate,, sex offenses,
offenses, such as child
molestati
molestation,on, rape,
rape, and incest.
incest.
(C) Sociolo
Sociologist:
gist: Religion
Religion cuts across
across all
all segments
segments of society.
society. True believers
believers and those
those
merel
merelyy supers
superstit titiou
iouss seek
seek sanctu
sanctuary
ary in it. The
The masses
masses and the powerf
powerful,ul, simila
similarly
rly
fearful of external damnation, commingle in this fraternity.
(D) Lawyer:
Lawyer: One of the the most serious
serious signs
signs of the increase
increase in moral
moral corruptio
corruption n is
the bound
boundleslesss exaltat
exaltation
ion of sex.
sex. Sex
Sex has even
even invade
invaded d the field
field of educat
education
ion with
with
the help of mass
mass media
media..

PRACTICE BOOKLET
BKL PhiLSAT_AA_0317 GO ON TO
TO THE NEX
NEXT
NEXT
XT PAGE
PA
PAGE
AG
GE
E
18

20. Only Golden


Golden Harvest
Harvest Beer
Beer has a biting
biting efferves
effervescent
cent taste that gives
gives you that all-night
all-night
relaxed
relaxed feeling.
feeling. If this advertisi
advertising
ng claim is true, which
which of the following
following propositi
propositions
ons
CANN
CANNOT OT be true?
true?

I. Maha
Maharli
rlika
ka Beer
Beer is a mor
more popu
popular
lar beer
beer than
than Golde
Golden n Harve
Harvestst Beer.
Beer.
II. Sulta
Sultana
na Beer
Beer give
givess you that
that all-nig
all-night
ht relax
relaxed
ed feelin
feeling.
g.
III.
III. Golde
Goldenn Har
Harvest
vest Cola has that
that biting
biting efferv
effervesc
escent
ent taste
taste of the Golde
Goldenn
Harvest Beer.

(A) I and II only


(B) I and III only

(C)
(D) II II,
(D) I, and
II anIII
, and only
d III
III
  ______________________________________________________________________________ 

For
For items
items 21
21 and
and 22,
22, refer
refer to the follow
following
ing speec
speech.
h.

A privilege
privileged
d speech
speech by a lawmaker
lawmaker against
against the distributi
distribution
on of contrace
contraceptive
ptivess in schools:
schools:

Greeti
Greeting
ngs,
s, fellow
fellow citize
citizens
ns!! Today,
Today, I stand
stand before
before you
you to expres
expresss my disap
disappro
proval
val of the
the
  propose
proposedd distribut
distribution
ion of contra
contracept
ceptives
ives in schools
schools to prevent
prevent teenage
teenage pregnan
pregnancy.
cy. First
First of all,
schools
schools are built not
not just
just to train our kids
kids in academic
academicss but also
also to instil
instil positive
positive values
values among
among
the youth
youth.. Hence
Hence,, distri
distribu
butin
tingg contra
contracep
ceptiv
tives
es is tantam
tantamou
ount
nt to sugges
suggestin
ting
g to our
our youn
young g ones
ones
that promiscu
promiscuity
ity is permissib
permissible.le. Furtherm
Furthermore,
ore, these
these kids are sent to school
school by their parents
parents who
who
dream
dream of givi
giving
ng them
them the best
best life possi
possib
ble. Wouldn’t
Wouldn’t parents
parents also be dismayed
dismayed by this move?
move?
They
They sent
sent their
their kids
kids to schoo
schooll to study
study and not
not to engag
engagee in immora
immorall acts.
acts.

21. What form of argument


argumentation
ation did the lawmaker
lawmaker use in his speech?
speech?
(A) Post hoc   –      Two
          things
things happen
happening
ing one after another,
another, and it is assumed
assumed that these
two are connecte
connected. d.
(B) Straw man  –  Refuting Refuting an argumentargument by raising
raising an irrelevan
irrelevantt issue.
issue.
(C)
(C) Swee
Sweepi
ping
ng gene
genera raliz lizat
atio
ionn –  Comi
Coming
ng up with
with a conc
conclu
lusi
sion
on base
based d on a limit
limited
ed
number of examples.
(D)   Ad hominem 
hominem   –        Attack
Att
      acking
ing the perso
personn who
who propo
proposed
sed the argum
argument
ent instead
instead of the
argument itself.

22. Which
Which of the
the follow
following
ing statem
statement
entss is most
most logica
logically
lly soun
sound?
d?

(A) Schools
Schools are built not just for academic
academicss but
but also to nurture
nurture values.
values.
(B) Distribu
Distributing
ting contrace
contraceptive
ptivess promote
promote sexual
sexual promiscu
promiscuity.
ity.
(C) The
The gove
governm
rnmenentt becom
becomeses insensi
insensitiv
tivee to paren
parents
ts when contr
contrace
acepti
ptives
ves
are distribut
distributed.
ed.
(D) Parents
Parents will get angry
angry if contrace
contraceptiv
ptives
es are distribut
distributed
ed in schools.
schools.

PRACTICE BOOKLET
BKL PhiLSAT_AA_0317 GO ON TO THE NEXT PAGE
19

Civilian:
Civilian: All policeme
policemen n are corrupt.
corrupt.
Policeman
Policeman:: That is not true. I know
know some civilians
civilians who are corrupt,
corrupt, too.

23. The policeman’s response shows his interpretation


interpretation of the
the civilian’s statement to
mean
mean that
that

(A) some
some policeme
policemenn are corrupt
corrupt
(B) civilians
civilians are more corrupt
corrupt than policemen
policemen
(C) policeme
policemen n are more corrupt
corrupt than civilians
civilians
(D) only policemen
policemen are corrupt
corrupt
  ______________________________________________________________________________ 

One
One way
way to determ
determine
ine wheth
whether er or not
not a socia
sociall practic
practicee is moral
morally
ly corre
correct
ct is to see if it is
eith
either
er perm
permit
itte
ted
d or requ
requir
ired
ed by the
the prin
princi
cipl
plee of equa
equalit
lity.
y. To say
say that
that a soci
social
al prac
practi
tice
ce is
permit
permitted
ted is to say that
that its
its inst
institu
itutio
tionn will
will not
not violat
violatee the moral
moral princ
principl
iple.
e. To say that
that itit is
requ
requir
ired
ed is to say
say that
that we must
must inst
institu
itute
te such
such a prac
practitice
ce if we are
are to act
act in a mora
morally
lly corr
correcectt
way.
way. Accord
According
ing to that
that princi
principle
ple,, equal
equalss must
must be treate
treatedd as equal
equals,
s, while
while unequ
unequals
als must
must be be
treated
treated unequally
unequally,, in proportio
proportionn to their differenc
differences.
es.

24. The author


author of this passage
passage would
would most likely agree
agree that ______
__________
_______
______
______
______
_____.
__.

(A) race is relevant


relevant in the hiring
hiring of teachers
teachers in a predomin
predominantly
antly Chinese
Chinese
exclusiv
exclusivee school
school
(B) women
women cannot
cannot be as good as or better
better than men in politics
politics or diplomacy
diplomacy
(C) it is in the best
best interest
interest of society that social institutions be nonegalitarian
in order
order to avoid
avoid direc
directt competi
competitio
tion
n betwe
betweenen men and wome
women n
(D) young
young children
children cannot
cannot be given
given the same rights
rights and responsi
responsibiliti
bilities
es as adults
adults

______________________________________________________________________________
The area
area of value
valuess educat
education
ion has been
been fraug
fraught
ht with
with confu
confusio
sionn and diffic
difficult
ulty.
y. Not
Not long
long
ago,
ago, some
some educat
educators
ors insisted
insisted on values
values-fr
-free
ee educa
educatio
tion
n and pride
pridedd themse
themselvelvess on dealin
dealingg only
only
with
with fact
facts,
s, and
and not
not valu
values
es,, whic
whichh they
they cont
conten
ende
ded
d were
were the
the prov
provin
ince
ce of the
the ____
______ ____
____
____
___.
_.
The very act of of teaching
teaching,, however
however,, presupp
presupposes
oses its own
own set ofof values,
values, including
including for examp
example,
le,
achi
achiev
evem
emen
ent,t, nurt
nurtur
uran
ance
ce,, and
and the
the valu
valuee of educ
educatatio
ion
n itsel
itself.
f. To the
the exte
extent
nt that
that the
the teac
teache
her r 
interacts
interacts with students,
students, these
these values
values are being
being modeled.
modeled.

25.
25. Whic
Which
h of the
the foll
follow
owin
ing
g best
best fit
fitss in the
the blan
blank?
k?

(A) students themselves


(B) church
church and home
(C) parents only
(D) parish
parish priest
priest

PRACTICE BOOKLET
BKL PhiLSAT_AA_0317 GO ON TO
TO THE NEX
NEXT
NEXT
XT PAGE
PA
PAGE
AG
GE
E
20

Section
Section 2. Analytica
Analy
Analytical
ticall Reasoning
Reasonin
Reasoning
g

Direct
Direction
ions:
s: In this
this sectio
section,
n, some
some sets
sets of condit
condition
ionss or propo
proposit
sition
ionss are given.
given. Each
Each set is
follow
followed
ed by quest
question
ionss based
based on the cond
conditio
itions
ns or propo
proposit
sition
ions.
s. Draw
Drawing
ing a diagra
diagramm to visua
visually
lly
estab
establis
lish
h linka
linkages
ges or relati
relations
onship
hipss stat
stated
ed in the condit
condition
ionss may help
help in answe
answerin
ring
g some
some of the
question
questions.
s. Select
Select the best answer
answer to each question
question from the choices
choices given.
given.

For
For items
items 26 to 29, refer
refer to the follow
following
ing condit
condition
ions.
s.

I. Mr. Cruz
Cruz bough
boughtt a small
small printi
printing
ng busin
business
ess of his friend
friend who
who was
was migrat
migrating
ing to Canad
Canada.
a.

He neede
andne eded
hire
hir e adteam
to m
tea familia
famofiliariz
onrize
ly efour,
only himsel
him
fou r, self
allfof
owith
wi th the
f whom
who m business
busin ess,
are former
for , soemp
mer he decid
de
employcided
eesed
loyee s oftohis
start
stafrie
frtriend
on
ond.
n a. small
small scale
sca
He neede
ne le
ededd
at leas
leastt two
two to operoperat
atee the
the prin
printi
ting
ng pres
presss mach
machinine,
e, with
with the
the othe
otherr two
two as bind
binder 
er 
and collator.
collator.
II. The applic
applican ants
ts for operat
operator
or are Simon
Simon,, Josh,
Josh, and Mike.
Mike. The
The appli
applicacants
nts for binder
binder and
collat
collator
or are Eric,
Eric, Charl
Charlie,
ie, Fred,
Fred, and
and Harry.
Harry.
III.
III. Duri
During
ng a talk
talk with
with his
his frien
friend,d, Mr.
Mr. Cruz
Cruz foun
found d out
out that
that Fred
Fred and
and Harry
arry do notnot work
ork well
ell
toge
togeth
ther
er,, Josh
Josh disl
dislik
ikes
es Eric
Eric,, and
and Simo
Simon n has
has a long
long-s-sta
tand
ndin
ing
g feud
feud with
with Fred
Fred.. He must
must
therefore
therefore choose
choose the right
right combina
combinationtion of four people
people to avoid
avoid future
future frictions
frictions..

26. If Harry
Harry is chosen
chosen,, which
which of the follow
following
ing combi
combinat
nation
ionss CANN
CANNOT
OT be chosen
chosen to form
form
the team?
team?

(A)
(A) Simon
imon,, Josh
Josh,, and
and Char
Charli
liee
(B)
(B) Simo
imon, Mike
ike, and
and Eric
ric
(C)
(C) Simon
imon,, Mike
Mike,, and
and Char
Charli
liee

(D) Josh,
Josh, Mike,
Mike, and Eric
Eric
27. If Fred
Fred is chose
chosen,
n, who
who among
among the follow
following
ing groups
groups of appli
applica
cants
nts shou
should
ld be employ
employed
ed as
the three other
other member
memberss of the team?

(A)
(A) Josh
Josh,, Mike,
ike, and
and Char
Charli
liee
(B)
(B) Mike,
ike, Eric
Eric,, and
and Char
Charli
liee
(C)
(C) Harry
arry,, Mike
ike, and
and Eric
Eric
(D)
(D) Simon
imon,, Josh
Josh,, and
and Char
Charliliee

28. If Josh
Josh is hired
hired and
and Mike
Mike is not,
not, which
which of the follow
following
ing statem
statement
entss must
must be true?
true?

I. Harry
arry will
ill be a memb
member
er of the
the team
team..
II.
II. Char
Charli
liee will
will bea memb
member
er of the
the team
team..

(A) I only
(B) II only
(C) Both I and II
(D) Neith
Neither
er I nor
nor II

PRACTICE BOOKLET
BKL PhiLSAT_AA_0317 GO ON TO THE NEXT PAGE
21

29.
29. If Eri
Ericc is chos
chosen
en as a bind
binder
er,, whic
which
h of the foll
follow
owin
ing
g coul
could
d be the
the othe
otherr thre
threee memb
member
erss
of the team?
team?

I. Simon
imon,, Mike
Mike,, and
and Char
Charli
liee
II. Simo
imon, Mike
ike, and
and Fred
red
III.
III. Simo
imon, Mike
ike, and
and Harry
rry

(A)
(A) I and II only
(B)
(B) I and III
III only
(C)
(C) II an
and III
III on
only
(D) I, II, and III

  ______________________________________________________________________________ 
For
For items
items 30 to 33, refer
refer to the follow
following
ing condit
condition
ions.
s.

I. Ther
Theree are
are four
four newl
newlyy born
born babi
babies
es in the
the nurs
nurser
ery,
y, thre
threee girl
girlss and
and one
one boy,
boy, all
all chi
child
ldre
ren
n of                 
differ
differen
entt diplom
diplomati
aticc perso
personn
nnel
el work
workinging in the Philip
Philippin
pinee embass
embassies
ies of
of Brazil
Brazil,, Italy,
Italy,
Sing
Singap
apor
ore,
e, and
and Viet
Vietna
nam.
m. The
The crib
cribss are
are arra
arrang
nged
ed alph
alphababet
eticical
ally
ly in a row
row acco
accord
rdin
ing
g to
nation
nationali
ality
ty for easy
easy identi
identific
ficati
ation
on by visiti
visiting
ng relati
relations
ons and
and frien
friends
ds who
who may view
view the
  babies
babies from behind
behind the glass
glass panel
panel of the
the nursery
nursery room.
room.
1 1
II.
II. The
The babi
babies
es have
have vari
variou
ouss wei
weigh
ghts
ts of
of 6 lb,
lb, 6 lb, 7 lb, and 7 lb, not
not neces
necessar
sarily
ily in that
that
2 2
orde
order.
r. The
The thir
third
d baby
baby is a boy
boy and
and is the
the heav
heavie
iest
st,, whil
whilee only
only the
the Sing
Singap
apor
orea
ean
n baby
baby is
exceed
exceeded
ed in weigh
weightt by the Vietna
Vietnames
mesee baby
baby..

30. “There’s
“There’s your
your sister!”
sister!” said the Vietna
Vietnames
mesee who
who had lifted
lifted his son
son and was
was point
pointing
ing from
from
  behind
behind the glass
glass pane
panell at a baby
baby in the ____
______
____
____
_____
___ crib.
crib.

(A) first
(B) second
(C) third
(D) fourth

31.
31. Base
Basedd on the
the give
given
n info
inform
rmat
atio
ion,
n, it can
can be decl
declar
ared
ed that
that the
the baby
baby boy
boy is
a(an) _____________.

(A) Ital
talian
(B)
(B) Brazil
azilia
ian
n
(C)
(C) Vietn
ietnam
amesesee
(D)
(D) Singa
ingapo
pore
rean
an

32. Of the
the four
four newly
newly born
born babie
babies,
s, the lighte
lightest
st in
in weigh
weightt is the ____
______
____
_____
_____
__..

(A)
(A) Vietn
ietnam
ames
esee
(B) Singaporean
(C) Ital
talian
(D)
(D) Brazil
azilia
ian
n

PRACTICE BOOKLET
BKL PhiLSAT_AA_0317 GO ON TO
TO THE NEX
NEXT
NEXT
XT PAGE
PA
PAGE
AG
GE
E
22

33. Which
Which of the follow
following
ing statem
statement
entss is
is true
true??

(A) The
The Brazil
Brazilian
ian and Singa
Singapor
porean
ean babies
babies togeth
together
er weig
weigh
h heavie
heavierr than
than the Italia
Italian
n and
and
Vietnamese babies.
(B) The
The Singap
Singapore
orean
an and
and Vietna
Vietnamemese
se babies
babies tog
togeth
ether
er weig
weigh
h heavi
heavier
er than
than the
Brazilian
Brazilian and Italian
Italian babies.
babies.
(C) The
The Singap
Singapore
orean
an and
and Italia
Italian
n babie
babiess togeth
together
er weigh
weigh the same
same as the Brazil
Brazilian
ian and
and
Vietnamese babies.
(D) The
The Vietna
Vietnames
mesee and Italia
Italian
n babies
babies togeth
together
er weigh
weigh the same
same as the Singap
Singapore
orean
an
and Brazilian
Brazilian babies.
babies.
  ______________________________________________________________________________ 

For
For items
items 34 to 36, refer
refer to the follow
following
ing condit
condition
ions.
s.

Employee
Employeess in a bus compan
company:
y:

I. All
All mech
mechananic
icss are
are also
also driv
driver
ers.
s.
II. Half
Half of the driver
driverss are mecha
mechanicnics.
s.
III.
III. Some
Some driver
driverss and ticket
ticket inspec
inspector
torss started
started out
out as condu
conducto
ctors.
rs.
IV. Drivers
Drivers cannot
cannot be ticket
ticket inspec
inspectors,
tors, but mechanic
mechanicss can.
can.
V. Condu
Conducto
ctors
rs cannot
cannot become
become mechan
mechanics ics..
VI. Some
Some condu
conducto
ctors
rs and
and ticket
ticket inspe
inspecto
ctors
rs do not
not kno
knoww how
how to drive.
drive.

34.
34. Whic
Which
h posi
positi
tion
on can
can be acce
access
ssed
ed by anyo
anyone
ne in the
the comp
compan
any?
y?

(A) Driver 
(B)
(B) Mech
echanic
nic

(C)
(D) Conductor 
(D) Tick
Ticket
et ins
inspe
pect
ctor 
or 

35.
35. Whic
Which
h posi
positio
tion
n is the
the most
most flex
flexib
ible
le??

(A) Driver 
(B)
(B) Mech
echanic
nic
(C)
(C) Conductor 
tor 
(D)
(D) Tick
Ticket
et ins
inspe
pect
ctor 
or 

36.
36. Whic
Which
h posi
positi
tion
on CANN
CANNOT
OT be acqu
acquir
ired
ed at entr
entry
y leve
level?
l?

(A) Driver 
(B)
(B) Mech
echanic
nic
(C)
(C) Conductor 
tor 
(D)
(D) Tick
Ticket
et ins
inspe
pect
ctor 
or 

PRACTICE BOOKLET
BKL PhiLSAT_AA_0317 GO ON TO THE NEXT PAGE
23

For
For items
items 37 to 39, refer
refer to the follow
following
ing condit
condition
ions.
s.

I. Mr. Yu’s newly constructed three-story commercial-residential


commercial-residential building along Amorsolo
St. in Makat
Makatii cons
consist
istss of
of nine
nine equal
equal unit
units.
s. He has allott
allotted
ed the three
three units
units on the groun
groundd
floor for his hardwareand
hardwareand electrical
electrical supply
supply business
business,, while
while he and his wifeplan to occupy
occupy
the middlemo
middlemost st unit on
on the second
second floor asas their residence
residence..
II.
II. The
The coup
couplele gift
gifted
ed thei
theirr new
newly
ly marr
marrie
ied
d son
son with
ith the
the righ
rightm
tmos
ostt uni
unitt on
on the
the thir
thirdd floo
floor.
r.
They
They also
also exec
execut
uted
ed a cont
contra
ract
ct with
with a grou
group p of dent
dentis
ists
ts for
for a two-
two-yeyear
ar leas
leasee on two
two
adjace
adjacent
nt units
units on one
one floor
floor to be conv
convert
erted
ed into
into a dental
dental clinic
clinic..
III.
III. The
They prefe
eferre
rred to ren
rent the
the rem
remain
aining
ing two
two units
its to fam
familie
iliess with
ith no mor
more than
than two
two
children
children each.
each.

To answe
answerr questi
question
onss 37 and 38,
38, refer
refer to the follo
followin
wing
g units
units in the building
building::

w The
The unit
= unit on the secon
secondd floor
floor dire
directl
ctly
y to the left
left of the
the coupl
couple’s
e’s unit
unit
x The leftmo
= leftmost
st unit
unit on the third
third floor 
floor 
y The unit
= unit direc
directly
tly to the left
left of the
the son’s
son’s unit
unit
z The unit directly
= directly below
below the son’s
son’s unit

37.
37. Whic
Which
h two
two unit
unitss CANN
CANNOT
OT be rent
rented
ed to fami
famili
lies
es??

(A) w an
a nd x
(B) w an
a nd y
(C) x an
a nd z
(D) x an
a nd y

38. Which
Which unit
unit is
is diag
diagon
onall
ally
y adjace
adjacent
nt to the couple’s
couple’s unit?

(A) w
(B) x
(C) y
(D) z

39. If the couple


couple decid
decides
es NOT
NOT to occup
occupy
y the middle
middlemos
mostt unit
unit but
but have
have it rented
rented also,
also, from
from
how many possible
possible pair(s)
pair(s) of adjacent
adjacent units
units will
will the
the dentists’
dentists’ group
group be able to choose?
choose?

(A) Only one


(B) Two
(C) Three
(D)
(D) At mos
mostt fou
four r 

PRACTICE BOOKLET
BKL PhiLSAT_AA_0317 GO ON TO
TO THE NEX
NEXT
NEXT
XT PAGE
PA
PAGE
AG
GE
E
24

For
For items
items 40 to 43, refer
refer to the follow
following
ing condit
condition
ions.
s.

Four
Four pre
preme
medd grad
gradua
uate
tes,
s, Stev
Steve,
e, Anne
Anne,, Vict
Victor
or,, and
and Elai
Elaine
ne,, each
each obta
obtain
ined
ed diff
differ
eren
entt passi
passing
ng
sco
scores
res, 83, 86, 90,
90, and
and 95,
95, not neces
ecessa
sari
rily
ly in that
that ord
order,
er, in the
the natio
tional admiss
missio
ion
n tes
test
for medical schools.

I. Steve
Steve and Victor
Victor obtain
obtained
ed lower
lower scores
scores than
than Anne
Anne..
II.
II. Vict
Victor
or did
did not
not get
get 90.
90.
III.
III. Stev
teve obtain
tained90
ed90..
IV.
IV. Elai
Elaine
ne did
did not
not get
get 95.
95.

40. If Elaine
Elaine obtai
obtained
ned the lowest
lowest score
score among
among the four,
four, then
then Victor
Victor must
must have obtain
obtained
ed
(A) 95
(B) 90
(C) 86
(D) 83

41. Which
Which of the
the follow
following
ing is an accur
accurate
ate list
list of the
the scores
scores tha
thatt Victor
Victor cou
could
ld have
have obtai
obtained
ned??

(A) 95, 86
(B) 90, 86
(C) 90, 83
(D) 86, 83

42. If Victor
Victor obtai
obtained
ned the score
score of 83,
83, then
then Elaine
Elaine obtai
obtaine
ned
d _____
_______
____
__..

(A)
(B) 95
90
(C) 86
(D) 83

43. If the
the scores
scores that
that Vict
Victor
or and Anne
Anne obtaine
obtained
d were
were interc
intercha
hang
nged,
ed, which
which of the srcina
srcinall
statement
statementss would
would NO longer
longer be true?

(A)
(A) State
tateme
ment
nt I
(B)
(B) State
tateme
ment
nt II
(C)
(C) State
tateme
ment
nt III
III
(D)
(D) State
tateme
ment
nt IV

PRACTICE BOOKLET
BKL PhiLSAT_AA_0317 GO ON TO THE NEXT PAGE
25

For
For items
items 44 to 47, refer
refer to the follow
following
ing condit
condition
ions.
s.

Joe
Joe Reye
Reyess and Rach
Rachelel Cru
Cruzz will
ill be wed in a week’
eek’ss time.
time. Melis
elissa
sa Cruz
Cruz,, the
the brid
bride’
e’ss
mother
mother and
and the eldest
eldest amon
among g the Cruz
Cruz siblin
siblings,
gs, is plan
plannin
ning
g to invite
invite her
her entire
entire family
family as well
well
as that
that of
of the
the groom
groom in this exclusive
exclusive occasion
occasion..

Meli
Meliss
ssa’
a’ss par
paren
ents
ts,, Fely
ely and
and Arth
Arthurur,, are
are stil
stilll ali
alive
ve and
and can
can make
make it to the the weddi
edding
ng..
Aside
Aside from
from Melis
Melissa,
sa, this
this couple
couple has three
three other
other childr
children:
en: Micha
Michael,
el, Miche
Michelle
lle,, and
and Mann
Manny.y.
The
The youn
younge
gest
st,, Mann
Manny,
y, went
went on to beco
becomeme a Cath
Cathol olic
ic prie
priest
st and
and will
will offi
offici
ciat
atee the
the wedd
weddining
g
cele
celebr
brat
atio
ion.
n. Mich
Michae
aell and Mich
Michel
elle
le are
are both
both marr
marrie ied,
d, with
with Mich
Michaeaell havin
havingg thre
threee child
childre
ren,
n,
and Michellelle haviinng
g two
two. Mich
Michae
ael’
l’ss elde
eldestst is alre
alread
adyy marr
marrie
ied
d withith one
one chil
child,
d, whil
whilee

the other
othe
accide r two
accident.
nt. Micare
Michel
hellestill
le is singl
sstill
ingle.
sti e. Michelle’
Mich
ll living
livi elle’s
ng with
wit s youngest
young
h her est
husb
husban ofMelis
and.
d. twolissa
Me children,
child ren,
sa only
onl unfor
unsfortun
y has
ha tunate
one ately,
ly,; her
child
child; died
diedhusb
insband
hu a and
car 
died of heart
heart attack
attack just a few weeks
weeks after she has given
given birth.
birth.

On the other
other hand
hand,, Joe is alread
already
y orpha
orphaned
ned by his moth
mother.
er. His fath
father
er is still
still alive
alive and
and no
longer
longer remar
remarried
ried.. Joe has two siblings,
siblings, Jennifer
Jennifer and Jack.
Jack. Jennifer
Jennifer is happ
happily
ily married
married with two
chil
childr
dren
en who
who are
are stil
stilll in prim
primar
aryy scho
school
ol,, whil
whilee Jack
Jack is curre
current
ntly
ly deta
detain
ined
ed in the
the city
city jail
jail for a
heinous crime.

44. How
How many
many will
will be attend
attending
ing the Reyes-C
Reyes-Cruz
ruz nuptia
nuptiall includ
including
ing the primar
primary
y partic
participa
ipants
nts
(bride
(bride,, groom
groom,, and
and priest
priest)?
)?

(A) 18
(B) 19
(C) 20
(D) 21

45.
45. How
How are
are Jenn
Jennif
ifer
er and
and Melis
Melissa
sa rela
relate
ted?
d?

(A)
(A) Cousins
ins
(B)
(B) Sibli
iblin
ngs
(C)
(C) Sist
Sister
ers-
s-in
in-l
-law
aw
(D)
(D) Aunt
unt and
and niec
niecee

46.
46. How
How is Fel
Fely
y rela
relate
ted
d to Rach
Rachel
el??

(A)
(A) She is her
her aun
aunt.
t.
(B)
(B) She is her
her cou
cousi
sin.
n.
(C)
(C) She is her
her mot
mothe
her.r.
(D)
(D) She
She is her
her gra
grand
ndmomothther
er..

47. Which
Which of follow
following
ing statem
statement
entss is INCORR
INCORRECT
ECT??

(A) All
All of the Cruz
Cruz siblin
siblingsgs have
have childr
children.
en.
(B) The
The occas
occasion
ion is a religi
religiou
ouss celebr
celebrati
ation
on..
(C)
(C) Rach
Rachel
el is
is not
not the
the firs
firstt to be marr
marrieied
d amon
amongg the
the Cruz
Cruzes
es..
(D)
(D) Ther
Theree are
are some
some fami
familyly memb
member erss who
who coul
could
d not
not make
make it to
to the
the wedd
weddin
ing.
g.

PRACTICE BOOKLET
BKL PhiLSAT_AA_0317 GO ON TO
TO THE NEX
NEXT
NEXT
XT PAGE
PA
PAGE
AG
GE
E
26

For
For items
items 48 to 50, refer
refer to the follow
following
ing condit
condition
ions.
s.

A race of aliens
aliens from
from Planet
Planet X has visited
visited Earth.
Earth. Earth’s linguists
linguists attempted
attempted to decode
decode
their language:

I. Kaka
Kaka me ga shin
shing 
g     means
   “what is your name?”
II.   Ham
Hamee ne gu shin g  means
shing  mea
          ns      “where
“where do youyou live?
live?””
III.
III. They
They intr
introd
oduc
ucee them
themse selv lves es by sayi
saying
ng  Me ga shan
shang
g Krak
Krakov ov .                 
IV.
IV. When
When poin
pointi
ting
ng at thei
theirr home home plan
planet
et,, they
they say
say  Hame   na ra kong 
kong .       
V. Memu
Memu me ga shin g    means
shing          “who
“who is your father?”
father?”

48. What is the


the most likely meaning of the word  Hame ?             
likely meaning
(A) You
(B) Live
(C) Name
(D) Father 

49.
49. How
How do the
the inha
inhabi
bita
tant
ntss of Plan
Planet
et X say
say “I am
am Sam”
Sam”??

(A) Kaka
Kaka me Sam.
Sam.
(B) Sam
Sam me ga shin
shing.
g.
(C)   Me ga shan
shang Sam .           
g Sam 
(D) Memu
Memu me ga shin
shing
g Sam. Sam.

50.
50. In the
the gram
gramma
marr of
of Pla
Plane
nett X,
X, how
how does
does the
the word
word shing   function?

(A)
(B)
(B) It states
It state
poi
pointssatoname.
nts name . tion
a loca
location..
(C)
(C) It ind
indic
icat
ates
es a ques
questi
tion
on..
(D) It conne
connects
cts words
words into
into a sente
sentence
nce..

PRACTICE BOOKLET
BKL PhiLSAT_AA_0317 GO ON TO THE NEXT PAGE
27

TEST
TEST
TEST C. VERBAL
VERBA
VERBAL
L REASON
REASONING
ING

Direction
Directions:
s: This section
section contains
contains reading
reading selectio
selections
ns that
that are followed
followed by a set of
of items.
items. Answer 
Answer 
the items
items accor
accordin
dingg to what
what is stated
stated or implie
implied
d in the select
selection
ion..

Selection
Selection 1

(1) Kno
Knowin
wingg that
that Mrs.
Mrs. Malla
Mallard
rd was
was afflict
afflicted
ed with
with a heart
heart troub
trouble,
le, great
great care
care was
was taken
taken to
  break
break to her as
as gently
gently as possible
possible the news
news of her
her husband’
husband’ss death.
death. It was her
her sister,
sister, Josephin
Josephine,
e,
who
who told
told her,
her, in broken
broken senten
sentences
ces;; veil
veiled
ed hints
hints that
that reveal
revealed
ed in half
half con
concea
cealin
ling.
g. Her husband’s
friend
friend Richar
Richards
ds was
was there,
there, too,
too, near
near her.
her. It was
was he who
who had been
been in the news
newspap
paper
er office
office when
when
intelligen
intelligence
ce of the railroad
railroad disaster
disaster was received
received,, with Brently
Brently Mallard’s
Mallard’s name leading
leading the list of                 
“killed.”

(2)
(2) The
There
re stoo
stood,
d, faci
facing
ng the
the open
open wind
window
ow,, a comf
comfor
orta
tabl
ble,
e, room
roomy
y armc
armcha
hair
ir.. Into
Into this
this she
she
sank,
sank, presse
pressed
d down
down by a physic
physical
al exhau
exhausti
stion
on that
that haunt
haunted
ed her
her body
body and seeme
seemed
d to reach
reach into
into her 
her 
soul.

(3) She could


could see in the open
open squar
squaree befor
beforee her
her house
house the tops
tops of trees
trees that
that were
were all aquiv
aquiver 
er 
with
with the
the new
new spri
spring
ng life.
life. The
The deli
delici
ciou
ouss brea
breath
th of rain
rain was
was in the
the air.
air. In the
the stre
street
et below
below,, a pedd
peddle
ler r 
was crying
crying his wares
wares.. The notes
notes of a distant
distant song
song which
which someone
someone was singing
singing reached
reached her faintly,
faintly,
and countless
countless sparrows
sparrows were twittering
twittering in the eaves.
eaves.

(4)
(4) She
She sat
sat wit
withh her
her hea
head
d thro
thrown
wn back
back upon
upon the
the cush
cushio
ion
n of the
the chai
chair,
r, quit
quitee motio
motionl
nles
ess,
s,
exce
except
pt whe
when n a sob
sob came
came up into
into her
her thro
throat
at and
and shoo
shook
k her,
her, as a child
child who
who has
has cried
cried itse
itself
lf to slee
sleep
p
continue
continuess to sob in its dreams.
dreams.

(5) She
She did not
not stop
stop to ask if it
it were
were or were
were not
not a monst
monstrou
rouss joy
joy that
that held
held her.
her. A clear
clear and
and
exalted
exalted perceptio
perception
n enabled
enabled her to dismiss
dismiss the suggestio
suggestionn as trivial.
trivial.

(6)
(6) She
She knew
knew that
that she
she woul
wouldd weep
weep agai
again
n when
when she
she saw
saw the
the kind
kind,, tend
tender
er hand
handss fol
folde
dedd in
deat
death;
h; the
the face
face that
that had
had neve
neverr look
looked
ed save
save with
with love
love upon
upon her,
her, fixe
fixed
d and
and gray
gray and
and dead
dead.. A kind
kind
inten
intentio
tion
n or a cruel
cruel intent
intention
ion made
made the act seem
seem no less
less a crime
crime as she
she looked
looked upon
upon it in that
that brie
brief f                 
moment of illumination.

(7) What
What cou
could
ld love,
love, the unsol
unsolve
ved
d myster
mystery,y, coun
countt for
for in face
face of this
this posse
possessi
ssion
on of self
self--
assertion
assertion which
which she suddenly
suddenly recogniz
recognized
ed as the
the stronges
strongestt impulse
impulse of her
her being!
being!

(8) “Free!
“Free! Body
Body and soul free!”
free!”   she kept whispering.
whispering.

(9) Josep
Josephin
hinee was
was kneelin
kneeling
g befor
beforee the closed
closed door
door with her
her lips to the keyho
keyhole,
le, implor
imploring
ing
for admission
admission.. “Louise,
“Louise, open the door!
door! For
For heaven’s
heaven’s sake,
sake, open the door.”
door.”

(10) No, Louise


Louise was drinking
drinking in the very elixir
elixir of life through
through that open window.
window. Her fancy
fancy
was runnin
running g riot
riot alon
along
g those
those days
days ahea
aheadd of her.
her. Sprin
Springg days,
days, and
and summe
summerr days
days,, and
and all sorts
sorts of 
of                 
days
days tha
thatt woul
wouldd be her
her own.
own. She
She brea
breath
thed
ed a quic
quick k pray
prayerer tha
thatt life
life migh
mightt be long
long.. It was
was onl
only
y
yester
yesterda
dayy she
she had
had thoug
thought
ht with
with a shud
shudder
der that
that life might
might be long.
long.

PRACTICE BOOKLET
BKL PhiLSAT_AA_0317 GO ON
ON TO THE NEXT
NEXT PAGE
PAGE
28

(11)
(11) She
She aros
arosee at len
lengt
gth
h and
and open
opened
ed the
the door
door toto her
he r sister’s
sister’s importun
importunities.
ities. There
There was a
feveri
feverish
sh triump
triumph h in her eyes,
eyes, and
and she
she carrie
carried
d hersel
herselff unwit
unwittin
tingly
gly like
like a godd
goddess
ess of Victor
Victory.
y. She
She
clasped
clasped her sister’s
sister’s waist,
waist, and together
together they descend
descended
ed the stairs.
stairs.

(12)
(12) Someo
Someone ne was
was open
opening
ing the front
front door
door with
with a latchke
latchkey.y. It was
was Bren
Brently
tly Malla
Mallard
rd who
who
entere
entered,
d, a little
little trave
travel-s
l-stai
taine
ned,
d, compo
composed
sedly
ly carryi
carrying ng his grips
gripsack
ack and
and umbre
umbrella
lla.. When
When the doctor
doctorss
came,
came, they
they said
said she had died
died of heart
heart disea
disease
se  – of
o f the joy that
that kills.
kills.

  An excerpt
excerpt from “The Story
Story of an Hour” by Chopin, 1894.
by Kate Chopin,

1. The
The statem
statement
ent “Louis
“Louisee was
was drinking
drinking in the very
very elixir
elixir of life” is ironic
ironic becaus
becausee she found
found life
sweet knowing she

(A)
(A) gain
gained
ed cari
caring
ng,, true
true frie
friend
ndss
(B)
(B) would
ould be free
free from
from hear
hearta
tach
ches
es
(C)
(C) was
was supp
suppos
osed
ed to be in emot
emotio
iona
nall pain
pain
(D)
(D) would
would be reliev
relieveded of her
her heart
heart proble
problemm

2. The doctors’
doctors’ statement
statementss contrad
contradicted
icted the real cause
cause of Mrs.
Mrs. Mallard’
Mallard’ss death
death which
which was

(A) the appearan


appearancece of Brently
Brently who was surpr
surprised
ised at Richards’
Richards’ sudde
suddenn movement
movement
and unexpec
unexpected
ted reaction
reaction
(B)
(B) the
the disc
discov
overery
y that
that Bren
Brentl
tly
y was
was alive,
alive, a twis
twistt that
that advers
adversel
ely
y affe
affect
cted
ed her 
her 
heart condition
(C) the sad news
news about
about Brently’s
Brently’s tragic
tragic death
death which
which she found
found  unbearable
unbearable and
difficult
difficult to accept
accept
(D) Joseph
Josephine
ine’s
’s shriek
shrieking
ing cry which
which jolted
jolted Mrs.
Mrs. Malla
Mallard,
rd, causin
causing g her
her to scream
scream and
eventually
eventually collapse
collapse

3. What
What figur
figuree of speech
speech is applied
applied in the 
the  phrase “monstrous joy?”

(A) Irony
(B) Allusion
(C) Metonymy
(D) Oxymoron

4. From
From what
what point
point of view
view is the selection
selection written?
written?

(A) First
(B) Second
(C) Limited
(D) Omnisciscient

PRACTICE BOOKLET
BKL PhiLSAT_AA_0317 GO ON
ON TO THE NEXT
NEXT PAGE
PAGE
29

5. Which
Which of the follow
following
ing best
best states
states the theme
theme of the story?
story?

(A)
(A) All
All marr
marria
iage
ges,s, even
even the
the kind
kindes
estt ones
ones,, are
are inher
inheren
ently
tly oppr
oppres
essi
sive
ve..
(B) Women’s
Women’s true
true calling
calling is in their kitchens
kitchens and laundry
laundry rooms.
rooms.
(C)
(C) Wome
Women n are
are expe
expect
cted
ed to bear
bear and
and rear
rear child
childre
ren
n as dict
dictat
ated
ed by the
the soci
societ ety.
y.
(D)
(D) Inde
Indepe
pend
nden
encece can
can be imag
imaginined
ed only
only priv
privat
atel
ely
y by wome
women n beca
becaus
usee of         
dominatio
dominationn of men.

6. What
What devic
devicee did the autho
authorr use to end the story?
story?

(A)
(A) Open
Open-e
-end
nded
edne
ness
ss

(B)
(C) TComic
Comi
ragicc irelief 
relie
ronyf    
(D) Symbolism
  ______________________________________________________________________________ 

Selection
Selection 2

(1) The
The quality
quality of our
our thoughts
thoughts becom
becomes
es visible
visible through
through our words
words,, our behav
behavior,
ior, or even
even
thro
throug
ughh the
the expr
expresessi
sion
on of our
our fac
face.
e. The
The seed
seed of word
wordss and
and acti
action
onss is
is tho
thoug
ught
ht.. By know
knowining
g
and
and unde
unders
rsta
tand
ndin
ing g the
the type
typess of tho
thoug
ught
htss our
our mind
mind can
can crea
create
te,, we will
will bebe able
able to cons
consci
ciou
ousl
sly
y
redire
redirect
ct tho
though
ughts
ts tow
toward
ardss the
the posit
positive
ive.. Hence
Hence we will
will impro
improve
ve the quality
quality of our
our mind
mind,, body
body,,
and relations
relationships
hips..

(2) Necessary thoughts are those


those that
that deal
deal with our
our daily
daily routin
routinee like,
like, “What
“What will
will I eat?”;
eat?”;
“What
“What do I need
need to do today?
today?”;
”; “When
“When do I hav
havee to pay my bills?
bills?”” These
These are practi
practical
cal thoug
thought
htss to
attend
attend to our
our daily
daily needs.
needs.

(3) Wasteful
Wasteful (or superfluou
superfluous)
s) thoughts
thoughts are those
those that are of no use, neither
neither constructi
constructive
ve nor 
  particul
particularl
arly
y negat
negative
ive.. They
They deal
deal with
with things
things of
of the past,
past, like
like “If this
this had not
not happene
happened;”
d;” “If only I
have done
done it this
this way.”
way.”

(4) Negativ
Negativee thoughts
thoughts are
are most harmfu
harmful,l, especially
especially to ourselve
ourselves. s. Negative
Negative thoughts
thoughts which
which
are based
based on materi
materiali
alisti
sticc desire
desires,
s, anger,
anger, jealou
jealousysy,, hatre
hatred,
d, extre
extreme
me lazine
laziness,
ss, and
and posses
possessiv
sivene
eness
ss
cause
cause us great
great loss
loss of inner
inner peac
peacee and
and stren
strength
gth.. Such
Such thoug
thoughts
hts pois
poisonon our mind
mind as well
well as the
the
atmo
atmosp
sphe
here
re.. No matt
matter
er how
how righ
rightt we are,
are, by thin
thinki
king
ng nega
negati
tive
vely
ly,, we lose
lose beca
becaus
usee nega
negati
tive
ve
thoughts
thoughts take away
away our self-re
self-respec
spect,
t, and also,
also, others lose
lose respect
respect for
for us.
us.

(5) Positive
Positive thoughts
thoughts allow us to accumula
accumulate te inner
inner stren
strength
gth and enable
enable us to be creative
creative and
construc
constructive.
tive. To think
think positivel
positively
y doesn’t
doesn’t meanthat we ignore
ignore the reality.
reality. To think positive
positively
ly means
means
to use the proble
problemm and
and to find
find solut
solution
ionss for
for that
that probl
problem.
em.

(6) A person
person who thinks
thinks positiv
positively
ely will be aware of thethe weaknes
weaknesses
ses of
of others,
others, but will still
turn
turn his attent
attention
ion toward
towardss the
the good
good tenden
tendencie
ciess of others
others.. Posit
Positive
ive thoug
thoughts
hts give
give us a feelin
feeling
g of                 
conten
contentme
tment
nt within
within.. When
When we are conten
contentt within
within,, then
then we have
have the stren
strength
gth to accept
accept others
others as
they
they are witho
without
ut want
wanting
ing to chang
changee them
them as we think
think is righ
right.
t.

PRACTICE BOOKLET
BKL PhiLSAT_AA_0317 GO ON
ON TO THE NEXT
NEXT PAGE
PAGE
30

7. The author
author devel
developed
oped his topic by using
using all of
of the
the following
following methods
methods EXCEPT
EXCEPT

(A) narra
rratio
tion
(B) defin
finitio
ition
n
(C)
(C) enum
enumererat
atio
ion
n
(D)
(D) exem
exemplplif
ific
icat
atio
ion
n

8. In which
which parag
paragrap
raph
h is the main
main idea
idea of the essay
essay found
found??

(A) Para
aragrap
raph 1
(B) Paragraph 2

(C) Paragrap
Para
(D) Paragraph
aragraph6
raph 3

9. In parag
paragrap
raph
h 4, the unde
underlin
rlined
ed statemen
statementt “such thoughts
thoughts poison
poison our mind
mind as well as
the atmosph
atmosphere”
ere” implies
implies that negative
negative thoughts
thoughts

(A) leadto
leadto desp
espair 
air 
(B) tarnish one’s character
(C)
(C) affe
affect
ct the
the surr
surrou
ound
ndin
ings
gs
(D)
(D) have
have soci
social
al impl
implic
icat
atio
ions
ns
  ______________________________________________________________________________ 

Selection
Selection 3

Some
Some prima
primary
ry care
care provid
providers
ers may be appre
apprehe
hensi
nsive
ve about
about using
using insuli
insulin
n in patien
patients
ts with
with
type
type 2 diabet
diabetes.
es.

Concerns about hypoglycemia


hypoglycemia (diminished contents of glucose in the blood) and/or ability
to inject
inject insulin
insulin are good
good reasons
reasons why many providers
providers may approach
approach insulin
insulin therapy
therapy with caution.
caution.
Compo
Compoun undin
ding
g this
this relucta
reluctance
nce is the perce
percepti
ption
on that
that insulin
insulin therap
therapy
y is too comple
complexx to manage
manage in a
busyprimary care practic
practice;
e; prescr
prescribinginforma
ibinginformation
tion provided
provided bymanufacturer
bymanufacturerss has been somewha
somewhatt
vague
vague regardin
regardingg initial
initial dosing
dosing and titration.
titration.

Becaus
Becausee of the kind
kind of inform
informati
ation
on provi
provide
ded
d by the manufa
manufactu
cturer
rers,
s, provi
provide
ders
rs may delay
delay in
making
making the necessary
necessary transition
transition from oral agents
agents to insulin.
insulin. Indeed,
Indeed, recent
recent evidence
evidence suggests
suggests that
the
the hemo
hemogl
glob
obin
in A1c result
result that triggers
triggers glucose-lo
glucose-loweri
wering
ng action
action is ≥ 9% (Bro(Brown
wn and
and Nich
Nichol
ols,
s,
2003).
2003). This is unfortu
unfortunate
nate because
because numerou
numerouss studies
studies have shown
shown that excelle
excellent
nt glycemic
glycemic control
control
can be achiev
achieved
ed with
with insuli
insulin
n therap
therapyy in patien
patients
ts with
with type
type 2 diabet
diabetes.
es.

Subjects
Subjects of a cohort
cohort study (Epidemi
(Epidemiolog
ologyy of Diabetes
Diabetes Interve
Interventio
ntions
ns and Complica
Complication
tionss
Study),
Study), who had been intensive
intensively
ly treated
treated during
during the Diabetes
Diabetes Control
Control and Complica
Complications
tions Trial, at
seven
seven years
years show
showeded significan
significantt decrea
decreases
ses in risk for nephropa
nephropathy
thy and retinopat
retinopathy
hy compared
compared with
subjects from the conventional
conventional treatment
treatment arm.
arm.

Malmb
Malmberg
erg,, Norh
Norhamm
ammar,
ar, Wedel,
Wedel, and Ryden
Ryden (1999
(1999)) demo
demonst
nstrat
rated
ed that
that the unfav
unfavora
orable
ble
long-t
long-term
erm progn
prognosi
osiss for
for myocar
myocardia
diall infar
infarcti
ction
on (heart
(heart attack
attack)) cou
could
ld be improv
improveded by insuli
insulin
n
treatment.

PRACTICE BOOKLET
BKL PhiLSAT_AA_0317 GO ON
ON TO THE NEXT
NEXT PAGE
PAGE
31

Recent
Recent data (Ceriello
(Ceriello et al., 2004)
2004) show
show that postpra
postprandia
ndiall hyperglyc
hyperglycemia
emia is accompa
accompanied
nied
by endothe
endothelial
lial dysfunc
dysfunction
tion in patients
patients with type 2 diabetes.
diabetes.

This significan
significantt body
body of evidence
evidence strongly
strongly supports
supports the rationale
rationale for initiating
initiating therapy
therapy to
achi
achiev
evee glyc
glycem
emic
ic cont
controroll in pati
patien
ents
ts with
ith type
type 2 diab
diabet
etes
es much
much earl
earlie
ierr and
and much
much more
more
aggressively.

  An excerpt from an online


online journal article “A Real  -World
journal article                             Approach
      to Insulin             
Therapy
Therapy in Primary
Primary Care Practi
Practice”
ce” by Hirsch, Hirsch, I.B. et al., 2005.

10. The first paragrap


paragraph
h says that primary
primary care providers
providers view the use of insulin
insulin in patients
patients with
type 2 diabetes
diabetes with

(A) anger  
(B) anxiety
(C) approv
roval
(D)
(D) awar
awaren
enesesss

11. Primary
Primary care provider
providerss are reluctant
reluctant to administe
administerr insulin
insulin therapy
therapy because
because of                 
the manufacturers’
manufacturers’

(A)
(A) inabil
inability
ity to provid
providee instru
instructi
ction
onss on
on initia
initiall dos
dosing
ing and
and titrat
titration
ion
(B)
(B) incomp
incomplet
letee proced
procedure
ure regard
regarding
ing initia
initiall dosin
dosingg and
and titrat
titration
ion
(C)
(C) lack
lack of inform
informatiation
on regard
regarding
ing initia
initiall dosin
dosing g and
and titrati
titration
on
(D)
(D) fuzzy
fuzzy expla
explananatio
tionn regard
regarding
ing initia
initiall dos
dosing
ing and
and titrati
titration
on

12. Which
Which of the following
following statemen
statements
ts summarizes
summarizes the medical
medical article?
article?

(A) Insulin
Insulin therapy
therapy is very
very beneficia
beneficial.
l.
(B)
(B) Peop
People
le must
must be be consci
consciou
ouss of their
their diet
diet..
(C)
(C) Endo
Endothe
thelia
liall damag
damagee is actu
actuall
ally
y preven
preventedted by insuli
insulin
n therap
therapy.
y.
(D)
(D) Insuli
Insulin
n is consid
considere
ered
d as the most
most effecti
effectiveve treatm
treatmen
entt for diabete
diabetes.
s.

13. Which
Which of the follow
following
ing concl
conclusi
usion
onss can be made
made from
from the medica
medicall report?
report?

(A)
(A) It is risky
risky for patien
patients
ts with
with type
type 2 diabet
diabetes
es to have
have insuli
insulin
n therap
therapy.
y.
(B)
(B) It is common
common amongamong type
type 2 diabet
diabetes
es patien
patients
ts to have
have insuli
insulin
n therap
therapy.
y.
(C)
(C) It is cost
costly
ly to have
have insu
insuli
lin
n ther
therap
apyy amon
among g pati
patienents
ts with
with type
type 2 diab
diabet
etes
es..
(D)
(D) It is
is bette
betterr for
for patie
patients
nts with
with type
type 2 diabe
diabetes
tes to have
have insuli
insulin
n therap
therapyy earlie
earlier.
r.

PRACTICE BOOKLET
BKL PhiLSAT_AA_0317 GO ON
ON TO THE NEXT
NEXT PAGE
PAGE
32

Selection
Selection 4

(1) Filipi
Filipino
noss offe
offerr egg
eggss to
to Santa
Santa Clara
Clara to assure
assure good
good weath
weather
er on a partic
particula
ularr day
day.. It is
intere
interesti
sting
ng to contra
contrast
st that
that with the rain
rain dance
dance of the
the Nativ
Nativee Ameri
America
cans
ns to induce
induce rain.
rain. Why
Why is this
this
so? To be able
able to answ
answer er this,
this, it is nece
necessa
ssary
ry to unde
underst
rstand
and how
how rains
rains hav
havee shape
shaped
d our
our cultu
culture.
re.

(2) The import


importanance
ce of rains
rains in Philip
Philippin
pinee cultur
culturee is atteste
attested
d by the rich
rich vocab
vocabulaulary
ry relate
related
d
to prec
precip
ipit
itat
atio
ion.
n. In Taga
Tagalolog,
g, ther
theree are
are term
termss such
such as ambon   (rain
(rain show
shower)
er),, ulan   (rain), bagyo  
(typhoon), unos     (squall), sigwa   (tempest),
(tempest), and siyam-siyam   (incessan
(incessantt rains).
rains). Other
Other Philipp
Philippine
ine
langu
language
agess have
have more
more elabo
elaborat
ratee terms.
terms. For
For examp
example,
le, the Ivatan
Ivatan of Batan
Bataneses distin
distingu
guish
ish differ
different
ent
types of
of typhoon,
typhoon, e.g. salawsaw     (wind
(windyy day
day with
with rain),
rain), nisu   (windy
(windy day without
without rain),
rain), adipogpog  

(torna
(tornado
and do
then
the with
wit
n back
ba h again
ck rain),
rain),
aga dipanchi 
in to rain),     (hea
rain), anin ( heavyvy rain
rastrong
  (very in with
wigthwind
stron stro
strong
ng wind
wicauses
that ndses
cau that
thatdestruct
sudd
sudden
dest enly
ly chan
chand
ruction),
ion), ange
gess to du
anin a sunn
sunny
y day
day            
vanveyeh 
(winds
(winds strong
strong enough
enough to destroy
destroy coconutcoconut trees).
trees). This elaborate
elaborate typhoon
typhoon terminolo
terminology
gy is not at all
surprisin
surprising
g since
since 13 out of the
the 20 typhoon
typhoonss that
that hit the Philippin
Philippines
es annua
annually
lly pass through
through Batanes.
Batanes.

(3) Sev
Severa
erall Filipi
Filipino
no cultu
cultural
ral trai
traits
ts have
have develo
developeped
d as an
an adapta
adaptatio
tion
n to the presen presencece of a
long,
long, distin
distinct
ct rainy
ainy season
season.. For
For examp
example,le, the struc
structur
turee of the bahay kubo  (native
(nati
            ve hut) has
has been
been
design
designed
ed to weath
weatherer heavy
heavy rains.
rains. The roof
roof is steep
steeply
ly struct
structure
uredd so as to allow
allow rainrain to fallfall freel
freely
y to
the ground
ground.. There
There are awnin
awning g windo
windowsws that
that prote
protect
ct the
the inner
inner part
part of the
the house
house from from gettin
gettingg wet.
wet.
The
The hous
housee is buil
builtt on stil
stilts
ts to prev
preven
entt flood
flood wate
waters
rs from
from reac
reachi
hing
ng the
the hous
housee floor
floor.. On the the othe
other r 
hand,
hand, the Ivatan
Ivatan house
house  —made
made of ston
stonee walls
walls and
and roofe
roofed d with
with thick 
thick  cogon   thatch 
thatch —is a hous housee that
that
can withstan
withstand d typhoon
typhoons. s. Unfortun
Unfortunately
ately,, modern
modern houses have have lost theserain-adapted
theserain-adapted characterist
characteristics
ics
and this is why they have
have become
become vulnerab
vulnerable le to destructio
destructionn brought
brought about
about by floods
floods and and typhoon
typhoons. s.

(4) Cult
Cultura
urally
lly,, rains
rains are
are also
also percei
perceive
ved
d as being
being laden
laden with
with certai
certain
n symbo
symbolic
lic meanin
meanings
gs.. A
little
little amoun
amountt of rain durin
during g a weddi
weddingng ceremo
ceremony
ny is consid
considere
ered
d as a blessi
blessing
ng from
from the heave
heavens.
ns. On

the contra
contrary,
ry, heavy
heavy rains
rains are frown
frowned
ed upon
upon since
since they
they bring
bring abou
aboutt flood
floodss and destr
destruc
uctio
tion.
n.
(5) Gettin
Getting g wet
wet in the rain
rain is belie
believed
ved to cause
cause certai
certain
n illnes
illnesse
ses.
s. There
There is a common
common belie
belief f                 
that
that one
one catc
catche
hess a cold
cold beca
becaus
usee he or she
she got
got dren
drenchched
ed in the rain ( naulanan )  . An excep
the rain ceptio
tion,
howeve
however,
r, is when
when one
one takes
takes a show
showerer from
from the first
first rain
rain durin
during
g the month
month of May.
May.

(6) Anot
Anothe
herr popula
popularr devoti
devotion
on among
among Filipi
Filipinonoss is the offeri
offeringng of eggs
eggs to Santa
Santa Clara
Clara to
  bring
bring about
about good
good weather.
weather. One possible
possible explanatio
xplanation n for associati
associating
ng the saint
saint with
with good
good weather
weather is
her name,
name, Clara,
Clara, which
which means
means “cle
“clear”
ar” in Spani
Spanishsh.. Becau
Because se of the
the   offering
offering,, the weather
weather becom
becomeses
clear
clear even
even durin
duringg the rainy
rainy season
season.. Moreo
Moreovever,
r, the egg
egg white,
hite, in Spani
Spanish sh is called
called “clara de huevo ”.           
The
The vene
venera
rati
tion
on to Sant
Santaa Clar
Claraa was
was intr
introd
oduc
uced
ed by thethe Span
Spanis
ish
h fria
friarsrs,, she
she bein
being
g one
one of the
the sain
saints
ts
introduc
introduced
ed early to Filipinos
Filipinos..

  Adapted
Adapted from “Ulan,
“Ulan, unos,
unos, atbp.”
atbp.” By N. T. Castro
Castro..
  Retrieved from http://mb.com.ph.

14. To which
which part
part of
of the
the house
house does
does the under
underlin
lined
ed word
word “thatc
“thatch”
h” in parag
paragrap
raph
h 3 refer?
refer?

(A) Roof                  
(B) Walls
(C) Floor  
(D) Wind
indows

PRACTICE BOOKLET
BKL PhiLSAT_AA_0317 GO ON
ON TO THE NEXT
NEXT PAGE
PAGE
33

15. What
What is the chara
characte
cteris
ristic
tic of a house
house on stilts
stilts??

(A)
(A) It is built
built on wate
water.r.
(B)
(B) It con
consi
sist
stss of two
two floo
floors
rs..
(C)
(C) It has
has a conc
concrerete
te base
baseme
ment nt..
(D)
(D) It is rais
raised
ed from
from the
the grou
groundnd..

16. What
What do the
the rain
rain danc
dancee and
and the
the devo
devoti
tion
on to Sant
Santaa Clar
Claraa shar
sharee in comm
common
on??

(A)
(A) Both
Both are
are aski
asking
ng for
for rai
rain.
n.
(B)
(B) Both
Both are pre-co
pre-colon
lonial
ial pract
practice
ices.
s.

(C)
(D) Both
Both
th impl
(D) Both
Bo are
areply
im done
do
y ne
theduri
thedu
bering
ngf athat
belie
lief dry
drtyrain
tha spel
spnell.
rai l. cont
is contro
roll
lled
ed by a high
higher
er bein
being.
g.

17. Which
Which of the following
following is conside
considered
red a welcome
welcome occurren
occurrence?
ce?

(A)
(A) Havi
Having
ng a down
downpopour
ur duri
during
ng a wedd
weddin
ing
g
(B)
(B) Gett
Gettin
ing
g wet
wet in the
the firs
firstt rain
rain of May
May
(C)
(C) Getti
etting
ng dren
drench
ched
ed in a heav
heavy y rain
rain
(D)
(D) Gett
Gettin
ing
g caug
caught
ht in a ligh
lightt dri
drizz
zzle
le

18. What
What does the use of different
different express
expressions
ions for
for rain in the Philippin
Philippinee languag
languages
es suggests?
suggests?

(A)
(A) The
The richn
richness
ess and
and sophis
sophistic
ticati
ation
on of Philip
Philippin
pinee langu
languag
ages
es
(B)
(B) The
The differ
differen
ences
ces among
among the Philip
Philippin
pinee langua
languages
ges
(C)
(C) The
The change
changess in the preva
prevailin
iling
g weath
weather er condit
condition
ionss
(D)
(D) The
The infl
influe
uenc
ncee of forei
foreign
gn lang
langua
uage
gess on ours
ours

  ______________________________________________________________________________ 
Selection
Selection 5

We all kno
knoww that
that the norma
normall human
human daily
daily cycle
cycle of activ
activity
ity is some
some seven
seven to eight
eight hours
hours of 
of                 
sleep alternating
alternating with some sixteen
sixteen to seventeen
seventeen hours
hours of wakefuln
wakefulness
ess and that, broadly
broadly speaking
speaking,,
sleep normally
normally coincide
coincidess with the hours
hours of darkness
darkness.. Our present
present concern
concern is with how easily
easily and
to what
what exten
extentt this
this cycle
cycle can be modified
modified..

The
The ease
ease,, for
for exam
exampl
ple,
e, with
with whic
whichh peop
peoplele can
can chan
changege from
from work
workining
g in the
the day
day to work
workin
ing
g
at nigh
nightt is a questi
questionon of grow
growing
ing import
importanc
ancee in indust
industryry where
where autom
automati
ation
on calls
calls insis
insisten
tently
tly for 
round-
round-the
the-cl
-cloc
ockk worki
working
ng night.
night. Unfor
Unfortun
tunate
ately,
ly, it is often the case
case in indus
industry
try that
that the eight-
eight-ho
hour 
ur 
work
work shifts
shifts are
are chang
changeded every
every week
week.. This
This mean
meanss that
that no soone
soonerr has he
he gotte
gottenn used
used to one routin
routinee
than
than he has
has to chang
changee to anoth
another,
er, so that
that much
much of his his time
time is spe
spent
nt neit
neithe
herr worki
working
ng nor
nor sleep
sleeping
ing
very efficientl
efficiently.
y.

One
One answe
answerr would
would seem
seem to be longe
longerr period
periodss on each
each shift,
shift, a month
month,, or even
even three
three month
months.
s.
Resear
Researchch by Bonje
Bonjerr (1960)
(1960) of the Nethe
Netherla
rland
nds,
s, howe
howevever,
r, has
has shown
shown that
that peop
peoplele on such
such syste
systems
ms
will revert
revert to their normal
normal habits
habits of sleep and wakefuln
wakefulnessess during
during the weekend
weekend and that this is quite
quite
enoug
enough h to destr
destroy
oy any
any adap
adaptat
tation
ion to night
night work
work built
built up durin
duringg the week
week..

PRACTICE BOOKLET
BKL PhiLSAT_AA_0317 GO ON
ON TO THE NEXT
NEXT PAGE
PAGE
34

The
The only
only real
real solu
soluti
tion
on appe
appear
arss to
to be the
the hand
handin
ing
g over
over of the
the nigh
nightt shi
shift
ft to a corp
corpss of 
of                 
permanent night workers
workers whose nocturnal wakefulness
wakefulness may persist through all weekends weekends andand
holida
holidays
ys.. An intere
interesti
sting
ng study
study of the dome
domesti
sticc life
life and healt
healthh of night
night shift
shift worke
workers rs was
was carried
carried
out
out by Brow
Brown n in 1957
1957.. She foun
foundd a high
high inci
incide
denc
ncee of dist
distur
urbe
bed
d slee
sleep,
p, dige
digest
stiv
ivee diso
disord
rder
er,, and
and
domest
domestic
ic disrup
disruptio
tion
n among
among those
those on altern
alternati
ating
ng day andand night
night shifts
shifts,, but
but no abno
abnorma
rmall occur
occurren
rence
ce
of these
these symptoms
symptoms among
among those
those on permanen
permanentt night work.
work.

This latter
latter syste
systemm then appears
appears to be the best long-term
long-term policy,
policy, but meanwhile,
meanwhile, somethin
something g
may
may be done
done to reli
reliev
evee the
the stra
strain
inss of altern
alternat
atee day
day and
and nigh
nightt work
work by sele
select
ctin
ing
g thos
thosee peop
people
le who
who
can adapt
adapt most
most quick
quickly
ly to the chang
changeses of routin
routine.
e. One
One way
way of know
knowing
ing when
when a perso
person
n has
has adapted
adapted
is by measur
measuring
ing his perfo
performa
rmancnce,
e, but
but this
this can be labori
laboriou
ous.
s. An easie
easierr way
way is to take
take a perso
person’s
n’s

  body temperatu
temp
in normal
norm erature
re at
al daytime
daytime intervals
inter
work
wor valshave
k will of
of two hours
hou
high rs through
thro
temperatu
temp ughout
reout
erature durithe
ng period
during period of
rs wakefuln
the hours
hou wak efulness.
ess.
of wakeful
wakefulness
nessPeople
Peopleaengaged
and engaged
low one
at nigh
night.
t. When
When they
they chang
changee to night
night work,
work, the patter
pattern
n will
will only
only gradu
graduall
ally
y revers
reversee to match
match the new
new
routine
routine and the speed
speed with which
which it does
does parallels
parallels the adaptatio
adaptation n of the bodyas a whole,
whole, particula
particularly
rly
in terms
terms of perfor
performan
mancece and gener
general
al alertne
alertness.
ss. Know
Knowledledge
ge of how quick
quicklyly a perso
personn can adapt
adapt to a
revers
reversed
ed routin
routinee could
could be used
used as a basis
basis for select
selection
ion.. So far,
far, howe
however
ver,, such
such a form
form of select
selection
ion
does
does not
not seem
seem to have
have been
been applie
applied
d in practi
practice.
ce.

19. Body
Body tempe
temperat
rature
ure can be used
used as a physi
physiolo
ologic
gical
al measur
measuree of perfo
performa
rmanc
ncee to indica
indicate
te

(A)
(A) effi
effici
cien
ency
cy in work 
work 
(B)
(B) modi
modifified
ed slee
sleepi
ping
ng habi
habits
ts
(C)
(C) adap
adapta
tati
tion
on to work
work sche
schedu
dule
le
(D)
(D) endu
endura
ranc
ncee for
for nigh
nightt
ttim
imee work 
work 

20. The
The autho
authorr sug
sugges
gests
ts that
that the
the soluti
solution
on to proble
problems
ms arisin
arising
g from
from nightt
nighttime
ime work
work sched
schedule
ule

shou
should
ld be adopte
adopted
d by compa
companie
niess on a
(A)
(A) long
long-t
-ter
erm
m basi
basiss
(B)
(B) shor
short-
t-te
termbas
rmbasisis
(C)
(C) trial-
trial-and
and-er
-error
ror basis
basis
(D)
(D) comb
combin ined
ed shor
shortt and
and long
long-t
-ter
erm
m basi
basiss

21. The
The cycle
cycle of sleep
sleep can be modifi
modified
ed extens
extensive
ively
ly and succe
successf
ssfull
ully
y by

(A)
(A) alte
altern
rnat
atin
ing
g shift
shiftss ever
every
y week
week and
and week
weeken
endd
(B)
(B) alte
altern
rnat
atin
ing
g day
day and
and nigh
nightt work
work sche
schedu
dule
less with
withinin the
the week 
week 
(C)
(C) assi
assign
gnin
ingg a mont
month h or mor
moree of eac
eachh work
work shif
shiftt or sche
schedu
dule
le
(D)
(D) chang
changing
ing the work
work schedu
schedules
les of the day
day and
and night
night shift
shift work
workers
ers

PRACTICE BOOKLET
BKL PhiLSAT_AA_0317 GO ON
ON TO THE NEXT
NEXT PAGE
PAGE
35

Selection
Selection 6

Trav
Travel
elin
ing
g to us from
from the
the burn
burnin
ingg hear
heartt of the sun
sun 93 mill
millio
ion
n mile
miless away,
away, light
light not
not only
only
illuminate
illuminatess our
our little planet
planet but
but create
createss on
on its thin surface
surface the essentials
essentials of life itself.
itself. No wonder 
wonder 
men from
from ancien
ancientt times,
times, awed
awed by its glor
gloriou
iouss light,
light, worsh
worshipe
ipedd the sun
sun for cent
centuri
uries.
es. Its wors
worship
hip
spran
sprang g from
from true
true intuit
intuition
ion since
since in the light
light that
that comes
comes fro
from
m this
this nuclea
nuclearr ball
ball of fire the secret
secret of 
of                 
life on this
this plane
planett does,
does, in fact,
fact, dwell.
dwell. All life
life is the
the offspr
offspring
ing of the
the sun,
sun, for the
the light
light pouri
pouring
ng from
from
it is a wonderf
wonderfulul chemi
chemist.
st. Leaves
Leaves are chemical
chemical factories
factories in which
which sunlight
sunlight and water
water are
are turned
turned
into
into food
food and
and from
from which
which life-g
life-givi
iving
ng oxygen
oxygen is poure
poured d into
into the air for
for man
man to breath
breathe.
e.

Light
Light m
moves
oves to us through
through the atmosphe
atmosphere
re in waves
waves of different
different lengths,
lengths, each carrying
carrying its

own
ow
than
than color.
color.
let,, The
n violet
vio Th
noeneeyes
none eyesger
seethan
longer
lon only
onlyred.
than a small
redsma llhtsegme
. Light
Lig seg
fament
lls nt
falls of the
from
from this
thissky
great
great range
rang
e theepetal
like
lik oftals
pe wavel
was veleng
engths
thss;– every
of flower
flo wers; none
non
ev e drop
ery shorter 
shor
dropter 
of                 
dew is a glory
glory in miniat
miniature
ure and we who
who live
live by the blessi
blessing
ng of this
this distan
distantt star
star can learn
learn to rejoic
rejoicee
in it.
it.

Anyo
Anyone ne can
can see
see how
how whit
whitee brea
breaks
ks into
into colo
colorr by hold
holdin
ingg a refr
refrac
acti
ting
ng pris
prismm in a narr
narrow
ow
beam
beam of sunli
sunligh
ghtt in a darke
darkenened
d room
room and
and projec
projectin
ting g the result
result onto
onto a white
white scree
screen.
n. There
There you
you will
will
discover
discover the band of colors
colors that
that we call the solar
solar spectru
spectrum.m. Similarly
Similarly,, rainbows
rainbows are caused
caused by the
refraction
refraction and reflection
reflection of sunlight
sunlight from the raindrops
raindrops of a vanishin
vanishingg storm.
storm. Today
Today we know
know more
about
about light
light than
than our
our ancest
ancestors
ors did.
did. Our
Our scient
scientist
istss know
know enoug
enough h to use the secret
secretss of the
the sun
sun as a
stepp
stepping
ing-st
-ston
onee to the stars
stars or
or as the
the end
end of life on earth
earth..

22. The sun emits life-givin


life-giving
g light which
which travels
travels through
through space
space in

(A)
(A) equal
equal wave
wavelen
length
gthss
(B)
(B) even
evenly
ly colo
colore
red
d wave
wavess

(C)
(D) spurts
(D) vario
va uslike
rious gusts
wavel
wavelen of wind
ength
gths
s and colors
colors

23. The
The autho
authorr seems
seems to consi
consider
der the
the sun
sun as a source
source of                 

(A)
(A) natur
natural
al calam
calamitie
itiess like
like droug
drought
ht
(B)
(B) ener
energy
gy and
and beau
beauty ty that
that man
man can
can enjo
enjoy
y
(C)
(C) threat
threat to life
life becau
because se of its nucle
nuclear
ar powe
power r 
(D)
(D) nucle
nuclear
ar powe
powerr that
that is bein
being
g waste
wasted
d in space
space

24. As a whole
whole,, the purpo
purpose
se of the select
selection
ion is to
to

(A) describe
describe man’s
man’s ancient
ancient worship
worship of the sun
(B)
(B) prai
praise
se the
the beau
beauty
ty of the
the sun
sun and
and its impo
import
rtan
ance
ce to life
life
(C)
(C) state
state the possi
possibil
biliti
ities
es of
of the
the tremen
tremendo
dous
us pow
powerer of
of the
the sun
sun
(D)
(D) discus
discusss the
the effort
effortss of
of man
man to discov
discover
er the secret
secretss of
of the sun
sun

PRACTICE BOOKLET
BKL PhiLSAT_AA_0317 GO ON
ON TO THE NEXT
NEXT PAGE
PAGE
36

Selection
Selection 7

I was
was born
born in a log
log cabi
cabinn on a wint
winterer.. The
The firs
firstt thi
thing
ng I rem
remem
embe berr is
is bei
being
ng grat
gratef
eful
ul for 
for 
wind
window
ows.s. I was thr
three
ee year
yearss old.
old. My moth
mother
er had
had set
set me to play
play on a matt
mattre
ress
ss care
carefu
fully
lly plac
placed
ed in
the one
one ray of sun
sunlig
light
ht stre
stream
aming
ing throu
through
gh the one
one glass
glass wind
window ow of our
our log
log cabin.
cabin. Baby
Baby as I was,
was, I
hadachedin the agonizi
agonizingng cold
cold of a pione
pioneer
er winter.
winter. Lyingthere
Lyingthere,, warme
warmed d by that
that blesse
blessed
d sunsh
sunshine
ine,,
I was suddenly
suddenly aware
aware of wonde
wonderr and joy and gratitude
gratitude.. It was
was gratitud
gratitudee for glass,
glass, which
which could
could keep
out
out the biti
biting
ng cold
cold and
and let
let in the
the warm
warm sun.
sun...
..

My father
father came
came from
from a family
family of schoo
schooll teache
teachers
rs in New
New Englan
England.
d. My mothe
motherr was
was the
daughter
daughter of a hardwork
hardworking
ing Scotch immigrant
immigrant.. Father’s
Father’s family
family set store
store on ancestry.
ancestry. Mother’s
Mother’s side

was more practical.


practical...
..
The year
year befo
before
re my birth,
birth, these
these two youn
young
g peop
people
le had
had starte
started
d West
West in a prairie
prairie schoo
schoone
nerr to
stake
stake a home
homeste
stead
ad claim.
claim...
..

Afte
Afterr moth
mother
er’s
’s eig
eight
hth
h and
and last
last bab
baby,
y, she
she lay
lay ill for
for a year
year.. The
The care
care of the
the chil
childr
dren
en fell
fell
  princip
principall
ally
y on my youn
youngg should
shoulders
ers.. One
One day I found
found her cryi
crying
ng..

“Mary
“Mary,”,” she said,
said, with
with a tende
tenderne
rness
ss that
that was
was rare
rare,, “If I die,
die, you
you must
must take
take care
care of all
all your 
your 
  brothe
brothers
rs and
and sister
sisters.
s. You
You will
will be the only
only woman
woman within
within eighte
eighteenen miles
miles.”
.”

I was ten years old.


old.

That night
night and
and many
many other
other nights
nights I lay awake,
awake, trembling
trembling at the
the possibilit
possibility
y of being
being left the
only
only woma
woman
n within
within eighte
eighteen
en miles.
miles.

But
Butead
his homes t moth
mother
homestead erner
didcould
partner
part cnot
not die.
oulddie. I e,
must
must
spare,
spar have
hav
I kept
kep e been
bee
t that n a esturdy
home
hom sturdygchild
going
goin child,, she
until for,
for, was
with
withstrong
the little
little
strong help
help fathe
again...
agai n... fatherr and
and

Every
Every fall, the shoemake
shoemakerr made his rounds
rounds through
through the country,
country, reaching
reaching our place
place last, for 
  bey
beyon
ond d us lay only
only untame
untamed d forest
forest and wild
wild beast
beasts.
s. His visit
visit thril
thrilled
led us more
more than
than the arriva
arrivall of                 
any
any king
king toda
today.
y. We hadhad been
been cut
cut off from
from the
the worl
world d for
for month
months. s. The
The shoe
shoema
make
kerr broug
brought
ht news
news
from
from neigh
neighbo bors
rs eighte
eighteen,
en, forty,
forty, sixty,
sixty, even
even a hund
hundred
red and fifty
fifty miles
miles away.
away. Usual
Usuallyly he broug
brought
ht a
few newsp
newspapeapersrs too,
too, treasu
treasured
red afterw
afterward
ard for mont
months
hs.. He remain
remaineded a royal
royal guest
guest,, for many
many days,
days,
until all
all the family
family was shod..
shod....

By the time I was


was fourteen
fourteen,, three
three tremendo
tremendous
us events
events had marked
marked my life: sunlight
sunlight through
through
a windo
windowp
wpane
ane,, the log-ro
log-rolli
lling
ng on the river
river when fathe
fatherr added
added tworooms
tworooms to our
our cabin,
cabin, andthe night
night
I thoug
thought
ht mothe
motherr would
would die and leave
leave me the only
only woman
woman within
within eighte
eighteen
en miles.
miles...
..

But there
there was a fourth
fourth event
event that
that was
was the most tremendo
tremendous.us. One night
night father
father handed
handed my
mother
mother a letter
letter.. Our
Our Great
Great-Au
-Aunt nt Mart
Marthaha had
had willed
willed fathe
fatherr her
her house
househol
holdd good
goodss and
and person
personal
al
belo
belong
ngin
ings
gs and
and a mode
modestst sum
sum that
that to
to us was
was a fort
fortun
une.e. Some
Someononee back
back East
East “aw
“awai
aite
ted
d his
his
instru
instructi
ction
ons.”
s.” Many
Many discus
discussio
sions
ns followe
followed,d, but in the end my mothe
motherr gaine
gainedd her way.
way. Great
Great-Aunt
-Aunt
Marth
Martha’s
a’s househ
household old good
goodss were
were sold
sold at aucti
auction
on.. Fathe
Father,
r, howe
however
ver,, insist
insisted
ed that
that her “per
“perso
sonal
nal
  belong
belonging
ings”
s” be shipp
shipped
ed to us...
us...

PRACTICE BOOKLET
BKL PhiLSAT_AA_0317 GO ON
ON TO THE NEXT
NEXT PAGE
PAGE
37

Afte
Afterr a long
long,, long
long wait
wait,, one
one day
day a trun
trunkk and
and two
two pack
packin
ingg case
casess cam
came.
e. It was
was a sole
solemn
mn
moment
moment when the first box was opened.
opened. Then mother
mother gave
gave a cry of delight.
delight. Sheets
Sheets and bedspre
bedspreads
ads
edged
edged with lace!
lace! Real
Real linen
linen pillowcas
pillowcases
es with
with crochete
crocheted
d edgings
edgings.. Soft woolen
woolen blankets
blankets and bright
bright
handmade
handmade quilts.
quilts. Two heavy,
heavy, lustrous
lustrous tablecloth
tableclothss and two dozen
dozen napkins,
napkins, one white
white set hemmed,
hemmed,
and one
one red-an
red-and-w
d-whit
hite,
e, borde
bordered
red with
with a soft
soft fring
fringe.
e.

What
What the
the world
world calls
calls wealt
wealth
h has
has come
come to me in years.
years. Nothi
Nothing
ng ever
ever equal
equaled
ed in my eyes
eyes the
the
  priceless value
value of Great-Aunt
Great-Aunt Martha’s “personal belongings.”

I was
was in a seve
sevent
nth
h heav
heaven
en of delig
delight
ht.. My fath
fatherer pick
picked
ed up the
the book
bookss and
and bega
begann to read
read,,
  paying
paying no attention
attention to our excla
exclamatio
mations
ns over
over dresse
dressess and
and ribbons,
ribbons, the boxful
boxful of laces,
laces, or the little

shell-c
shell-cov
overe
ered
d case
case holdi
holding
ng a few orname
ornaments
nts in gold
gold and silve
silverr and jet.
jet. Then
Then I picked
picked up a napkin
napkin..
“What
“What are these
these for?”
for?” I asked
asked curiously
curiously..

My fathe
fatherr slam
slammed
med his book
book shut.
shut. I had
had never
never seen
seen such
such a look
look on his face.
face.

“How
“How old are you,
you, Mary?
Mary?”” he deman
demanded
ded sudde
suddenly
nly..

I told him that


that I was going
going on fiftee
fifteen.
n.

“And
“And you
you never
never saw a table
table napkin
napkin?”
?”

His tone was bitter


bitter and accusing
accusing.. I didn’t understan
understand—
d—ho
how
w coul
could
d I?

Fathe
Fatherr bega
began
n to talk,
talk, his words
words growin
growingg more
more and more
more bitter
bitter.. Mothe
Motherr defe
defend
nded
ed herse
herself 
lf                 
hotly.
hotly. Today,
Today, I know
know that justice
justice was on
on her side.
side. But in that first
first adoles
adolescent
cent self-c
self-consc
onscious
iousness
ness,,

my sympathie
sympathiess were
were all with
with father.
father.
Mothe
Motherr had neglec
neglected
ted us 
us —she had not
not taugh
taughtt us to use
use table
table napk
napkins
ins!! Peop
People
le in histor
history
y
used
used them.
them.

From
From that
that time
time on,
on, we used
used napki
napkins
ns and a tablec
tableclot
loth
h on Sund
Sundays
ays..

  An excer
excerpt from “ The
pt from                 Log-     
                Cabin Lady: An Anonymous Autobiography”      by
        Marie
                    Mattingly
  Meloney                   

25. What
What was the
the proof
proof that Mary
Mary was a healthy
healthy ten-year
ten-year old?
old?

(A)
(A) She
She lear
learne
nedd to hunt
hunt for
for food
food..
(B)
(B) She
She neve
neverr got
got sic
sick
k in spit
spitee of the
the hard
hardsh
ship
ips.
s.
(C)
(C) She
She helpe
helpedd her father
father cut the logs
logs for
for their
their cab
cabin.
in.
(D)
(D) She
She ran
ran the
the hous
househehol
old
d when
when herher mothe
motherr was
was sick.
sick.

26. Who
Who is tellin
telling
g the story?
story?

(A) The father 


(B) The mother 
(C) The sho
shoemak
maker 
(D)
(D) The
The elde
eldest
st daug
daught
hter 
er 

PRACTICE BOOKLET
BKL PhiLSAT_AA_0317 GO ON
ON TO THE NEXT
NEXT PAGE
PAGE
38

27. What
What experien
experience
ce made the greatest impact
impact on Mary’s life?

(A)
(A) Roll
Rollin
ingg logs
logs on
on the
the river
river wit
withh her
her fath
father 
er 
(B)
(B) The
The fear
fear of
of bein
being g orph
orphananed
ed by her
her moth
mother 
er 
(C)
(C) Inhe
Inheri
ritin
ting
g thei
theirr grea
great-t-aunt’s
aunt’s personal belongings
(D)
(D) The
The warmt
warmth h of sun
sunlig
light
ht stre
stream
aming
ing throu
through
gh the glass
glass wind
window
ow

28. Besides making


making their shoes, why was the shoemake
shoemaker’s
r’s visit eagerly
eagerly awaited?
awaited?

(A)
(A) He help
helped
ed in clea
cleari
ring
ng the
the land
land..
(B)
(B) He broug
brought
ht supp
supplie
liess for
for the family
family..

(C)
(D) He brou
(D) He enter
enough
br tertai
taine
ghttned
d the
news
new child
chm
s from
fro ildren
thren
the with
wit
e outs
ou h ehis
tsid
ide tricks
tri
worl
wo cks.
d. .
rld.

29. What does the statemen


statement,
t, “He
“He remain
remained
ed a royal
royal guest
guest”” mean?

(A)
(A) He was
was a give
giverr of gift
gifts.
s.
(B)
(B) He was
was a loya
loyall sub
subje
ject
ct of the
the king
king..
(C)
(C) He rece
receiv
ived
ed the
the best
best trea
treatm
tmenentt from
from the
the peop
people
le..
(D)
(D) He was
was know
knownn to ever
everyb
ybod
ody y in the
the comm
commununity
ity..

30. How
How many
many siblin
siblings
gs did Mary
Mary have?
have?

(A) Seven
(B) Eight
(C) Nine
(D) Ten

31. What
What triggered
triggered the bitter
bitter argumen
argumentt between
between husband
husband and wife?

(A) The wife’s


wife’s decis
decision
ion to sell great
great-aunt Martha’s household
household goods
(B) The wife’s
wife’s insistenc
insistencee on using
using table napkins
napkins on Sundays
Sundays only
(C)
(C) The
The real
realiz
izat
atio
ion
n that
that the
the child
childre
ren
n did
did not
not have
have tabl
tablee mann
manner
erss
(D) The
The wife’s
wife’s failu
failure
re to teach
teach her child
children
ren ta 
ta ble manners
  ______________________________________________________________________________ 

Selection
Selection 8

During
During the 1970’s,
1970’s, experts
experts predictedthat
predictedthat the world
world populati
population
on of three
three and one-thirdbillion
one-thirdbillion
woul
would
d doub
doublele in the
the next
next thi
thirt
rty
y year
years.
s. Acco
Accord
rdin
ing
g to Dr.
Dr. Paul
aul Erli
Erlich
ch,, a scie
scient
ntis
istt at Sta
Stanf
nfor
ord
d
Unive
Universi
rsity,
ty, popu
populat
lation
ion explos
explosion
ion was
was causing
causing mankin
mankindd “…to
“…to run out
out of our food resou
resource
rces,
s, and
we were destroyi
destroying
ng the ecologica
ecologicall system
system of our planet
planet which
which keeps
keeps us alive.
alive.””

In the PhilipPhilippin
pines
es,, 3,295
3,295 babies
babies were
were being
being added
added to our
our popula
populatio
tion
n every
every day,
day, that
that is, two
  babi
babies
es were
were beinbeing g born
born ever
every
y one-
one-ha
half
lf minut
minute,
e, repo
report
rted
ed Dr. Juan
Juan Salce
alcedo
do Jr.,
Jr., who was
was then
then
Chairm
Chairmanan of      the
  National
National Scien
Science
ce Develop
Development
ment Board.
Board. Our country
country’s
’s yearly
yearly populati
population
on growth
growth
of 3.5
3.5 perce
percent
nt was was con
consid
sidere
eredd to be one
one of the
the highe
highest
st in the world
world..

PRACTICE BOOKLET
BKL PhiLSAT_AA_0317 GO ON
ON TO THE NEXT
NEXT PAGE
PAGE
39

An inte
intere
rest
stin
ing
g aspe
aspect
ct of
of the
the popu
popula
latio
tionn issu
issuee was
was brou
broughghtt out
out by Dr.
Dr. Merc
Mercededes
es B.
B.
Concepci
Concepcion,on, who was then
then Director
Director of the
the Populati
Population
on Institute
Institute of the Universit
Universityy of the Philippin
Philippines.
es.
She reported
reported that almos
almostt fifty percent
percent of
of the Filipinos
Filipinos were below
below fifteen
fifteen years
years of age.
age. Mr. Alfredo
Alfredo
R. Roces,
Roces, columnis
columnistt of the Manila
Manila Times,
Times, predicted
predicted that the vast majority
majority of our society
society would,
would, in
the near
near futur
future,
e, be compr
compriseisedd of teena
teenager
gers.
s. In conne
connecti
ction
on with
with this,
this, one
one aspect
aspect that
that neede
needed d close
close
stud
study,
y, acco
accordrdin
ingg to Dr.
Dr. Conc
Concepepci
cion
on,, was thethe omin
ominou ouss repe
repercrcus
ussi
sion
on whic
whichh may
may upse
upsett our 
our 
establishe
establishedd economi
economicc structure
structure because
because of a very young
young populati
population.
on. Also noted
noted by her was that the
rapid
rapid growth
growth of the popu
populat
lation
ion here
here had
had broug
broughtht about
about a heavy
heavy depen
depende dency
ncy rate
rate among
among Filipi
Filipino
nos.
s.
She
She stated
stated that
that there
there were
were about
about eighty
eighty-ni
-nine
ne child
child depen
dependen
dents
ts for every
every one
one hundr
hundred
ed worke
workersrs in
this
this cou
count
ntry
ry.. Furt
Furthe
herm
rmorore,
e, a reve
revela
latio
tionn of for
forme
merr Supr
Suprem
emee Cour
Courtt Chie
Chieff Just
Justic
icee Robe
Robertrto
o
Concepcion
Concepcion revealed that only seven seven percent
percent of the Filipinos
Filipinos had a “living”
“living” income,
income, while ninety
ninety--

three
three perce
percent
nt lived
lived on a “hand
“hand-to-
-to-mou
mouth”
th” existenc
existence.
e.
Among
Among the myriads
myriads ofof solutio
solutions
ns to
to the populati
population
on problems
problems that have cropped
cropped up was one
that
that adv
advoca
ocated
ted the accele
accelerat
ration
ion of the growth
growth of the econo
economymy to 10.5
10.5 percen
percentt eve
every
ry year
year whic
whichh
meant
meant that our
our econo
economic
mic productio
production n should
should be increase
increased
d three times the
the growth
growth of our
our populati
population;
on;
that we should
should practice
practice austerity
austerity of consump
consumption
tion and austerity
austerity of imports.
imports. Another
Another solution
solution being
being
advocated
advocated was that
that the affluent
affluent section
section of our
our society
society should
should cut their
their standar
standardd of living in half and
and
use
use that
that mon
moneyey to impr
improv
ovee and
and grow
grow more
more food
food.. Anot
Anothe
herr solu
soluti
tion
on,, whic
whichh was
was per
perha
haps
ps bein
beingg
cons
consid
ider
ered
ed,, was
was the
the expo
export
rtat
atio
ion
n of huma
humansns to the
the moon
moon.. One
One solu
soluti
tion
on whic
which h was
was caus
causin
ingg a
confusin
confusing g debacle
debacle of minds
minds was concern
concerning
ing the use of the pill.

32. Dr. Paul


Paul Erlich
Erlich believed
believed that populati
population
on explosio
explosion
n would
would

(A) bring
bring about
about hung
hunger
er througho
throughoutut the world
world
(B) control
control man’s
man’s dream
dream to explore
explore outer
outer space
(C) force
force man to explore
explore the earth’s
earth’s natural
natural resource
resourcess

(D)  bring about stability and bridge the gap between the rich and the poor 
33. Acco
Accordi
rding
ng to the selec
selectio
tion,
n, Dr. Merce
Mercede
dess B. Conc
Concepc
epcion
ion feared
feared that
that the

(A)
(A) Filipi
Filipino
noss might becom
becomee too depen
dependen
dentt on other
other countr
countries
ies
(B)
(B) youn
youngg peop
people
le migh
mightt esta
establ
blis
ish
h an econ
econom
omy y base
based
d on soci
social
alis
ism
m
(C)
(C) influe
influence
nce of a youn
youngg popul
populati
ation
on might
might upset
upset our
our econ
econom
omicic struc
structur
turee
(D)
(D) youn
youngg popu
populat
lation
ion might
might contro
controll the econo
economic
mic reins
reins of the gover
governm
nmenentt

34. Accordin
According
g to popu
populat
lation
ion expert
experts,
s, during
during the 1970
1970’s,
’s, the earth’s
earth’s popu
populat
lation
ion for the next
next thirty
thirty
years
years would
would be

(A) 1.6 billio


lion
(B) 3.3 billio
lion
(C) 3.5 billio
lion
(D) 6.6 billio
lion

PRACTICE BOOKLET
BKL PhiLSAT_AA_0317 GO ON
ON TO THE NEXT
NEXT PAGE
PAGE
40

Selection
Selection 9

(1) In everyd
everyday
ay life,
life, we tend
tend to assign
assign less
less weigh
weightt to oral
oral speech
speech than
than to the writte
writtenn word.
word.
The
The spok
spoken
en is pres
presum
umed
ed to be “eph
“ephememer
eral
al,”
,” liabl
liablee to be mish
mishea
eard
rd,, rei
reinter
nterpr
pret
eted
ed,, or deni
denied
ed.. The
The
writte
written
n word,
word, in contr
contrast
ast,, is thoug
thought
ht to be “inde
“indelib
lible,
le,”” its meanin
meaning g cast
cast in stone.
stone. But,
But, the litera
literary
ry
scholar
scholar Rolan
Rolandd Barthes,
Barthes, in his classi
classicc essay,
essay, “The Death
Death of the Author,”
Author,” reverses
reverses this equation.
equation. He
argue
argues:
s: “[I]t
“[I]t is ephem
ephemera
erall speech
speech which
which is indeli
indelible
ble,, not
not monume
monumenta ntall writing
writing….…. Speec
Speech h is
irreversib
irreversible:
le: a [spoken]
[spoken] word cannot
cannot be retracted…
retracted…””

(2) Nowhere, perhaps, is this insight truer


truer than in international gatherings of heads
heads of state
or foreign
foreign ministers,
ministers, where
where diplomatic
diplomatic language
language is the norm.
norm. Participa
Participants
nts studious
studiously
ly avoid
avoid giving
giving
off-the-c
off-the-cuff
uff remar
remarks
ks in order
order not
not to be misinterpr
misinterpreted
eted.. Most
Most speak
speak from detailed
detailed notes
notes or simply
simply
read
read a prepa
prepared
red speech
speech.. At the
the end of suc
such
h meetin
meetingsgs,, a carefu
carefully
lly crafte
crafted
d commu
communiqniqué
ué is typ
typica
ically
lly
agreed
agreed upon
upon and issue
issued
d to make
make sure
sure there
there is no
no room
room for misun
misunder
dersta
stand
nding
ing..

(3)
(3) In
In such
such gath
gather
erin
ings
gs,, word
wordss do
do matt
matter
er.. Utmo
Utmostst cord
cordia
iali
lity
ty is obse
observ
rved
ed even
even amon
among g
represent
representative
ativess of rival
rival nations.
nations. That is why
why presiden
presidents,
ts, prime ministers,
ministers, and monarch
monarchss bring with
them
them a staff
staff of season
seasoneded diplom
diplomatsats and
and write
writers
rs who prepa
prepare
re the groun
groundd for their
their princ
principa
ipals
ls long
long
before
before the latter
latter actua
actually
lly meet.A
meet.A stro
strong
ng opini
opinionloose
onlooselyly uttere
uttered
d byone head
head of state
state inthe presen
presence
ce
of anothe
anotherr could
could be taken
taken as a slight.
slight. The conseque
consequences
nces that follow
follow could
could be costly,
costly, and unfortun
unfortunate
ate
because unintended.
unintended.

(4) It is one
one thin
thing
g for
for a nati
nation
on’s
’s lead
leaderer to spea
speakk and
and act
act in a cali
calibr
brat
ated
ed way
way in thethe purs
pursui
uitt
of a clea
clearr and
and cohe
cohere
rent
nt for
forei
eign
gn poli
policy
cy.. It is
is qui
quite
te anot
anothe
herr to allo
allow w ones
onesel
elf,
f, espe
especi
cial
ally
ly on the
the
intern
internati
ation
onal
al stage,
stage, to be a captiv
captivee of the
the conti
contingngenc
encies
ies of oral
oral speec
speech. h. “Spee
“Speech
ch can be a stream
stream of                 
cons
consci
ciou
ousn
snes
ess,
s, with
with unfi
unfininish
shed
ed utte
uttera
ranc
nces
es,, half
half-f
-for
orme
medd thou
though
ghts
ts,, and
and a heal
healththy
y smat
smatteteri
ring
ng of         
mess
messyy slan
slang.
g. You
You don’
don’tt have
have to know
know how
how a sentsenten
ence
ce will
will end
end bef
before
ore you
you star
startt it.”
it.” (Chi
(Chi Luu,
Luu, “Is
“Is
writing
writing a technolo
technology
gy or language
language?”)?”) It is from this thatthat extempora
extemporaneo neous
us speech
speech draws
draws its charm
and power,
power, as well as its inherent
inherent risks.
risks. This is partic
particularl
ularly
y so if one
one happens
happens to be a head of state.state.

(5) Under
Under our system
system of gove
governme
rnment,
nt, the Presiden
Presidentt is vested
vested with the power
power to determin
determinee
foreign
foreign policy
policy and to conduct
conduct foreign
foreign relations
relations with other
other states.
states. He alone
alone can negotiate
negotiate treaties
treaties on
behalf
behalf of the country,althoug
country,although, h, under
under the 1987
1987 Constitu
Constitution,
tion, suchtreaties
suchtreaties are subject
subject toto ratificat
ratification
ion
  by the
the Sena
Senatete.. A 1988
1988 Supr
Suprem
emee Cour
Courtt rulin
ruling
g denyi
denying
ng medi
mediaa acce
access
ss to minu
minute
tess taken
taken at a treat
treaty
y
negotiatio
negotiationn (G.R.
(G.R. No. 84642)
84642) made this very clear:
clear: “The
“The Presiden
Presidentt alone
alone has
has the powe
powerr to speak 
speak 
or listen as a represen
representativ
tativee of the Nation.
Nation. Congres
Congresss itself
itself is powe
powerless
rless to invade
invade it. The Presiden
Presidentt
is the sole
sole organ
organ of the Natio
Nationn in its extern
external
al relati
relation
ons,s, and
and its sole
sole repres
represen
entat
tative
ive with
with foreig
foreignn
  Nations.”

(6) This respon


responsibili
sibility
ty makes
makes it imperative
imperative for any presiden
presidentt to always
always speak
speak with caution.
caution.
Whether
Whether it is to communi
communicate cate a substanti
substantiveve shift
shift in
in foreign
foreign policy,
policy, or it is simply
simply to reiterate
reiterate time-
honore
honored
d princi
principle
pless in foreig
foreignn relati
relation
onss —such
such as sove
soverei
reign
gn equali
equality
ty and nonin
noninter
terfer
feren
ence
ce  —it is
important
important for any administr
administratio
ation
n to clearly
clearly state if it is veering
veering toward
toward a radical
radical rethinkin
rethinking g of the
country’s
country’s existing
existing alliances.
alliances. If it is, our people
people deserve
deserve to know.
know.

(7) The Presiden


Presidentt canno
cannott say
say one thing and then,
then, immediatel
immediately y after,
after, his staff scramble
scrambless to
redesc
redescrib
ribee what
what he actua
actually
lly said.
said. If done too often
often,, it can unde
undermi
rmine
ne the Presid
President
ent’s
’s credibi
credibility
lity.. In
an era where
where speech
speeches
es are
are instan
instantly
tly record
recorded,
ed, trans
transcri
cribe
bed,
d, and retrie
retrieve
ved,
d, there
there is hardl
hardly
y any
any room
room

PRACTICE BOOKLET
BKL PhiLSAT_AA_0317 GO ON
ON TO THE NEXT
NEXT PAGE
PAGE
41

for widely differing


differing interpreta
interpretations
tions.. Fortunat
Fortunately,
ely, the rules
rules of diplomacy
diplomacy give parties
parties enoughleeway
to offer
offer or demand
demand clarificat
clarifications
ions of contro
controvers
versial
ial statemen
statementsts  — precisely because words have
contexts
contexts and conseque
consequences
nces.. Parties
Parties at the receiving
receiving end of strong
strong statement
statementss often
often want to know
know if                 
there
there is a hidden
hidden messa
messagege behind
behind these
these uttera
utterance
nces.s.

An   excerpt from “Speech and the written


written word” by Randy David,
http://opinion.inquirer.net/97234/speech-and-the-written-word#ixzz4azunO3IX      

35. Which
Which of the follow
following
ing is highli
highligh
ghted
ted by the write
writerr in his essay
essay??

(A)
(A) The
The impo
import
rtan
ance
ce of dipl
diplom
omac
acyy
(B)
(B) The
The impo
import
rtan
ance
ce of prepa
prepare
red
d spee
speech
ches
es
(C) The mandated
mandated power
power of the
the presiden
presidentt
(D)
(D) The
The diff
differ
eren
ence
ce betw
betwee
een
n oral
oral spee
speech
ch and
and writt
written
en word
word

36. What
What writin
writing
g style
style did the write
writerr use to develo
develop
p his essay
essay??

(A)
(A) Deduc
educti
tive
ve
(B) Indu
nductiv
tive
(C)
(C) Induc
Inductiv
tive-d
e-ded
educt
uctive
ive
(D)
(D) Dedu
Deducti
ctive
ve-in
-indu
ducti
ctive
ve

37. In the first


first three paragr
paragrap
aphs,
hs, the writer
writer implies
implies the import
importan
ance
ce of             

(A) since
inceri
rity
ty
(B)
(B) blun
bluntn
tnes
esss
(C)
(C) tact
tactfu
fuln
lnes
esss
(D)
(D) cunn
cunnin
ingn
gnesesss

38. Accordi
According
ng to the author,
author, communic
communication
ation,, whether
whether written
written or spoken,
spoken,

(A)
(A) shou
should
ld be care
carefu
fully
lly craf
crafte
ted
d
(B)
(B) can
can be misu
misund
nder
erst
stoo
oodd
(C)
(C) shou
should
ld be rehe
rehear
arse
sed
d
(D)
(D) can
can spre
spread
ad quic
quickl
kly
y

39. What is the


the attitude of the writer towards extemporaneous
extemporaneous speech of dignitaries?

(A)
(A) Indi
Indiff
ffer
eren
ence
ce
(B)
(B) Relu
Reluct
ctan
ance
ce
(C) Tolera
lerannce
(D)
(D) Passi
assivi
vity
ty

PRACTICE BOOKLET
BKL PhiLSAT_AA_0317 GO ON
ON TO THE NEXT
NEXT PAGE
PAGE
42

40. Which
Which of the follow
following
ing is an import
important
ant consid
considera
eratio
tion
n when
when speaki
speaking
ng as implied
implied in the
  passage?

(A) Audienience
(B) Autho
thority
rity
(C) Fllu
ueency
(D) Contextext

41. Which
Which of the
the follow
following
ing words
words can
can be used
used synon
synonym
ymou
ously
sly with
with the inten
intended
ded meanin
meaning
g of the
the
writer
writer in paragrap
paragraph h 7?

(A)
(B) Retract
Restate
(C) Rectif
tify
(D) Rephrase
rase

42. The
The essa
essay
y as a whol
wholee is true
true to

(A)
(A) cele
celebr
brit
itie
iess
(B)
(B) head
headss of stat
states
es
(C)
(C) chur
church
ch offi
offici
cial
alss
(D)
(D) peop
people
le from
from all walks
walks of life
life
  ______________________________________________________________________________ 

Selection
Selection 10

When
When a hacker
hacker claime
claimed
d to have
have breach
breached
ed Mitt
Mitt Romney’s personal
personal email account in 2012,

he didn’t
didn’t do
  password
password itking
by
by infect
cracking
crac infecting
ing
k  —his
attack 
attac compu
computer
he did ter with
it with wit
thehword
a data
dat“Seamus.”
a -leeching
-leeching virus
virus or by launchin
launching
g a brute-fo
brute-force
rce

Seam
Seamus us was
was the
the name
name of Rom Romneney’
y’ss dog,
dog, and
and appa
appare
rent
ntly
ly the
the answer
answer to his passwo
passwordrd
reminder
reminder questio
question,
n, “what
“what is your
your favorit
favoritee pet.” Because
Because Romney’
Romney’ss email
email address
address had
had been made
made
publ
public
ic in a news
news sto
story
ry seve
severa
rall days
days earl
earlie
ier,
r, and
and the
the dogg
doggo-
o-in
in-q
-que
uest
stio
ionn was
was the
the subj
subjec
ectt of an
unfort
unfortun
unate
ate media
media scanda
scandall for havin
havingg been
been strapp
strapped
ed to the roof
roof of the family
family car durin
duringg a 1983
1983
road
road trip
trip,, the
the alle
allege
gedd hack
hacker
er had
had ever
everyt
ythi
hingng he need
neededed to expl
exploi
oitt a noto
notoririou
ousl
sly
y weak
weak gate
gatewa
wayy to
  password
password security:
security: the
the password
password recovery
recovery question.
question.

Whil
Whilee sett
settin
ing
g a pass
passwo
wordrd remi
remind
nder
er ques
questio
tion
n is a fine
fine idea
idea in theo
theory
ry (so
(so many
many pass
passwo
wordrds,
s,
so little mental
mental space!),
space!), it has probably
probably encourag
encouraged ed you to make your passworpassword d overly
overly vulnerab
vulnerable.le.
The
The simp
simple
le trut
truth
h is tha
thatt in our
our age
age of soc
socia
iall medi
mediaa over
over-s -sha
hari
ring
ng,, it is far
far too
too easy
easy to suss
suss out
out
anyone’s
anyone’s answers
answers toto the question
question “where
“where did you meet your your spouse
spouse,” ,” or “wha
“whatt is your
your mother’
mother’ss
maid
maidenen name
name.”
.” If you
you have
have a publ
public
ic Face
Facebo
book
ok,, Twitt
Twitter
er,, or Inst
Instag
agraramm acco
accoun
unt,
t, you
you also
also have
have a
dossier
dossier of clues
clues for would-
would-bebe hackers to peruse
peruse at will. Many security
security industry
industry professi
professional
onalss wish
the passwor
password d reminder
reminder questio
questionn would
would be outright
outright abolish
abolisheded from account
account setup,
setup, but until
until that day
comes,
comes, what
what can you
you do to work
work with
with the system
system and keep
keep yours
yourself
elf secure
secure??

For
For one
one thing
thing,, pick
pick a harde
harderr ques
questio
tion.
n. A Micro
Microso
soft
ft and
and Carne
Carnegiegie Mello
Mellonn study
study found
found that
that
the safest
safest passwor
password d reminder
reminder quest
question
ion may be “What’s
“What’s your
your father
father’s
’s middle
middle name,”
name,” as it’s
it’s easy to
reme
rememb
mberer,, hard
hard to gues
guess,s, and
and unlik
unlikel
ely
y to be publ
public
ic know
knowleledg
dgee on the
the Inte
Intern
rnet
et.. (Oth
(Other
er safe
safe-i
-ish
sh

PRACTICE BOOKLET
BKL PhiLSAT_AA_0317 GO ON
ON TO THE NEXT
NEXT PAGE
PAGE
43

question
questionss were,
were, “What
“What was
was your
your first
first phone
phone number?
number?,”
,” “Who
“Who was your
your favor
favorite
ite teacher,”
teacher,” and
“Who is your favorite
favorite singer?”)
singer?”)

Some
Some experts
experts recommen
recommend d answering
answering the question
question with a non sequitur (What is your mom’s
maid
maiden
en name
name?? Plat
Platyp
ypus
us).
). But
But even
even a rand
random
om,, one-
one-wo
word
rd answ
answerer is vuln
vulner
erab
able
le to a brut
brute-
e-fo
forc
rcee
hacking
hacking attack
attack designed
designed to rapidly
rapidly guess
guess every
every combinati
combination
on of letters
letters and numbers
numbers in sequence
sequence..

No matt
matter
er whic
which h secu
securi
rity
ty ques
questi
tion
on you
you go with,ith, your
your best
best bet
bet isis to
to trea
treatt it
it lik
likee
another password 
password —a long
long string
string of letter
letters,
s, numbe
numbers,
rs, and specia
speciall charac
character
terss that
that could
could not
not be
guesse
guessed
d or divine
divined
d from
from a cursor
cursoryy glance
glance at Facebo
Facebook
ok.. Use
Use a full
full phras
phrasee instea
instead d of a single
single word
word
(mayb
(maybee even
even one
one that
that inspir
inspires
es you
you to be a better
better perso
person
n every
every time
time you
you type
type it).
it). Need
Need an examp
example?le?

If you
you pick
pick the
the ques
questi
tion
on,, “wha
“H3NeverC@meWhen1C@lled!”. “whatt was
was your
your fir
first
st dog
dog’s
’s nam
namee ,” a strong answ
answer
er woul
would
d be

Make
Make sure
sure you
you use a uniqu
uniquee answe
answerr for every
every accou
accountnt that
that require
requiress a remind
reminderer quest
question
ion,,
and log them
them in a passw
password
ord manag
manager er along
alongsid
sidee your
your other
other passw
password
ords.
s. Of cours
course,
e, this
this is all moot
moot
for
for the
the esti
estima
mate
ted
d 17%
17% of Amer
Americican
anss who
who secu
secure
re thei
theirr data
data with
with the
the pass
passwo
word
rd “123
“12345456.
6.”” But
But
  progress happens inbaby steps (just maybe
maybe don’t use your baby’s name
name as your passwordreminder 
answer).

  An excerpt from “Your Password Recovery Questions Are Insanely Easy                     


To Hack—and
Hack—and You Might Blame”     b    y    Brandon
Might Be to Blame”                        Specktor 
http://www.rd.com/advice/work-career/password-recovery-questions/

43. In the first


first two paragr
paragrap
aphs,
hs, the writer
writer implie
implied
d that
that

(A)
(A) what
what we
we thou
though
ghtt to be priv
privat
atee is act
actua
ually
lly publ
public
ic
(B)
(B) what
what a public
public figure
figure does
does excite
excitess hacke
hackers
rs
(C) the answ
answer
er to the securi
security
ty quest
question
ion is weak 
weak 
(D)
(D) the
the hack
hacker
er is a geni
genius
us

44. The purpose


purpose of the writer
writer is to

(A) entert
tertaain
(B)
(B) pers
persua
uade
de
(C) anally
yzze
(D) inform

45. Which
Which of the follow
following
ing is most
most likely
likely the reaso
reason
n why securi
security
ty indus
industry
try profes
professio
sional
nalss
would
would like to abolish
abolish the password
password reminder
reminder question
question??

(A)
(A) Answ
Answer
erss are easi
easily
ly deco
decodeded.
d.
(B)
(B) Answ
Answers
ers are identi
identical
cal to others
others..
(C)
(C) Answ
Answers
ers are diffic
difficult
ult to const
construc
ruct.
t.
(D) Answers
Answers are
are most
most of the time forgotten
forgotten..

PRACTICE BOOKLET
BKL PhiLSAT_AA_0317 GO ON
ON TO THE NEXT
NEXT PAGE
PAGE
44

46. The
The articl
articlee as a whole
whole explor
explores
es the
the

(A)
(A) flaw
flawss of
of set
setti
ting
ng up a pass
passwoword
rd
(B)
(B) ways
ays to crea
create
te a pass
passwo
word
rd
(C)
(C) poss
posses
essi
sion
on of an
an acco
accoun
untt
(D)
(D) hack
hackin
ing
g of anan acco
accoun
untt

47. The last paragrap


paragraph
h is an example
example of a(an)
a(an)

(A)
(A) inst
instru
ruct
ctio
ion
n
(B)
(B) sugg
sugges
esti
tion
on

(C) command
(D) demand

48. Which
Which of the
the following
following statemen
statements
ts is most
most likely
likely NOT true?

I. Acco
Accoun
untt holde
holders
rs inten
intentio
tional
nally
ly leave
leave clues
clues upo
upon n passw
password
ord set up.
up.
II.
II. Hack
ackers
ers only attack famous personalities’
personalities’ account
account..
III.
III. Ever
Everyo
yone
ne can
can be a hack
hackin
ing
g vict
victim
im..
IV.
IV. Identi
Identity
ty theft
theft is possi
possible
ble when
when the accoun
accountt is hacked.
hacked.

(A) I and
and II only
(B) I and III only
(C) II an
and III
III onl
only
y
(D) II an
and IV onllyy

49. Which
Which quote
quote can be direct
directly
ly assoc
associat
iated
ed with
with the articl
article?
e?

(A) “It’s more than what


what meets
meets the
the eye.”
eye.”
(B) “The
“The world
world is watch
watching
ing us.”
us.”
(C) “Be
“Be the first
first to
to know.
know.””
(D) “It’s a small world.”
world.”

50. Based
Based on the article,
article, account
account holders
holders should
should

(A) answer
answer remind
reminderer questi
questions
ons discreetl
discreetly
y
(B)
(B) deact
deactiva
ivate
te their
their accou
accounts
nts every
every two
two week
weekss
(C)
(C) aboli
abolish
sh remind
reminder er quest
question
ionss
(D)
(D) skip
skip remi
remind
nder
er quest
questio
ions
ns

PRACTICE BOOKLET
BKL PhiLSAT_AA_0317 GO ON
ON TO THE NEXT
NEXT PAGE
PAGE
45

TEST
TEST D. QUANTITATIVE REASONING

Section
Section 1. Pattern
Pattern Recogniti
Recognitio
Recognition
on
n

Direct
Direction
ions:
s: In this
this test,
test, the stem
stem consis
consiststs of a serie
seriess of numb
numbers
ers.. In each
each series
series,, discov
discover
er the
logica
logicall rule.
rule. Selec
Selectt from the five
five choice
choicess the
the numbe
numberr that shou
should
ld come
come next
next in the series
series..

1. 61 54 47 41 34 29 22

(A)
(B) 16
15 ((E
D)) 1
28
0
(C) 17

2. 16 32 8 48 6 60 5

(A) 50 (D) 70
(B) 55 (E) 75
(C) 80

3. 1 2 4 7 28 33 198

(A) 1,386 (D) 1,188


(B) 206 (E) 191
(C) 205

4. 5 30 25 5 35 30 6
(A) 48 (D) 36
(B) 1 (E) 25
(C) 54

5. 18 20 10 8 10 5

(A) 3 (D) 10
(B) 7 (E) 15
(C) 0

6. 88 102 99 111 105 115

(A) 123 (D) 124


(B) 106 (E) 103

(C) 104

PRACTICE BOOKLET
BKL PhiLSAT_AA_0317 GO ON TO THE
HE NEXT PAGE
46

7. 54 162 62 186 86

(A) 89 (D) 248


(B) 172 (E) 258
(C) 264

8. 69 72 24 27 9 12

(A) 3 (D) 8
(B) 4 (E) 15
(C) 6

9. 16 8 24 16 48 40

(A) 20 (D) 120


(B) 32 (E) 160
(C) 80

10. 25 15 15 6 12 4 12

(A) 8 (D) 48
(B) 6 (E) 3
(C) 5

11. 10 12 17 20 24 28

(A) 33 (D) 30
(B) 32 (E) 29
(C) 31

12. 3 8 18 23 32 37 45

(A) 48 (D) 49
(B) 50 (E) 51
(C) 52

13. 81 9 64 8 49 7 36

(A) 25 (D) 5
(B) 16 (E) 4
(C) 6

14. 107 102 93 86 79 70 65

(A) 62 (D) 55
(B) 59 (E) 54
(C) 58

PRACTICE BOOKLET
BKL PhiLSAT_AA_0317 GO ON TO THE
HE NEXT PAGE
47

15. 126 138 134 144 136 144 132

(A) 140 (D) 119


(B) 139 (E) 118
(C) 138

Sectio
Section
n 2. Data
Datta
Da a Suffic
Sufficien
iency
cy

Dire
Direct
ctio
ionsns:: The
The datadata suff
suffic
icie
ienc
ncyy prob
proble
lemm cons
consis
ists
ts of a quesquesti
tion
on and
and two statstatem
emen
ents
ts,,
labele
labeled
d (I) and (II),
(II), in whichcerta
whichcertain in data
data aregiven.
aregiven. Decid
Decidee wheth
whetherer the data
data given
given in thestateme
thestatements
nts
are suffic
sufficien
ientt for answeri
answeringng the quest
question
ion.. Using
Using the data
data given
given in the statem
statement
ents,
s, plus
plus knowled
knowledgege
of mathem
mathematicsatics and everyday
everyday facts (such
(such as the
the number
number of of days
days in July or the
the meaning
meaning of thethe word
counterclockwise),
counterclockwise), determine whether:
 Statement (I) ALONE
ALONE is sufficient,
sufficient, but statement
statement (II) alone is not sufficient to answer the
question
question asked.
asked.
 Statement (II) ALONE
ALONE is sufficient,
sufficient, but statement
statement (I) alone is not sufficient to answer the
question
question asked.
asked.
 BOTH
BOTH statement
statementss (I) and
and (II) TOGE
TOGETHE THER R are sufficien
sufficientt to answer
answer the question
question asked,
asked,
  but NEITHER
NEITHER statement
statement ALONE
ALONE is sufficien
sufficientt to answer
answer the question
question asked.
asked.
 EACH
EACH statement
statement ALON
ALONE E is sufficien
sufficientt to answer
answer the question
question asked.
asked.
 Statement
Statementss (I)(I) and
and (II) TOGETHE
TOGETHER R are NOT sufficien
sufficientt to answer
answer the
the question
question asked,
asked, and
additio
additionanall data
data speci
specific
fic to the proble
problem m are neede
needed. d.

16. What
What is the large
largest
st numb
number
er among
among the three
three numbe
numbers
rs A, B, and
and C?

I. A + B = 3 and B –  C = 5
II. 2A + 3C = 11 and
and A, B, and
and C can
can be nega
negativ
tivee or posi
positi
tive
ve numb
number
ers.
s.

(A) Statemen
Statementt (I) ALON
ALONE E is sufficie
sufficient,
nt, but statem
statement
ent (II)
(II) alone
alone is not
not sufficien
sufficient.
t.
(B) Statemen
Statementt (II) ALONE
ALONE is sufficie
sufficient,
nt, but statem
statement
ent (I)
(I) alone
alone is not
not sufficie
sufficient.
nt.
(C)
(C) BOTH
BOTH statem
statemenents
ts TOGETH
TOGETHER ER are suffi
sufficie
cient,
nt, but
but NEITH
NEITHER ER statem
statement
ent ALONE
ALONE
is sufficient.
(D)
(D) EACH
EACH statem
statemenentt ALO
ALONE NE is suff
suffici
icien
ent.
t.
(E) Statem
Statemenents
ts (I) and (II)
(II) TOG
TOGETETHE
HER R are NOTNOT suffi
sufficie
cient.
nt.

17. In how many


many minute
minutess can Raj
Raj wash
wash the car?
car?

I. Raj
Raj and
and Russ
Russel
elll toge
togeth
ther
er can
can wash
wash the
the car
car in 30 minu
minute
tes.
s.
II.
II. Russ
Russel
elll alo
alone
ne can
can fini
finish
sh washi
ashing
ng the
the car
car in
in 1 hour
hour..

(A) Statemen
Statementt (I) ALON
ALONE E is sufficie
sufficient,
nt, but statem
statement
ent (II)
(II) alone
alone is not
not sufficien
sufficient.
t.
(B) Statemen
Statementt (II) ALONE
ALONE is sufficie
sufficient,
nt, but statem
statement
ent (I)
(I) alone
alone is not
not sufficie
sufficient.
nt.
(C) BOTH
BOTH statement
statementss TOGETHER
TOGETHER are sufficien
sufficient,
t, but NEITHER
NEITHER statement
statement ALONE
ALONE
is sufficient.
(D)
(D) EACH
EACH statem
statemenentt ALO
ALONE NE is suff
suffici
icien
ent.
t.
(E) Statem
Statemenents
ts (I) and (II)
(II) TOG
TOGETETHE
HER R are NOT
NOT suffi
sufficie
cient.
nt.

PRACTICE BOOKLET
BKL PhiLSAT_AA_0317 GO ON TO THE
HE NEXT PAGE
48

18. How much does


does a pharmace
pharmaceutica
uticall sales represen
representativ
tativee earn in a year?
year?

I. A pharmace
pharmaceutica
uticall sales
sales represen
representativ
tativee averages
averages ₱42,000
₱42,000 a month
month including bonuses.
II. A pharmace
pharmaceutica
uticall sales representati
representative
ve averages
averages ₱40,000
₱40,000 a month
month and
and ₱2,5
₱2,500
00 in
commissions per month.

(A) Statemen
Statementt (I) ALON
ALONE E is sufficie
sufficient,
nt, but statem
statement
ent (II)
(II) alone
alone is not
not sufficien
sufficient.
t.
(B) Statemen
Statementt (II) ALONE
ALONE is sufficie
sufficient,
nt, but statem
statement
ent (I)
(I) alone
alone is not
not sufficie
sufficient.
nt.
(C)
(C) BOTH
BOTH statem
statemenents
ts TOGETH
TOGETHER ER are suffi
sufficie
cient,
nt, but
but NEITH
NEITHER ER statem
statement
ent ALONE
ALONE
is sufficient.
(D)
(D) EACH
EACH statem
statemenentt ALO
ALONE NE is suff
suffici
icien
ent.
t.
(E) Statem
Statemenents
ts (I) and (II)
(II) TOG
TOGETETHE
HER R are NOTNOT suffi
sufficie
cient.
nt.

19. Blooms
Blooms & Petals
Petals Flower
Flower Shop put on sale bouquet
bouquetss of flowers
flowers on Valentine
Valentine’s
’s Day.
Each
Each bouq
bouquet
uet was sold
sold at ₱450
₱450.. How
How much
much profit
profit did
did the store
store make
make from
from selling
100 bouquet
bouquets?
s?

I. The estimated
estimated cost of one
one bouquet
bouquet is ₱200
₱200..
II. The store
store charged
charged an additional
additional ₱50 for delivery
delivery orders.
orders.

(A) Statemen
Statementt (I) ALON
ALONE E is sufficie
sufficient,
nt, but statem
statement
ent (II)
(II) alone
alone is not
not sufficien
sufficient.
t.
(B) Statemen
Statementt (II) ALONE
ALONE is sufficie
sufficient,
nt, but statem
statement
ent (I)
(I) alone
alone is not
not sufficie
sufficient.
nt.
(C)
(C) BOTH
BOTH statem
statemenents
ts TOGETH
TOGETHER ER are suffi
sufficie
cient,
nt, but
but NEITH
NEITHER ER statem
statement
ent ALONE
ALONE
is sufficient.
(D)
(D) EACH
EACH statem
statemenentt ALO
ALONE NE is suff
suffici
icien
ent.
t.
(E) Statem
Statemenents
ts (I) and (II)
(II) TOG
TOGETETHE
HER R are NOTNOT suffi
sufficie
cient.
nt.

20. Which
Which of the two fast-food
fast-food chains
chains in the universit
university
y belt is more popular
popular among
among
the students
students of Universit
University
y M? The total populati
population
on of studen
students
ts in the universit
university
y is
approximately
approximately 45,000.
1
I. The
The prop
propor
orti
tion
on of stu
stude
dent
ntss that
that frequ
frequen
ents
ts Fas
Fastt Food
Food A is   of the
the total
5
student population.
II.
II. An aver
averag
agee of 6,50
6,500
0 stud
studen
ents
ts goes
goes to Fast
Fast Food
Food B per
per day
day..

(A) Statemen
Statementt (I) ALON
ALONE E is sufficie
sufficient,
nt, but statem
statement
ent (II)
(II) alone
alone is not
not sufficien
sufficient.
t.
(B) Statemen
Statementt (II) ALONE
ALONE is sufficie
sufficient,
nt, but statem
statement
ent (I)
(I) alone
alone is not
not sufficie
sufficient.
nt.
(C)
(C) BOTH
BOTH statem
statemenents
ts TOGETH
TOGETHER ER are suffi
sufficie
cient,
nt, but
but NEITH
NEITHER ER statem
statement
ent ALONE
ALONE
is sufficient.
(D)
(D) EACH
EACH statem
statemenentt ALO
ALONE NE is suff
suffici
icien
ent.
t.
(E) Statem
Statemenents
ts (I) and (II)
(II) TOG
TOGETETHE
HER R are NOTNOT suffi
sufficie
cient.
nt.

PRACTICE BOOKLET
BKL PhiLSAT_AA_0317 GO ON TO THE
HE NEXT PAGE
49

21. How
How much
much tax will
will Man
Mang
g Tomas’
Tomas’ mini
mini grocery
grocery store must
must pay
pay at the
the end of the fiscal
fiscal year 
year 
if the tax rate
rate is 32%?
32%?

I. His taxable
taxable income
income amounted
amounted to ₱75,000
₱75,000..
II. His total sales
sales amounte
amounted
d to ₱250,00
₱250,000
0 and the cost of goods
goods sold is 80%
80% of his sales.

(A) Statemen
Statementt (I) ALON
ALONE E is sufficie
sufficient,
nt, but statem
statement
ent (II)
(II) alone
alone is not
not sufficien
sufficient.
t.
(B) Statemen
Statementt (II) ALONE
ALONE is sufficie
sufficient,
nt, but statem
statement
ent (I)
(I) alone
alone is not
not sufficie
sufficient.
nt.
(C)
(C) BOTH
BOTH statem
statemenents
ts TOGETH
TOGETHER ER are suffi
sufficie
cient,
nt, but
but NEITH
NEITHER ER statem
statement
ent ALONE
ALONE
is sufficient.
(D)
(D) EACH
EACH statem
statemenentt ALO
ALONE NE is suff
suffici
icien
ent.
t.
(E) Statem
Statemenents
ts (I) and (II)
(II) TOG
TOGETETHE
HER R are NOTNOT suffi
sufficie
cient.
nt.

22. How
How old is Rosa
Rosa today
today??

I. Rosa
Rosa is 5 year
yearss youn
younge
gerr than
than her
her brot
brothe
herr Jose
Joseph
ph..
II.
II. In thre
threee year
years,
s, Jose
Joseph
ph will
ill be
be twic
twicee as old
old as Rosa
Rosa..

(A) Statemen
Statementt (I) ALON
ALONE E is sufficie
sufficient,
nt, but statem
statement
ent (II)
(II) alone
alone is not
not sufficien
sufficient.
t.
(B) Statemen
Statementt (II) ALONE
ALONE is sufficie
sufficient,
nt, but statem
statement
ent (I)
(I) alone
alone is not
not sufficie
sufficient.
nt.
(C)
(C) BOTH
BOTH statem
statemenents
ts TOGETH
TOGETHER ER are suffi
sufficie
cient,
nt, but
but NEITH
NEITHER ER statem
statement
ent ALONE
ALONE
is sufficient.
(D)
(D) EACH
EACH statem
statemenentt ALO
ALONE NE is suff
suffici
icien
ent.
t.
(E) Statem
Statemenents
ts (I) and (II)
(II) TOG
TOGETETHE
HER R are NOTNOT suffi
sufficie
cient.
nt.

23. How
How much
much simple
simple intere
interest
st is earn
earned
ed at the
the end of one
one year?
year?

I. Lucy deposited ₱35,000


₱35,000 in a bank
bank that gives
gives 0.1%
0.1% interest
interest per year.
year.
II. Lucy
Lucy invested
invested her money
money amountin
amounting g to ₱35,000
₱35,000 in a financial
financial market
market that offers
offers
3% interest
interest rate
rate per annum
annum..

(A) Statemen
Statementt (I) ALON
ALONE E is sufficie
sufficient,
nt, but statem
statement
ent (II)
(II) alone
alone is not
not sufficien
sufficient.
t.
(B) Statemen
Statementt (II) ALONE
ALONE is sufficie
sufficient,
nt, but statem
statement
ent (I)
(I) alone
alone is not
not sufficie
sufficient.
nt.
(C)
(C) BOTH
BOTH statem
statemenents
ts TOGETH
TOGETHER ER are suffi
sufficie
cient,
nt, but
but NEITH
NEITHER ER statem
statement
ent ALONE
ALONE
is sufficient.
(D)
(D) EACH
EACH statem
statemenentt ALO
ALONE NE is suff
suffici
icien
ent.
t.
(E) Statem
Statemenents
ts (I) and (II)
(II) TOG
TOGETETHE
HER R are NOTNOT suffi
sufficie
cient.
nt.

PRACTICE BOOKLET
BKL PhiLSAT_AA_0317 GO ON TO THE
HE NEXT PAGE
50

24. Which
Which of the
the rectangu
rectangular
lar and
and square
square lots has
has a bigger
bigger area?
area?

I. The
The perim
perimete
eterr of
of the
the rectan
rectangu
gular
lar lot is twice
twice the perime
perimeter
ter of the square
square lot.
lot.
II.
II. The
The widt
width
h of the
the rect
rectan
angu
gula
larr lot
lot is equal
equal to the
the leng
length
th of one
one side
side of the
the squa
square
re lot.
lot.

(A) Statemen
Statementt (I) ALON
ALONE E is sufficie
sufficient,
nt, but statem
statement
ent (II)
(II) alone
alone is not
not sufficien
sufficient.
t.
(B) Statemen
Statementt (II) ALONE
ALONE is sufficie
sufficient,
nt, but statem
statement
ent (I)
(I) alone
alone is not
not sufficie
sufficient.
nt.
(C)
(C) BOTH
BOTH statem
statemenents
ts TOGETH
TOGETHER ER are suffi
sufficie
cient,
nt, but
but NEITH
NEITHER ER statem
statement
ent ALONE
ALONE
is sufficient.
(D)
(D) EACH
EACH statem
statemenentt ALO
ALONE NE is suff
suffici
icien
ent.
t.
(E) Statem
Statemenents
ts (I) and (II)
(II) TOG
TOGETETHE
HER R are NOTNOT suffi
sufficie
cient.
nt.

25. How
How much
much money
money does
does Mrs.
Mrs. Gome
Gomezz save
save from
from her sala
salary
ry per mon
month?
th?
2
I. Her mo
monthlysa
thlysala
laryis
ryis just
just of her husband’s
husband’s monthly
monthly salary.
3
II.
II. She spen
spends
ds 50%
50% of her
her sal
salar
ary
y on food
food..

(A) Statemen
Statementt (I) ALON
ALONE E is sufficie
sufficient,
nt, but statem
statement
ent (II)
(II) alone
alone is not
not sufficien
sufficient.
t.
(B) Statemen
Statementt (II) ALONE
ALONE is sufficie
sufficient,
nt, but statem
statement
ent (I)
(I) alone
alone is not
not sufficie
sufficient.
nt.
(C)
(C) BOTH
BOTH statem
statemenents
ts TOGETH
TOGETHER ER are suffi
sufficie
cient,
nt, but
but NEITH
NEITHER ER statem
statement
ent ALONE
ALONE
is sufficient.
(D)
(D) EACH
EACH statem
statemenentt ALO
ALONE NE is suff
suffici
icien
ent.
t.
(E) Statem
Statemenents
ts (I) and (II)
(II) TOG
TOGETETHE
HER R are NOTNOT suffi
sufficie
cient.
nt.

26. A publis
publishe
herr rece
recentl
ntly
y releas
released
ed a new
new scienc
sciencee fictio
fiction
n nove
novell in book
booksto
stores
res.. How
How much
much is
the sellin
selling
g price
price of one
one book
book??

I.
II. A
Thmarku
mar
The kupp of
e materi
mat als125%
erials12
an5%
and isnting
adde
added
d printi
pri ng dcost
coto
stthe srcina
srcinal
of one l cost
book
boo ckost.
. ounted
amoun
am ted to ₱250
₱250..

(A) Statemen
Statementt (I) ALON
ALONE E is sufficie
sufficient,
nt, but statem
statement
ent (II)
(II) alone
alone is not
not sufficien
sufficient.
t.
(B) Statemen
Statementt (II) ALONE
ALONE is sufficie
sufficient,
nt, but statem
statement
ent (I)
(I) alone
alone is not
not sufficie
sufficient.
nt.
(C)
(C) BOTH
BOTH statem
statemenents
ts TOGETH
TOGETHER ER are suffi
sufficie
cient,
nt, but
but NEITH
NEITHER ER statem
statement
ent ALONE
ALONE
is sufficient.
(D)
(D) EACH
EACH statem
statemenentt ALO
ALONE NE is suff
suffici
icien
ent.
t.
(E) Statem
Statemenents
ts (I) and (II)
(II) TOG
TOGETETHE
HER R are NOTNOT suffi
sufficie
cient.
nt.

PRACTICE BOOKLET
BKL PhiLSAT_AA_0317 GO ON TO THE
HE NEXT PAGE
51

27. Susan
Susan won
won ₱300
₱300,00
,0000 from
from the lotter
lottery.
y. She
She plans
plans to inves
investt a part
part of her winnin
winningsgs and
the rest
rest she will
will set aside
aside as emerg
emergen
ency
cy fund.
fund. How
How much
much did she set set aside for emerg
emergen
encie
cies?
s?

I. Sixty
Sixty perce
percent
nt of her
her winn
winning
ingss was
was invest
invested
ed to a finan
financia
ciall marke
markett that
that offers
offers
7% annual
annual interest
interest rate compoun
compounded
ded annually
annually for 5 years.
years.
II. She
She used
used 50% of her winnin
winnings
gs to start
start a buy-a
buy-and-
nd-sel
selll busin
business
ess..

(A) Statemen
Statementt (I) ALON
ALONE E is sufficie
sufficient,
nt, but statem
statement
ent (II)
(II) alone
alone is not
not sufficien
sufficient.
t.
(B) Statemen
Statementt (II) ALONE
ALONE is sufficie
sufficient,
nt, but statem
statement
ent (I)
(I) alone
alone is not
not sufficie
sufficient.
nt.
(C)
(C) BOTH
BOTH statem
statemenents
ts TOGETH
TOGETHER ER are suffi
sufficie
cient,
nt, but
but NEITH
NEITHER ER statem
statement
ent ALONE
ALONE
is sufficient.
(D)
(D) EACH
EACH statem
statemenentt ALO
ALONE NE is suff
suffici
icien
ent.
t.
(E) Statem
Statemenents
ts (I) and (II)
(II) TOG
TOGETETHE
HER R are NOTNOT suffi
sufficie
cient.
nt.

28.
28. A gradu
graduate
ate was
was calcul
calculati
ating
ng the average
average increa
increase
se in his daily
daily allow
allowanc
ancee durin
during
g his
college
college years.
years.

I. His
His daily allow
allowan
ance
ce durin
during
g his four
four years
years in colle
college
ge were
were ₱150
₱150,, ₱200
₱200,, ₱300
₱300 and
₱350, respectively.
II. His
His paren
parents
ts then
then increa
increased
sed his allow
allowan
ance
ce based
based on his Gene
General
ral Weigh
Weighted
ted Avera
Average
ge
every
every semester.
semester.

(A) Statemen
Statementt (I) ALON
ALONE E is sufficie
sufficient,
nt, but statem
statement
ent (II)
(II) alone
alone is not
not sufficien
sufficient.
t.
(B) Statemen
Statementt (II) ALONE
ALONE is sufficie
sufficient,
nt, but statem
statement
ent (I)
(I) alone
alone is not
not sufficie
sufficient.
nt.
(C)
(C) BOTH
BOTH statem
statemenents
ts TOGETH
TOGETHER ER are suffi
sufficie
cient,
nt, but
but NEITH
NEITHER ER statem
statement
ent ALONE
ALONE
is sufficient.
(D)
(D) EACH
EACH statem
statemenentt ALO
ALONE NE is suff
suffici
icien
ent.
t.
(E) Statem
Statemenents
ts (I) and (II)
(II) TOG
TOGETETHE
HER R are NOTNOT suffi
sufficie
cient.
nt.

29. Benjie’s
Benjie’s Bed and Breakfas
Breakfastt located
located in Baguio City is offering
offering fully furnishe
furnished
d transien
transientt
rooms
rooms at a discounte
discountedd rate of 20%
20% off its
its srcinal
srcinal rent per day
day during
during off-peak
off-peak season.
season.
How
How much
much will
will a tour
touris
istt have
have to pay for
for a day
day for
for two room
rooms?
s?

I. The
The rent
rent for
for thre
threee room
roomss with
with one
one extr
extraa bed
bed is ₱2,500.
II.
II. Durin
uring
g peak
peak seas
season
on,, one
one room
room is rent
rented
ed at ₱850
₱850 per day.
day.

(A) Statemen
Statementt (I) ALON
ALONE E is sufficie
sufficient,
nt, but statem
statement
ent (II)
(II) alone
alone is not
not sufficien
sufficient.
t.
(B) Statemen
Statementt (II) ALONE
ALONE is sufficie
sufficient,
nt, but statem
statement
ent (I)
(I) alone
alone is not
not sufficie
sufficient.
nt.
(C)
(C) BOTH
BOTH statem
statemenents
ts TOGETH
TOGETHER ER are suffi
sufficie
cient,
nt, but
but NEITH
NEITHER ER statem
statement
ent ALONE
ALONE
is sufficient.
(D)
(D) EACH
EACH statem
statemenentt ALO
ALONE NE is suff
suffici
icien
ent.
t.
(E) Statem
Statemenents
ts (I) and (II)
(II) TOG
TOGETETHE
HER R are NOTNOT suffi
sufficie
cient.
nt.

PRACTICE BOOKLET
BKL PhiLSAT_AA_0317 GO ON TO THE
HE NEXT PAGE
52

30. A senior
senior high school
school is offering
offering four strand
strandss in the academic
academic track:
track: General
General Academi
Academicc
Strand (GAS); Humanities and Social Sciences strand (HUMSS);(HUMSS); Science,
Science, Technology,
Technology,
Enginee
Engineering
ring,, and Mathemati
Mathematics cs strand
strand (STEM);
(STEM); and Accoun
Accountancy
tancy,, Business
Business,, and
Mana
Managem
gemenentt strand
strand (ABM)
(ABM).. How
How many
many enroll
enrolled
ed in the STEM
STEM strand
strand out
out of 5,400
5,400 enrol
enrollee
leess
in Grad
Gradee 11?
11?

I. Thir
Thirty
ty-t
-two
wo perc
percen
entt of the
the enro
enrollllee
eess chos
chosee GAS,
GAS, 25% chos chosee ABM,
ABM, and
and
11% chose
chose HUMSS.
HUMSS.
II.
II. The
The rati
ratio
o of STEM
STEM enro
enroll
llee
eess to ABM
ABM enro
enroll
llee
eess is 5 : 3.

(A) Statemen
Statementt (I) ALON
ALONE E is sufficie
sufficient,
nt, but statem
statement
ent (II)
(II) alone
alone is not
not sufficien
sufficient.
t.
(B) Statemen
Statementt (II) ALONE
ALONE is sufficie
sufficient,
nt, but statem
statement
ent (I)
(I) alone
alone is not
not sufficie
sufficient.
nt.
(C)
(C) BOTH
BOTH statem
statemenents
ts TOGETH
TOGETHER ER are suffi
sufficie
cient,
nt, but
but NEITH
NEITHER ER statem
statement
ent ALONE
ALONE
is sufficient.
(D)
(D) EACH
EACH statem
statemenentt ALO
ALONE NE is suff
suffici
icien
ent.
t.
(E) Statem
Statemenents
ts (I) and (II)
(II) TOG
TOGETETHE
HER R are NOTNOT suffi
sufficie
cient.
nt.

PRACTICE BOOKLET
BKL PhiLSAT_AA_0317 GO ON
ON TO THE
THE NEXT
NEXT PAGE
PAGE
53

Section
Section 3. Data Interpret
Interpretatio
pretation
ation
n

Direct
Direction
ions:
s: In this
this sectio
section,
n, some
some sets
sets of data
data are given
given.. Inter
Interpr
pret
et or analyz
analyzee the given
given data
data to
answer
answer the question
questionss that refer to the data presente
presented.
d. Select
Select the correct
correct answer
answer from the options
options
given.

For
For item
itemss 31
31 and 32,
32, refer
refer to the graph
graph below
below..

DISTRIBU
DISTRIBUTIO
TION
N OF SHARES
SHARES OF STOCKS
STOCKS OF DIFFER
DIFFERENT
ENT CORPOR
CORPORATIO
ATIONS
NS

40,000

s
e
r 30,000
a
h
S
f
o 20,000
r
e
b
m
u
N10,000

0
ABCDE
Corporation

31. The
The numbe
numberr of share
sharess owne
owned
d by A exceed
exceedss that
that of D by how many
many shares
shares??

(A) 10,000
(B) 20,000
(C) 25,000
(D) 37,500

32. What
What percent
percent of
of the number
number of
of shares
shares owne
owned
d by E is the number
number of
of shares
shares owne
owned
d by B?

(A) 42%
(B) 30%
(C) 25%
(D) 18%

PRACTICE BOOKLET
BKL PhiLSAT_AA_0317 GO ON TO THE
HE NEXT PAGE
54

For
For items
items 33
33 to 35, refer
refer to the chart
chart belo
below.
w.

PERCEN
PERCENTAG
TAGE
E DISTRIBU
DISTRIBUTIO
TIONN OF STUDEN
STUDENTS
TS ENROLL
ENROLLEDED IN THE DIFFER
DIFFERENT
ENT
COUR
COURSE
SES
S IN A CERTA
CERTAIN
IN UNIVE
UNIVERS
RSITY
ITY (N = 28,00
28,000)
0)

Dentistry
20%

  Nursing
8% Management
55%
Political Science
4%

Engineering
13%

33. Which
Which course(s
course(s)) comprise(
comprise(s)
s) one
one fifth of the total enrol
enrollmen
lmentt in the universit
university?
y?

(A) Denti
entist
stry
ry
(B)
(B) Engi
Engine
neer
erin
ing
g
(C)
(C) Nurs
Nursin
ing
g and
and Engi
Engine
neer
erin
ing
g
(D)
(D) Nursi
Nursing
ng,, Engine
Engineeri
ering
ng,, and Politi
Political
cal Scien
Science
ce

34. The differenc


differencee between
between the enrollmen
enrollments
ts in Managem
Management
ent and in Engineer
Engineering
ing is

(A) 14,280
(B) 13,160
(C) 11,760
(D) 9,800

35. What
What is the ratio
ratio of studen
students
ts enrol
enrolled
led in Mana
Managem
gement
ent to those
those who
who are enrol
enrolled
led in
Dentistry?

(A) 4 : 11
(B) 11 : 4
(C) 20 : 55
55
(D) 55 : 20

PRACTICE BOOKLET
BKL PhiLSAT_AA_0317 GO ON TO THE
HE NEXT PAGE
55

For
For item
itemss 36
36 to 38, refer
refer to the table
table below
below..

DISTRIB
DISTRIBUTI
UTION
ON OF DEGREES
DEGREES GRANTED
GRANTED
IN PHILIPP
PHILIPPINE
INE COLLEG
COLLEGES
ES AND UNIVER
UNIVERSITI
SITIES
ES

Degree N %
Education 33,911,000 39.24
Business 34
34,193,000 39.57
Law 1,105,000 1.28
Architecture 279,000 0.32

Engineering 4,462,000 5.16


Social Science 5,086,000 5.88
  Na
Natural Science 2,364,000 2.74
Medical Science 5,019,000 5.81
Medicine (36.5)
Dentistry (7.7)
  Nursing (41.8)
Optometry (3.8)
Pharmacy (10.2)

36. Approxim
Approximately
ately,, what
what part of the populatio
population
n of graduate
graduatess came from all other
other courses
courses
besides
besides Educatio
Education n and Business
Business??

(A) One third


ird
(B) One fourth
(C)
(C) Less
Less than
than one
one four
fourth
th
(D) Morethantw
rethantwo o thir
third
ds

37. How many percent


percent of the
the total populatio
population
n finished
finished Medicine
Medicine??

(A) 2.12%
(B) 6.29%
(C) 21.22%
(D) 36.54%

38. Which
Which of the following
following statemen
statements
ts does
does NOT correctly
correctly describe
describe the distributi
distribution
on
of degrees?

(A)
(A) The
The numb
numberer of degre
degrees
es in Medic
Medical
al Scien
Science
ce differ
differed
ed from
from that
that in Socia
Sociall Science
Science
  by 67,000.
67,000.
(B)
(B) The
The larges
largestt number
number of
of degree
degreess was gran
granted
ted in Busin
Business
ess..
(C)
(C) Ther
Theree were
were more
more degr
degree
eess giv
given
en in Nurs
Nursin
ing
g than
than in Educ
Educat atio
ion.
n.
(D)
(D) There
There were
were 386,4
386,463
63 degre
degreeses grante
granted
d in Denti
Dentistr
stry.
y.

PRACTICE BOOKLET
BKL PhiLSAT_AA_0317 GO ON TO THE
HE NEXT PAGE
56

For
For items
items 39
39 to 41, refer
refer to the chart
chart belo
below.
w.

PERCEN
PERCENT
T DISTRIB
DISTRIBUTIO
UTION
N OF OVERSE
OVERSEAS
AS FILIPIN
FILIPINOO WORKER
WORKERS
S (OFWs)
(OFWs)
BY PLAC
PLACEE OF WORK
WORK:: 2012
2012

SaudiArabia (20.6%)
(20.6%)

UnitedArab Emirates
Emirates
Europe (8.7%)
(14.9%)

  North and South America


America Singapore (7.5%)
Asia (79.7%)
(7.6%) Qatar (6.5%)
(6.5%)
Australia
Australia (2.2%)
(2.2%) Hongkong (5.5%)
Kuwait
Kuwait (4.2%)
(4.2%)
Africa
Africa (1.7%)
(1.7%)
Japan (3.4%)
(3.4%)

Other countries (17.1%)

Source: PhilippineS tatistics Authority, 2012 Survey on OverseasFilipinos


URL: https://psa.gov.ph/content/201
https://psa.gov.ph/content/2012-survey
2-survey-overseas-filipinos
-overseas-filipinos

39. Based
Based on the chart
chart,, which
which of the follow
following
ing gener
generali
alizat
zation
ionss are true?
true?

I. There
There are more
more OFWs
OFWs in Asian
Asian coun
countrie
triess than
than in other
other places
places combin
combined
ed..
II.
II. Ther
Theree are
are more
more OFWs
OFWs in Japa
Japan
n than
than in Aust
Austraralia
lia and
and Afri
Africa
ca comb
combin
ined
ed..
III.
III. Abou
Aboutt one
one fift
fifth
h of the
the OFWs
OFWs in Asia
Asia are
are in Saud
Saudii Ara
Arabi
bia.
a.
IV. Abou
Aboutt one
one fourth
fourth of the OFWs
OFWs are in North
North and South
South Ameri
America
ca and
and Europe
Europe..

(A) I and
and II only
(B) I and III only
(C) II and IV onllyy
(D) I, III, and IV only

40. Which
Which place(s)
place(s) of
of work
work comprise(
comprise(s)
s) approx
approximate
imately
ly one tenth of the
the total OFWs in 2012?
2012?

(A)
(A) Nort
North
h and
and Sout
South
h Amer
Americicaa and
and Aust
Austra
rali
liaa
(B)
(B) Nort
North
h and
and Sout
South
h Amer
Americicaa
(C)
(C) Euro
Europe
pe and
and Aust
Austra
rali
liaa
(D) Europe

41. What
What will
will happe
happen n to the ranking
ranking of coun
countries
tries according
according to the percentag
percentagee of OFWs
OFWs if the
the
frequen
frequency
cy distributi
distribution
on is reported
reported instead
instead of percent distribution
distribution??

(A)
(A) The
The rankin
ranking g canno
cannott be determ
determineined
d from
from the frequ
frequenc
ency
y distrib
distributi
ution
on..
(B)
(B) The
The rank
rankin
ingg will
will be reve
revers
rsed
ed..
(C)
(C) The
The rank
rankin
ingg will
will be the
the same
same..
(D)
(D) Ther
Theree will
ill be
be tied
tied rank
ranks.
s.

PRACTICE BOOKLET
BKL PhiLSAT_AA_0317 GO ON TO THE
HE NEXT PAGE
57

For
For items
items 42 to 46, refer
refer to the follow
following
ing situat
situation
ion..

In 2015
2015,, Toyo
Toyotata Motor
Motor Philip
Philippin
pines
es was
was able
able to sell
sell a total
total of
of 124,4
124,426
26 units
units.. This
This show
showss a
growth
growth of 17.8%
17.8% from
from its 2014
2014 sales
sales perfor
performan
mance.
ce. Below
Below is a table
table showin
showing
g the top 5 model
modelss
that have
have the largest
largest numbe
numberr of units sold during
during the year.
year.

Model Vios 1.3 Hiace Innova Di


Diesel Fortuner 4×2 Avanza
  No.
No. of Units
Units
30,754 16,675 16,079 15,148 8 ,646
Sold
Source: http://www.topgear.com.ph/n
http://www.topgear.com.ph/news
ews

42. How
How many
many units
units were
were sold
sold in 2014
2014??
(A) 71,762
(B) 87,302
(C) 102,278
(D) 105,62
,625

43. Supp
Suppose
ose that,
that, in the follow
following
ing year,
year, the numb
number
er of units
units sold
sold for the top 5 models
models
doubled
doubled except
except that for
for Vios
Vios 1.3. Which
Which model(s)
model(s) would
would have more number
number of units
sold
sold than
than Vios
Vios 1.3?

(A) Hiace only


(B)
(B) Inno
Innova
va Dies
Diesel
el and Hiac
Hiacee only
only
(C) Fortun
rtuneer 4×2, Inno
Innova
va Dies
Diesel
el,, and
and Hiac
Hiacee only
only
(D)
(D) All
All fou
fourr mod
model
elss

44. Which
Which of the follow
following
ing can be conclu
conclude
ded
d based
based on the table?
table?
(A)
(A) More
More peop
people
le buy
buy Vios
Vios 1.3
1.3 than
than any
any othe
otherr Toy
Toyot
otaa mode
model.l.
(B)
(B) Avan
Avanzaza is the
the most
most expe
expens
nsivivee Toyo
Toyota
ta mode
model.
l.
(C)
(C) Peop
People
le do not
not buy
buy a vehi
vehicl
clee base
basedd on the
the type
type of fuel
fuel it uses
uses..
(D)
(D) All of thes
thesee

45. If a 5% growt
growth
h sale
sale of Inno
Innova
va Diesel
Diesel mod
model
el is forec
forecast
asted
ed in 2018
2018,, how
how many
many units
units of
of this
model
model will be sold?
sold?

(A)
(A) Exac
Exactl
tly
y 16,8
16,800
00
(B) More tha
than 16,88
,880
(C)
(C) Less
Less than
than 16,8
16,880
80
(D)
(D) Exac
Exactl
tly
y 16,8
16,880
80

46. Which
Which of the following
following statemen
statements
ts abou
aboutt the
the units
units sold
sold in 2015 is FALSE?
FALSE?

(A)
(A) Abou
Aboutt one
one fourth
fourth of the total
total unit
unitss sold
sold are Vios
Vios 1.3.
1.3.
(B)
(B) Seve
Sevent
nty
y perc
percen
entt of the
the tota
totall units
units sol
sold
d belo
belong
ng to the
the top
top 5 mode
modelsls..
(C)
(C) Ther
Theree were
were more
more Vios
Vios 1.3
1.3 sold
sold than
than Fort
Fortun
uner
er 4×2 and Avan
Avanzaza combi
combine
ned.
d.
(D)
(D) Each
Each of the top 5 models
models is at least
least 10%
10% of the total
total units sold.
sold.

PRACTICE BOOKLET
BKL PhiLSAT_AA_0317 GO ON TO THE
HE NEXT PAGE
58

For
For item
itemss 47
47 to 50, refer
refer to the graph
graph below
below..

PERCEN
PERCENTAG
TAGE
E OF SMARTP
SMARTPHON
HONE E USAGE
USAGE IN 2014
2014
ACRO
ACROSS
SS AGE
AGE GROU
GROUPS
PS IN THE
THE U.S.
U.S.

18-29 30-49 50+

100
Text messaging 98
92

97
Internet use 90
80

Voice/video 93
91
calls 94

91
E-mail 87
87

Source: Pew Research Center


Center American Trends Panel, 2014.
URL: http://www.pewinternet.org/2
http://www.pewinternet.org/2015/04/0
015/04/01/chapter-three-a-wee
1/chapter-three-a-week-in-the-life-analysis-of-
k-in-the-life-analysis-of-
smartphone-users

47. The Pareto


Pareto chart
chart shown
shown below
below is constru
constructe
cted
d for the 30 –  49 age
age grou
group
p base
based
d on
the bar graph.
graph. Which
Which phone
phone feature
feature is misrepres
misrepresente
ented?
d?

100
90
sr
e 80
n
w 70
o
e
n 60
o
h
tpr
50
a 40
sm
f 30
         o
% 20
10
0
T ex
ex tm es
es sa
sagi ng
ng I nt
nte rn
rn et
et us
use V oi
oi ce
ce/ Vi
Vid eo
eo E-mail
calls
Phonefeatures

(A) Text messaging (C) Voice/Video ca


c alls
(B) E-mail
E-mail (D)
(D) Intern
Internet
et use
use

PRACTICE BOOKLET
BKL PhiLSAT_AA_0317 GO ON TO THE
HE NEXT PAGE
59

48.
48. Which
Which of the follow
following
ing statem
statemen
ents
ts abo
about
ut the patter
pattern
n of smartp
smartpho
hone
ne usage
usage among
among
Americans is true?

(A) The youn


younges
gestt age
age group
group was
was high
highly
ly depen
dependen
dentt on a smartp
smartpho
hone
ne for intern
internet
et use
use
compared
compared to text messagin
messaging.g.
(B) The rates
rates of voice/
voice/vid
video
eo callin
calling
g for the
the oldes
oldestt and young
youngest
est smart
smartpho
phonene owner
ownerss were
nearly
nearly identical.
identical.
(C) Using
Using e-mail
e-mail was
was the
the most
most vari
variabl
ablee behav
behavior
ior among
among age
age group
groups.
s.
(D) Text
Text messag
messaging
ing was
was equal
equally
ly popu
popular
lar amon
amongg age
age group
groups.
s.

49.
49. Which
Which smart
smartph
phon
onee featur
featuree showe
showed
d the highe
highest
st varia
variabil
bility
ity in usage
usage among
among owne
owners?
rs?

(A)
(A) Voic
Voice/
e/Vi
Vide
deo
o calls
calls
(B)
(B) Text
Text mess
messagagin
ing
g
(C)
(C) Inte
Intern
rnet
et use
use
(D) E-mail

50.
50. Whic
Which
h smar
smartp
tpho
hone
ne feat
featur
uree has
has a simi
simila
larr rate
rate of usage
usage betw
betwee
een
n two
two age
age grou
groups
ps??

(A)
(A) Text
Text mess
messagagin
ing
g
(B)
(B) Inte
Intern
rnet
et use
use
(C)
(C) Voic
Voice/
e/Vi
Vide
deo
o call
callss
(D) E-mail

STOP!
WAIT
WAIT FOR FURTHE
FURTHER
R
INSTRUCTIONS.
INSTRUCTIONS

PRACTICE BOOKLET
BKL PhiLSAT_AA_0317
Philippine
Philippine Law School
School Admission
Admis
Admission
sion Test
Answer Key

Test A. Communicat
Communication
ion & Language
Language Proficiency
Proficiency
It
Ite
te
emm No
No.
o. 1 2 3 4 5 6 7 8 9 10 10 11
11 12
12 1
13
3 14
14 1
15
5 16
16 1
17
7 18
18 19
19 20
20 21
21 22
22 23
23 24
24 25
25
Key 32 41 24 43 242 32 31 243 31 13 44 3

Ite
Item
tem No.
No
o.. 26 2 7 28 2 9 30 31 32 33 34 3 5 36 37 38 39 40 41 4 2 43 4 4 45 46 47 4 8 49 5 0
Key 13 44 23 34 343 42 11 232 23 32 12 2

Test
Test B. Critic
Critical
al Thinki
Thinking
ng
It
Ite
te
emm No
No.
o. 1 2 3 4 5 6 7 8 9 10
10 11
11 1
12
2 13
13 14
14 15
15 1
16
6 17
17 18
18 1
19
9 20
20 21
21 22
22 23
23 24
24 25
25
Key 23 31 33 23 23 42 42 414 23 3 214 42

Ite
Item
tem No.
No
o.. 26 2 7 28 2 9 30 31 32 33 34 3 5 36 37 38 39 40 41 4 2 43 4 4 45 46 47 4 8 49 5 0
Key 4 132 41 23 324 423 343 14 34 12 31

Test
Test C. Verbal
Verbal Reason
Reasoning
ing
It
Ite
te
emm No
No.
o. 1 2 3 4 5 6 7 8 9 10
10 11
11 12
12 1
13
3 14
14 1
15
5 16
16 1
17
7 18
18 19
19 20
20 21
21 22
22 23
23 24
24 25
25
Key 32 44 13 11 424 14 14 421 31 34 22 4

Ite
Item
tem No.
No
o.. 26 2 7 28 2 9 30 31 32 33 34 3 5 36 37 38 39 40 41 4 2 43 4 4 45 46 47 4 8 49 5 0
Key 43 43 14 13 411 32 24 241 21 14 12 1

Test D. Quantitat
Quantitative
ive Reasoning
Reasoning
It
Ite
te
emm No
No.
o. 1 2 3 4 5 6 7 8 9 10 10 11
11 1
12
2 13
13 14
14 15
15 1
16
6 17
17 18
18 1
19
9 20
20 21
21 22
22 23
23 24
24 25
25
Key 44 31 12 52 43 32 35 333 41 3 434 45

Ite
Item
tem No.
No
o.. 26 2 7 28 2 9 30 31 32 33 34 3 5 36 37 38 39 40 41 4 2 43 4 4 45 46 47 4 8 49 5 0
Key 34 121 3 21 323 13 213 42 12 44 23 4

PRACTICE BOOKLET
GENERAL DIRECTIONS

The Philip
Philippin
pinee Law Scho
Schoolol Admis
Admissio
sion
n Test
Test consis
consists
ts of four
four subte
subtests
sts.. Each
Each subtes
subtestt
contains multiple-choice items.

For each item, select


select your answer
answer from the
the options
options given.
given. On your
your answ
answerer sheet,
sheet, shade
shade
the circle
circle marke
marked
d with
with the letter
letter of your
your chosen
chosen answ
answer.
er. For
For examp
example,
le, if your
your answ
answerer to

an
sureitem
sureitem
yourismark
your opti
option
markon
onC, then
thcircle
the encle
cir comp
complelete
tely
ly shad
is dark.
dark. sh
Avade
e the
Avoid
oidthe orrec
circ
circle
incorr
inc let mark
ect ma rked
shadied
ng C
shading ofas
cirshow
shown
circle
cles ans belo
s as be
thlow
eywmay
they . Make
Ma ket
not
no
  be recog
recogniz
nized
ed as an answ
answer.
er.

Make
Make sure
sure you
you are markin
markingg the answ
answerer colum
columns
ns corre
correspo
spond
nding
ing to the item
item numbe
numberr you
you
are
are on.
on. Mark
Mark only
only one
one answ
answerer for
for each
each item.
item. If you
you want
want to chan
change
ge your
your answ
answer
er,, eras
erasee
the first answer
answer comp
completely
letely.. Incomple
Incomplete te erasures
erasures will be
be interprete
interpreted
d as another
another answer 
answer 
thereb
therebyy produ
producin
cingg “multip
multiple
le answe
answers.
rs.”” Items
Items with
with multip
multiple
le answ
answers
ers are autom
automati
atical
cally
ly
considered wrong.

Do not
not write
write anythi
anything
ng on this
this test
test book
booklet
let.. Use
Use the blank
blank pages
pages of
of your
your answer
answer shee
sheett for 
your scratch work.

Follo
Follow
w care
carefu
full
lly
y the
the spec
specif
ific
ic dire
directctio
ions
ns for
for each
each subt
subtes
estt or sect
sectio
ion.
n. When
When you
you fini
finish
sh a
subtes
subtest,
t, proce
proceed
ed to the next
next unti
untill you
you have
have compl
complete
eted
d the entire
entire test.
test.

DO NOT TURN THIS PAGE UNTIL YOU ARE TOLD TO DO SO.

PRACTICE BOOKLET
3

TEST
TEST
TEST A. COMMUNICATIO
COMMUNIC
COMMUNICA
COMMUNICATION
ATION
TION
N AND LANGU
LANGUAGE
AGE PROFICIE
PROF
PROFICIENCY
NCY

Section
Sectio
Section
n 1. Identifyin
Ident
Identi
Identifyi
ifying
fying
ng
g Sentence
Sente
Sentence
nce Errors
Errors
Errors

Direct
Direction
ions:
s: Read
Read each
each senten
sentence
ce carefu
carefully
lly but
but quick
quickly,
ly, paying
paying attent
attention
ion to the unde
underlin
rlined
ed word
word or 
  phrase
phrase.. Each
Each senten
sentence
ce conta
contains
ins eith
either
er a single
single error
error or
or no error
error at
at all.
all. If the
the senten
sentence
ce conta
contains
ins an
error,
error, select
select the under
underlin
lined
ed word
word or phras
phrasee that
that must
must be chang
changed
ed to make
make the senten
sentence
ce corre
correct.
ct. If                 
the sentence
sentence is correct,
correct, select
select choice
choice D.

In choosin
choosing
g answers,
answers, follow
follow the requireme
requirements
nts of standard
standard written
written English.
English.

1. Anyb
Anybod
ody
y who
who is ready
ready with
with theanswer
theanswer may
may raise
raise their
their right
right hand
hand.. No error 
error 
A B C D

2. The chair
chairma
man
n of the board
board had spend
spend nearly
nearly an hour listen
listening
ing to the member
memberss of
of the
the press
press
A B C

before
before issuin
issuing
g the order.
order. No error 
error 
D

3. Some
Some youn
young
g peop
people
le want
want succes
successs badly
badly but
but they
they are not
not willing
willing to work
work for it. No error 
error 
A B C D

4. Some universities has


has set up small colleges for
for closer relationships
relationships between
between professors and
and
A B C

stud
studen
ents
ts.. No erro
error r 
D

5. The
The admi
admini
nist
stra
rati
tion
on offi
office
cers
rs have
have expr
expres
esse
sed
d they’r
they’ree gratitu
gratitude
de to the rank and file
A B

for their loyalty


loyalty and support.
support. No error 
C D

6. It is imperativ
imperativee that the matter
matter be kept unknown
unknown from the organize
organizers.
rs. No error 
error 
A B C D

7. Ramire
Ramirez,
z, accept
accepting
ing his defeat
defeat like
like a true
true sport
sport,, cong
congrat
ratula
ulated
ted the newly
newly procl
proclaim
aimed
ed
A B C

champion
champion.. No error 
D

PRACTICE BOOKLET
BKL PhiLSAT_AA_0317 GO ON TO
TO THE NEX
NEXT
NEXT
XT PAGE
PA
PAGE
AG
GE
E
4

8. The const
construc
ructio
tion
n worke
workers
rs were
were tired
tired but
but the aftern
afternoo
oon
n nap rest
rest them.
them. No error 
error 
A B C D

9. Did you
you notic
noticee the way
way she look
look when
when she
she caugh
caughtt sigh
sightt of
of the
the accide
accident
nt victim
victims?
s? No error 
error 
A B C D

10.
10. Smoke
Smoke billow
billowed
ed abov
abovee the roofto
rooftops
ps as fireme
firemen
n battled
battled their
their way
way throu
through
gh the noisy
noisy crowd
crowd..
A B C

  No error 
error 
D

11.
11. Food
Food at the
the cant
cantee
een
n is expe
expens
nsiv
ive,
e, so I use
use to brin
bring
g a pack
packed
ed lunc
lunch.
h. No erro
error r 
A B C D

12.
12. Althou
Although
gh tired
tired and hung
hungry,
ry, the gover
governo
norr reluct
reluctant
antly
ly agreed
agreed to a twenty-
twenty-min
minute
utess press
press
A B C

conferen
conference.
ce. No error 
error 
D

13.
13. The compu
computer
ter,, a leadin
leading
g techn
technolo
ologic
gical
al dev
device
ice,, it make
makess human
human tasks
tasks easier
easier to do.
do.
A B C

  No error 
error 
D

14. At the
Atime
time of the revolu
revolutio
tion,
n, violen
violence
ce durin
during
g the
B vigil
vigil and momen
moments
ts of prayer 
prayer 

minimized
minimized by the leaders
leaders of differe
different
nt religiou
religiouss sectors.
sectors. No error 
error 
C D

15.
15. That
That nine
ninety
ty perce
percent
nt of the body
body consis
consists
ts of fluids
fluids,, parti
particul
cularl
arly
y blood
blood and
and water
water.. No error 
error 
A B C D

16.
16. Mayo
Mayon
n Volc
Volcan
ano
o is an acti
active
ve stra
strato
tovo
volc
lcan
ano
o in the
the Bico
Bicoll Regi
Region
on reno
renown
wned
ed for
for its
its
A

most
most perfect
perfect cone
cone and
and nomin
nominate
ated
d as one of the wond
wonders
ers of the
the world
world.. No error 
error 
B C D

17.
17. If the Filipi
Filipino
noss had
had not
not gon
gonee to the street
streetss to
to demon
demonstr
strate
ate their
their resist
resistanc
ancee agains
againstt
A B

the dictat
dictator,
or, we would
would not
not have
have regain
regained
ed our
our freedo
freedom.
m. No error 
error 
C D

PRACTICE BOOKLET
BKL PhiLSAT_AA_0317 GO ON TO
TO THE NEX
NEXT
NEXT
XT PAGE
PA
PAGE
AG
GE
E
5

18.
18. The Sena
Senate
te has unti
untill Novemb
November
er to cond
conduct
uct inter
intervie
views
ws for
for the confi
confirma
rmatio
tion
n of                 
A

the
the President’s appointees,  but the senators decided to do this after the budget session.
B C

  No error 
error 
D

19. If the
the phon
hone rin
rings and
and I’m not
not aroun
around,
d, please
please   answer
answer it and
and make sure
A

to get the caller’s


caller’s name,
name, his contact
contact number
number and what
what his
his messa
message
ge is. No error 
error 
B C D

20.
20. Most
Most of
of the
the athlet
athletic
ic equip
equipme
ments
nts used
used this
this year
year by the PE Depa
Departm
rtmen
entt are
are newly
newly acquir
acquired
ed..
A B C

  No error 
error 
D

21. Our history


history teacher
teacher believes
believes that twelve
twelve meetings
meetings are insufficie
insufficient
nt for us to master 
master 
A B C

the history
history of Japan
Japan,, so he assigned
assigned additiona
additionall readings
readings and
and projects
projects for suppo
support.
rt.

  NoD
error 
error 

22.
22. Despi
Despite
te the bad weath
weather
er last
last Sund
Sunday,
ay, the family
family fun-ru
fun-run
n spons
sponsore
ored
d by the paren
parents
ts were
were
A B C

so successf
successful
ul that they were able to raise sufficien
sufficientt funds
funds to buy school
school materials
materials for 

the
the Aeta
Aetas.
s. No erro
error r 
D

PRACTICE BOOKLET
BKL PhiLSAT_AA_0317 GO ON TO
TO THE NEX
NEXT
NEXT
XT PAGE
PA
PAGE
AG
GE
E
6

Section
Sectio
Section
n 2. Sentence
Sente
Sent
Senten
ence
nce
ce Completion
Completio
Completion
n

Direct
Direction
ions:
s: Choo
Choose
se the word
word or phras
phrasee that,
that, when
when insert
inserted
ed in the senten
sentence,
ce, best
best fits
fits the meanin
meaning
g
of the
the senten
sentence
ce as a whole
whole..

23. The class condoled


condoled with their teacher
teacher when
when her mother
mother _______
__________
______
___ last week.
week.

(A) passed up
(B) passed off                  

(C)
(D)  p
paassed
ssed aout
way

24.
24. The corne
cornered
red kidna
kidnappe
ppers
rs were
were force
forced
d to ____
______
____
_____
_____
__ their
their white
white flag
flag becau
because
se they
they
feared for their
their lives.
lives.

(A) rise
(B) race
(C) raze
(D) raise

25.
25. ____
______
____
____
____
___
_ his
his close
close rela
relati
tion
on with
with the
the vict
victim
im,, no one
one was
was allow
allowed
ed by the
the poli
police
ce to
enter
enter the scene
scene of the crime
crime..

(A) Regarding
(B)
(B) Irre
Irrega
gard
rdle
lesssly
sly of         

(C)
(D)
(D) Regardless
Irre
Irrega
gardlesssof 
rdle of                          

26.
26. _____
_______
____
____
____
__ the burgl
burglar
ar was,
was, he must
must have
have cased
cased our
our house
house for days
days befo
before
re actua
actually
lly
entering
entering our house
house that night.
night.

(A) Whoever  
(B) Whomever  
(C) Whosoever  
(D) Whomsoever 

27.
27. Mothe
Motherr tells
tells Sarah
Sarah that
that her small
small cat
cat named
named Elsie
Elsie ____
________
______
____
___
_ under
under the
the swing
swing on
the front
front porch.

(A) lays
(B) lying
(C) is lying
(D) is la
laying

PRACTICE BOOKLET
BKL PhiLSAT_AA_0317 GO ON TO
TO THE NEX
NEXT
NEXT
XT PAGE
PA
PAGE
AG
GE
E
7

28.
28. I _____
_______
____
____
____
__ the assign
assigned
ed task
task ahead
ahead of schedu
schedule.
le.

(A) done
(B) am do
d one
(C) was done
(D) have done

29.
29. The Presid
President
ent,, who
who is known
known to be a chain
chain-sm
-smok
oker,
er, might
might just
just ____
_______
_____
____
____
__ stop
stop
smokin
smokingg in his own
own time
time witho
without
ut being
being pressu
pressured
red by anyon
anyone.
e.

(A) quit

(B)
(C) quite
quitely
(D) quietly

30.
30. The employ
employees
ees are eager
eagerly
ly await
awaiting
ing for the date
date of ___
______
_____
____
____
___
_ of thei
theirr new
new
salary
salary scheme.
scheme.

(A) affectiv
tivity
(B) effectiv
tivity
(C)
(C) effe
effecctiv
tiveness
ess
(D)
(D) effe
effect
ctiv
ivel
elin
ines
esss

31.
31. Many
Many electi
election
on losers
losers susp
suspect
ect that
that some
some winne
winners
rs _____
_______
____
____
____
__ their
their way to
the victor
victory
y line.
line.

(A) cheat

(B)
(C) will
havecheat
cheated
(D) has cheated

32.
32. The invest
investors
ors ______
________
____
____
___
_ consid
considera
erable
ble capita
capitall when
when the tsuna
tsunami
mi devas
devastat
tated
ed
the province.
province.

(A) lose
(B) loss
(C) lost
(D) loose

33.
33. The rebels
rebels believ
believee that
that they
they are safe
safe in the mounta
mountains
ins where
where _____
_______
____
____
____
__ famili
familiar 
ar 
with
with the terrai
terrain.
n.

(A) there
(B) their  
(C) there are
(D) they a re
re

PRACTICE BOOKLET
BKL PhiLSAT_AA_0317 GO ON TO
TO THE NEX
NEXT
NEXT
XT PAGE
PA
PAGE
AG
GE
E
8

34. The propertie


propertiess of Alex’s rich unmarrie
unmarried
d aunt were
were divided
divided _______
__________
______ 
___  all
the nephews
nephews and nieces
nieces in accordan
accordance
ce with her last
last will
will and
and testament
testament..

(A) to
(B) with
(C) among
(D) between

35. Mother
Mother told us to _______
__________
______
___ our little
little brother’s
brother’s tantru
tantrums
ms since
since he’s a special
special child
child
and we all love
love him.
him.

(A)
(B)  p
puutt o
on
ff                  
(C) put acacross
(D) put up up with

36.
36. The
The snat
snatch
cher
er surp
surpri
rise
sed
d me from
from behi
behind
nd,, grab
grabbe
bed
d my hand
hand,, and
and ____
______
____
____
____
___
_ with
with
my handbag.
handbag.

(A) made do
(B) made out
(C) made off                  
(D) made away

37.
37. By Decemb
December
er next
next year,
year, Mr.
Mr. and Mrs.
Mrs. Ocamp
Ocampo
o ____
______
____
_____
_____
__ for forty
forty years
years..

(A) were married

(B)
(C) had
h avebeen
bee
ben
enmarrie
mar
marieddd
rried
ie
(D)
(D) will
ill ha
have beenmar
enmarrieried

38.
38. The Moun
Mountt Pinatu
Pinatubo
bo volca
volcanic
nic erupti
eruption
on is consid
considere
ered
d the ____
______
_____
_____
____
__ in
Philippine history.

(A) worse
(B) worst
(C) worsest
(D) most wo
worse

39.
39. _____
_______
____
____
____
__ Musli
Muslimm laws
laws are
are based
based on Islam,
Islam, specia
speciall shari
shariah
ah court
courtss have
have been
been
created
created to decide
decide on family
family and personal
personal conflicts
conflicts of Muslim
Muslimss in autonom
autonomousous regions.
regions.

(A) Since
(B) Although
(C) When
(D) Even though

PRACTICE BOOKLET
BKL PhiLSAT_AA_0317 GO ON TO
TO THE NEX
NEXT
NEXT
XT PAGE
PA
PAGE
AG
GE
E
9

Sectio
Section
n 3. Improv
Impro
Improving
rovin
ving
ing
g Senten
Sentence
tences
ces
s

Direct
Direction
ions:
s: Read
Read each
each senten
sentence
ce carefu
carefully
lly but
but quick
quickly
ly and
and determ
determine
ine wheth
whether
er the unde
underlin
rlined
ed
  port
portion
ion is cor
correc
rectt or whet
whether
her it need
needss to be revise
revised.
d. If the senten
sentence
ce needs
needs no
no revisi
revision
on,, choo
choose
se
optio
option
n D.

In choo
choosi
sing
ng answ
answer
ers,
s, follo
follow
w the
the requ
requir
irem
emenents
ts of stan
standa
dard
rd writt
written
en Engl
Englis
ish.
h. Pay
Pay atte
attent
ntio
ion
n to
gramm
grammar,
ar, choic
choicee of words
words,, senten
sentence
ce constr
construc
ucttion,
ion, and
and punct
punctuat
uation
ion.. Choo
Choose
se among
among the optio
options
ns
which
which revision
revision results
results in a sentence
sentence that is clear and precise.
precise.

40. Ally’s cat was finall


finally
y found
found shive
shiverin
ring
g under
under the
the stairs
stairs of the
the house
house covere
covered
d with
with mud
mud and
ravenously hungry.

(A) finall
finally
y found
found unde
underr thestairs
thestairs of the house
house,, shive
shiverin
ring,
g, cover
covered
ed with
with mud and
and
ravenously
ravenously hungry.
(B) found
found finall
finally
y shive
shiverin
ring
g covere
coveredd with
with mud
mud and raveno
ravenoususly
ly hung
hungry
ry unde
underr the stairs
stairs
of the house.
house.
(C) found
found shive
shiverin
ring,
g, and
and covere
covered
d with
with mud
mud and raven
ravenous
ously
ly hungry
hungry unde
underr the stairs
stairs of                 
the house
house finall
finally.
y.
(D) No
No change
change

41.
41. If the gover
governme
nmentnt decid
decides.
es. To amend
amend the Const
Constitu
itutio
tion.
n. One
One of the chang
changes
es should
should be the
abolition
abolition of the unneces
unnecessary
sary and expensi
expensive
ve party-list
party-list system
system..

(A) If the gover


governme
nment
nt decid
decides;
es; to amend
amend the Consti
Constitut
tution
ion,, one of the chang
changes
es shoul
should
d
be the abolition
abolition of the unneces
unnecessary
sary and expensi
expensive
ve party-list
party-list system.
system.
(B) If the gover
governme
nment
nt decid
decides
es to amend
amend the Const
Constitu
itutio
tion,
n, one
one of the chang
changes
es shou
should
ld be
the abolition
abolition of the unneces
unnecessary
sary and expensiv
expensivee party-list
party-list system.
system.
(C)
(C) If the
the gove
govern
rnme
ment
nt deci
decide
des,
s, to amen
amend d the
the Cons
Consti
titu
tuti
tion
on,, one
one of the
the chan
change
gess shoul
should
d be
the abolition
abolition of the unneces
unnecessary
sary and expensiv
expensivee party-lis
party-listt system.
system.
(D) No change

42. “Death was


was close
close that
that we were
were alread
already
y crying
crying and prayin
praying
g the rosary
osary,”
,” one
one of the ship
ship
disaster victims narrated.

(A)
(A) was
was too clos
closee that
that we were
were alre
alread
adyy cryi
crying
ng and
and pray
prayin
ing
g
(B)
(B) was
was so clos
closee than
than we were
were alre
alread
ady
y cryi
crying
ng and
and pray
prayin
ingg
(C)
(C) was
was so clos
closee that
that we
we were
were alre
alread
ady
y cryi
crying
ng and
and pray
prayin
ingg
(D) No change

43.
43. Driv
Drivin
ing
g alon
along
g the
the Imus
Imus Hig
Highw
hway
ay,, two
two cows
cows cro
cross
ssed
ed in fron
frontt of me.
me.

(A)
(A) Cros
Crossi
sing
ng in fron
frontt of
of me,
me, I saw
saw two
two cows
cows,, while
while driv
drivin
ing
g alon
along
g the
the Imus
Imus High
Highwaway.
y.
(B)
(B) As I was
was dri
drivi
ving
ng alon
alongg the
the Imus
Imus Hig
Highw
hway ay,, two
two cows
cows cro
cross
ssed
ed in fron
frontt of me.
me.
(C)
(C) Two
Two cows
cows cros
crosse
sedd in fron
frontt of me whil
whilee driv
drivining
g alon
along
g the
the Imus
Imus High
Highwaway.
y.
(D) No change

PRACTICE BOOKLET
BKL PhiLSAT_AA_0317 GO ON TO
TO THE NEX
NEXT
NEXT
XT PAGE
PA
PAGE
AG
GE
E
10

44. Most compress


compressors
ors operate
operate rather
rather simply
simply it cranks
cranks up energy
energy to cool your
your refrigerato
refrigerator r 
using
using the same
same amou
amountnt of pow
power
er all
all day long
long but
but that
that is not the
the case
case with
with ABC
ABC’s
Inverter Compressor.

(A) Most
Most com
compre
presso
ssors
rs oper
operate
ate rather
rather simply
simply.. It cran
cranks
ks up
up energy
energy to cool
cool your 
refrigerato
refrigeratorr using
using the same amount
amount of power
power all all day long but that
that is not
the
the case
case with
with ABC
ABC’s Inverter Compressor.
(B) Most
Most com
compre
presso
ssors
rs oper
operate
ate rather
rather simply
simply.. It cran
cranks
ks up energy
energy to cool
cool your 
your 
refrigerato
refrigeratorr using
using the same amount
amount of power
power all all day long,
long, but that
that is not
the
the case
case with
with ABC
ABC’s Inverter Compressor.
Compressor.
(C) Most
Most com
compre
presso
ssors
rs oper
operate
ate rather
rather simply
simply,, it cran
cranks
ks up energ
energyy to cool
cool your 
your 

refrigerato
refrig
the caerator
the case wirthusing
se with usin
ABgCthe
ABC same amount
amou
’s Inverter  nt of power
power all
Compressor. all day long but that
that is not
(D) No change

45.
45. My moth
mother
er ask
asked
ed me why
why I did
did not tak
takee more
more pict
pictur
ures
es of the
the chil
childr
dren
en play
playin
ing
g in
the park?
park?

(A) “why
“why I did
did not
not take
take more
more pictur
pictures
es of
of the
the child
children
ren playin
playing
g in the park.
park.””
(B) “why
“why I did
did not
not take
take more
more pictur
pictures
es of
of the
the child
children
ren playin
playing
g in the park?
park?””
(C)
(C) why
why I did
did not
not take
take more
more pict
pictur
ures
es of the
the chil
childr
dren
en play
playin
ing
g in the
the park
park..
(D) No change

46.
46. Actu
Actual
ally
ly,, flyi
flyinglem
nglemur
urss don’t
don’t fly they
they just
just glid
glidee from
from tree
tree to tree.

(A) don’t fly and


and glid
glidee from
from tree
tree to tree
tree
(B) don’t
don’t fly
fly but
but glid
gliding
ing from
from tree
tree to tree
tree

(C)
(D) don’t
No chfly;
angethey   just glide from tree to tree

47.
47. Drivin
Driving
g too slowly
slowly along
along SLEX
SLEX last
last Satur
Saturday
day.. A highwa
highway
y patrol
patrol office
officerr stopp
stopped
ed
Miss Perez.

(A)
(A) Miss
Miss Pere
Perezz was
was stopp
stopped
ed by a high
highway
way patr
patrol
ol offi
office
cerr drivi
driving
ng too
too slow
slowly
ly alon
along
g
SLEX last Saturday.
(B)
(B) Driv
Drivin
ing
g too
too slow
slowly
ly alon
along
g SLEX
SLEX last
last Satu
Saturd
rday
ay,, Miss
Miss Pere
Perezz was
was sto
stopp
pped
ed by a
highway patrol officer.
(C)
(C) Driv
Drivin
ing
g too
too slow
slowly
ly alon
along
g SLEX
SLEX last
last Satur
Saturda
day,
y, a high
highwawayy patr
patrol
ol offic
officer
er stopp
stopped
ed
Miss Perez.
(D) No change

48.
48. When
Whenev
ever
er he arri
arrive
vess from
from work
work,, Alex
Alex who
who lives
lives alon
alonee in a cond
condom
omin
iniu
ium
m unit,
unit, heat
heatss his
precooked dinner.

(A)
(A) Alex
Alex,, who
who live
livess alone
alone in a cond
condom
omin
iniu
ium
m unit,
unit, heat
heatss his pre
preco
cook
oked
ed dinn
dinner
er..
(B)
(B) Alex
Alex who
who live
livess alone
alone in a cond
condom
omin
iniu
ium
m unit
unit heat
heatss his pre
preco
cook
oked
ed dinn
dinner
er..
(C)
(C) Alex
Alex,, who
who live
livess alone
alone in a cond
condom
omin
iniu
ium
m unit
unit heat
heatss his prec
precoo
ooke
ked
d dinn
dinner
er..
(D) No change

PRACTICE BOOKLET
BKL PhiLSAT_AA_0317 GO ON TO
TO THE NEX
NEXT
NEXT
XT PAGE
PA
PAGE
AG
GE
E
11

49.
49. My mothe
motherr is a woman
woman of many
many talen
talents.
ts. She
She plays
plays the harp,
harp, bakes
bakes the best
best choco
chocolat
latee
cakes,
cakes, and you
you should
should see her
her orchid
orchidss and roses.
roses.

(A) There’s
There’s her talent
talent for
for playin
playing
g the harp,
harp, and for baking
baking the best chocola
chocolate
te cakes,
cakes,
and then
then there
there are the orchid
orchidss and
and roses.
roses.
(B) She
She plays
plays the harp,
harp, bakes
bakes the best
best choco
chocolat
latee cakes,
cakes, and grows
grows orchid
orchidss and roses.
roses.
(C)
(C) She
She is pla
playi
ying
ng the
the harp
harp,, baki
baking
ng the
the best
best cho
chococola
late
te cake
cakes,
s, and
and for
for grow
growin
ing
g orch
orchid
idss
and roses.
roses.
(D) No change

50.
50. After
After Anton
Antonius
ius Strad
Stradiva
ivariu
riuss learne
learned
d to play
play the violin
violin,, he experi
experime
mente
nted
d making
making a violin
violin

that
that had the soun
sound
d of a human
human voice
voice,, and succe
success
ss was
was achiev
achieved.
ed.
(A) he succeed
(B) he succeeded
(C)
(C) hewas
hewas suc
success
cessfu
full
(D) No change

PRACTICE BOOKLET
BKL PhiLSAT_AA_0317 GO ON TO
TO THE NEX
NEXT
NEXT
XT PAGE
PA
PAGE
AG
GE
E
12

TEST
TEST B. CRITICAL
CRITICAL THINKING
THINKING

Section
Section 1. Logical
Logical Reason
Reasoning
ing

Directions
Directions:: In this section,
section, brief conversa
conversations
tions or short
short passages
passages are given
given followed
followed by one or 
two
two ques
questi
tion
ons.
s. Thes
Thesee ques
questi
tion
onss ask
ask for
for the anal
analysysis
is and
and eval
evalua
uati
tion
on of the
the reas
reason
onin
ing
g in the
conversa
conversation
tion or passag
passage.e. It is conceiv
conceivable
able that all
all of the four choic
choices
es given
given for each
each question
question may
  be correc
correctt answers
answers.. The task
task is to sel
select the
the best
best answer
answer for
for each
each quest
question
ion.. The best
best answer
answer is the
the
one which
which does not
not require
require assumptio
assumptions ns that are irrelevant
irrelevant or inconsiste
inconsistent
nt with the statement
statements.
s.

1. About 90% of the world’s


orld’s populati
population
on growth
growth is occurring
occurring in the underde
underdevelo
veloped
ped and
devel
develop
oping
ing nation
nationss of the Third
Third World
World.. The
The popula
populatio
tion
n growth
growth rates
rates in these
these poor 
poor 
countries
countries make it diffic
difficult
ult to provide
provide the bare necessitie
necessitiess of
of housi
housing,
ng, fuel,
fuel, food,
food, and
medical
medical attention
attention.. Ironicall
Ironically,
y, there is a relations
relationship
hip between
between poverty
poverty and fertility:
fertility:
The greater
greater the proportio
proportion n of a given
given populati
population
on living
living in poverty,
poverty,___
______
______
______
_____.
__.

(A)
(A) the
the high
higher
er is the
the unem
unempl ploy
oyme
ment
nt rate
rate in the
the coun
countr
try
y
(B) the highe
higherr is the fertil
fertility
ity rate
rate of that
that countr
country y
(C) the great
greater
er is the tende
tendency
ncy toward
towardss disside
dissident
nt activi
activitie
tiess
(D) the great
greater
er is the
the propo
proportio
rtion
n of illega
illegall and crimin
criminal
al incid
inciden
ents
ts
  ______________________________________________________________________________ 

Jason: You
You should
shouldn’t
n’t smoke
smoke so much,
much, Honey
Honey.. Smok
Smoking
ing is not really
really good for your
your lungs.
lungs.
Kathy: Darlin
Darling,
g, I don’t
don’t agree
agree with
with you.
you. I have
have been
been smokin
smoking g for twenty
twenty years
years now
now,, and
and I
have not developed
developed emphysem
emphysema.a.

2. Whic
Which
h of the foll
follow
owin
ing
g resp
respon
onse
sess would
would best
best stren
strengt
gthe
hen
n and
and expla
explain
in Jaso
Jason
n’s argument?

(A) If you
you keep
keep smokin
smoking,g, you
you may find
find that
that you
yourr cigare
cigarette
tte consu
consump
mptio
tion
n will
will keep
keep
increa
increasin
singg in the future
future..
(B) Smok
Smoking
ing does
does not
not always
always devel
develop
op emphy
emphysemsema.a.
(C) Deve
Develop
loping
ing emphy
emphysemsemaa is not
not the only reaso
reason n why
why smoki
smoking
ng is not
not good
good for 
a person.
person.
(D) Many
Many perso
persons
ns who
who smoke
smoke as much
much as Kath
Kathyy does
does deve
develop
lop emphy
emphysem
sema.
a.
  ______________________________________________________________________________ 

For
For items
items 3 and
and 4, refer
refer to the follow
following
ing passa
passage
ge..

Today’s
Today’s methods
methods of discip
discipline
line are a far cry from those
those usually
usually employed
employed just twenty
twenty (20)
year
yearss ago.
ago. In the
the past
past,, the
the maxi
maxim m “spa
“spare
re the
the rod,
rod, and
and spoi
spoill the
the chil
child”
d” has
has been
been take
taken
n lite
litera
rall
lly
y
that
that corp
corpora
orall pun
punish
ishmen
mentt was
was the norm
norm of the day.
day. Span
Spankin
king,g, kneel
kneeling
ing on mongo
mongo seeds,
seeds, or 
stand
standing
ing unde
underr the heat
heat of the sun werewere some
some measu
measures
res taken
taken to ensur
ensuree that
that the child
child would
would
grow upright.
upright. This behavio
behaviorist
rist assum
assumption
ption was the
the norm of the day until research
research has exposed
exposed

the
internegative
nega
internatiotive
nal psychol
national psychologic
laws banogical
thealsaid
effects
effect s of such
practices
prac tices kinds
kind
and s ofprescribe
now corrective
correcti
pres cribeve
an behavio
beh avior.
r. e,
alternativ
alter Current
Curr
native, enthnational
which
whic natio nal andy
is popularl
pop ularly
know
known n as posi
positi
tive
ve disc
discip
ipli
line
ne.. In this
this styl
stylee of disc
discip
ipli
line
ne,, chil
childr
dren
en are
are made
made to real
realiz
izee
the conseque
consequencesnces of
of their wrong
wrongdoin
doing.g. Counselli
Counselling
ng and withdraw
withdrawinging of privilege
privilegess have now
replac
replaced
ed the ire of the beatin
beatingg stick
stick..

PRACTICE BOOKLET
BKL PhiLSAT_AA_0317 GO ON TO
TO THE NEX
NEXT
NEXT
XT PAGE
PA
PAGE
AG
GE
E
13

3. Whic
Which
h of the
the foll
follow
owin
ing
g assu
assump
mptio
tions
ns can
can be safe
safely
ly assu
assume
med
d as true
true??

(A)
(A) Chil
Childr
dren
en born
born at the time
time of the writi
writing
ng of the artic
article
le are
are most
mostly
ly undi
undisc
scip
ipli
line
ned.
d.
(B) There
There were
were no forms
forms of corpo
corporal
ral punis
punishme
hment
nt employ
employed ed from
from twent
twenty
y years
years ago until
until
the time
time of the articl
articlee’s writing.
(C)
(C) Thos
Thosee who
who were
were born
born twen
twentyty year
yearss and
and prio
priorr to
to that
that have
have expe
experie
rienc
nceded some
some form
form
of corporal punishment.
(D) Most
Most of those
those who
who havehave unde
undergo
rgone
ne corpo
corporal
ral pun
punish
ishmen
mentt exhibi
exhibitt psych
psycholo
ologic
gical
al
disorders.

4. Whic
Which
h of the
the defi
defini
niti
tion
onss belo
below
w may
may be said
said to be stra
strayi
ying
ng from
from the
the idea
ideass comm
commun
unic
icat
ated
ed

in the passag
passage?
e?
(A)
(A) Posit
ositiv
ivee disc
discipipli
line
ne is anyany form
form of dis
disci
cipl
plin
inee that
that yie
yield
ldss posi
positi
tive
ve resu
result
ltss on a
child’s behaviour.
(B)
(B) Beha
Behavi
viou
ouri
rist
stss bel
belie
ieve
ve that
that good
good deed
deedss are
are to be rewa
reward
rded
ed whil
whilee bad
bad deed
deedss are
are to
  be punishe
punished.
d.
(C) Corpora
Corporall punishm
punishment ent is a form of disciplin
disciplinee that involves
involves physical
physicallyly hurting
hurting a child
to correct behaviour.
behaviour.
(D)
(D) A maxi
maximm is a popu
populalarr sayin
sayingg whic
which
h also
also serv
serves
es as a guid
guidin
ing
g prin
princi
cipl
ple.
e.
  ______________________________________________________________________________ 

Since
Since all the
the floors
floors in
in the buildi
building
ng I have
have inspec
inspected
ted have
have marbl
marblee tiles,
tiles, it follow
followss that
that
the other
other floors
floors whic
whichh I have
have not
not seen
seen will
will also have
have marble
marble tiles.
tiles.

5. Whic
Which
h of the
the foll
follow
owin
ing
g have
have a para
parall
llel
el rea
reaso
soni
ning
ng to the
the stat
statem
emen
entt abov
above?
e?

(A) Ever
Ev ery
chur y town
church
chto
fawn
faci I have
cing
ng havpleaza
a plazvisi
viasite
ted
dsthas
must
mu has aa chur
be ch urch
town
town.ch
. faci
facing
ng a plaz
plaza;a; so a comm
commun unity
ity with
with a
(B) Some
Some newspa
newspaperperss are not
not selling
selling very
very well;
well; there
therefor
fore,
e, tabloi
tabloids
ds are
are not
not sellin
selling
g
very well.
(C) Since
Since the car deale
dealerr does
does not
not buy
buy cars
cars olde
olderr than
than five
five years
years old,
old, his new purcha
purchase
se
will
will proba
probably
bly be no older
older than
than five
five years
years old.
old.
(D)
(D) Sinc
Sincee all
all of the chil
childr
dren
en I know
know fear
fear monk
monkeyeys,s, ther
theree must
must be some
someththin
ing
g in the
monkeys that scare children.
  ______________________________________________________________________________ 

6. If a boy
boy grew
grew up in Mani
Manila
la,, then
then he is stre
street
et smar
smart.
t. This
This stat
statem
emen
entt can
can be deduc
deduced
ed
logically
logically from which
which of the following
following statement
statements?
s?

(A)
(A) Ever
Everyy stre
street
et smar
smartt boy
boy grew
grew up in Mani
Manila la or anoth
another
er city
city..
(B) The
The boy
boy in the statem
statemen
entt is either
either stree
streett smart
smart or app
appear
earss remar
remarka
kably
bly to be
street smart.
(C)
(C) Ther
Theree is no boy
boy who
who grew
grew up in Mani
Manilala who
who is not
not str
stree
eett smar
smart.
t.
(D)
(D) The
The majo
majoririty
ty of boys
boys who
who grew
grew up in Mani
Manila la are
are stre
street
et smar
smart.
t.

PRACTICE BOOKLET
BKL PhiLSAT_AA_0317 GO ON TO
TO THE NEX
NEXT
NEXT
XT PAGE
PA
PAGE
AG
GE
E
14

7. Food
Food prod
producucti
tion
on in the
the Thir
Third
d Worl
World d nati
nation
onss is
is mor
moree adve
advers
rsel
ely
y affe
affect
cted
ed by natu
natura
rall
disasters
disasters,, like floods
floods and
and droughts
droughts,, than it is in more affluent
affluent nations
nations becaus
becausee affluent
affluent
nations
nations are likely to have ______
__________
_____________
____._.

(A) mode
modernrn instru
instrume
ments
nts for accurat
accuratee foreca
forecasti
sting
ng of weathe
weather r 
(B) adequ
adequate
ate flood
flood contr
control,
ol, irriga
irrigatio
tion
n system
systems,s, and storag
storagee facili
facilitie
tiess
(C) enou
enough
gh agric
agricult
ultura
urall scient
scientist
istss and
and techn
technolo
ologis
gists
ts in food
food resear
research
ch and produ
producti
ction
on
(D) the neces
necessar
sary
y aircra
aircraft,
ft, pilots
pilots,, chemi
chemical
cals,
s, and equipm
equipmentent for
for undert
undertaki
aking
ng cloud
cloud
seedi
seeding
ng to induce
induce rain
rain

8. Doct
Doctor
orss and
and nurs
nurses
es are,
are, for
for the
the most
most part
part,, tota
totally
lly comm
committ
itted
ed to savi
saving
ng lives
lives.. A life
life lost
lost

is, fornasia
Eutha
Euthanasthem,
them
ia ,asalmost
almo stctice
a personal
a practi
pra pers
ce onal
ht failure,
might
mig failu
wellre, anthis.
alter
alter insult
insu
this. lt to their
It could
coul skills
skill
d have
hav e as corrup
and
andruptin
cor knowled
kno wledge.
ting infge.
g influe
luence
nce,, so
that
that in severe
severe cases,
cases, docto
doctors
rs and
and nurse
nursess might
might not
not try hard
hard enoug
enough h to save
save the patie
patient.
nt.
They
They might
might decid
decidee that
that the patient
patient would
would simply
simply be “bett
“better
er off
off dead”
dead” and take
take the steps
steps
neces
necessar
sary
y to make
make that
that come about
about.. This
This attitud
attitudee could
could then
then carry
carry over
over to their deali
dealing
ngss
with patients
patients less seriousl
seriouslyy ill. This would
would probably
probably result
result to _______
________________
_________
______..

(A) loss
loss of confid
confidenc
encee in the medica
medicall profe
professi
ssion
on
(B) nume
numerou
rouss cases
cases filed again
against
st doctor
doctorss and nurse
nursess
(C)
(C) an over
overal
alll decl
declin
inee in the
the qual
quality
ity of med
medic
ical
al car
caree
(D)
(D) a publ
public
ic outc
outcry
ry agai
agains
nstt euth
euthan
anas
asia
ia
  ______________________________________________________________________________ 

For
For items
items 9 and
and 10,
10, refer
refer to the follow
following
ing situa
situatio
tion.
n.

A conversa
conversation
tion between
between parents
parents of schoolch
schoolchildre
ildren:
n:

Sam:
Sam: Have
Have you
you heard
heard the news?
news? TheThe Princi
Principa
pall told
told me they
they are raisin
raising
g school
chool fees
fees nex
nextt year
year..
Pat: They’re heartless!
Sam: Well,
Well, accord
according
ing to him,
him, they’r
they’ree planni
planning
ng to put
put up a newscience
newscience labora
laborator
tory.
y. I alsoheard
they’re
they’re hiring
hiring more teachers
teachers next
next year.
year.
Pat: But why
why now?
now? Don’t
Don’t they
they realize
realize that prices
prices of goo
goods contin
continue
ue to inflat
inflate?
e? Do they
they want
want
us to suffer?
suffer?
Sam: I don
don’t
’t think
think it’s that,
that, altho
althougugh
h I hav
havee to admit
admit sud
sudde
denn increa
increases
ses can be worri
worriso
some.
me.
I was thinking
thinking of writing
writing a letter
letter asking for increment
incremental al increases.
increases.
Pat:
Pat: I am not sure
sure about
about that,
that, thoug
though.h. You
You see,
see, I got
got to talk
talk to one
one of the teach
teachers
ers.. His salary
salary
didn’t
didn’t increas
increasee in years!
years!

9. Whic
Which
h of the
the foll
follow
owin
ing
g is the
the main
main caus
causee of tens
tensio
ion
n in the
the conv
conver
ersa
sati
tion
on abov
above?
e?

(A) The teachers


teachers demand
demand higher
higher salaries
salaries in exchange
exchange for better
better teaching
teaching practices
practices..
(B) The
The impen
impendin
dingg increa
increase
se in tuitio
tuition
n fees
fees create
createss anxiet
anxiety
y among
among the paren
parents.
ts.
(C) Parents
Parents feel that the school
school is
is indiffer
indifferent
ent to their
their needs.
needs.
(D) The
The expert
expertise
ise of schoo
schooll employ
employees
ees is being
being quest
question
ioned
ed..

PRACTICE BOOKLET
BKL PhiLSAT_AA_0317 GO ON TO THE NEXT PAGE
15

10. Which
Which of the follow
following
ing argum
argument
entss is the LEAS
LEAST
T relev
relevant
ant to the issue
issue discus
discussed
sed in
the dialogue?
dialogue?

(A) The
The increa
increase
se in schoo
schooll fees
fees is
is justi
justifie
fied
d by the estab
establis
lishme
hment
nt of
of a new
new labora
laborator
tory.
y.
(B) An increa
increase
se in schoo
schooll fees
fees should
should trans
translat
latee to better
better salari
salaries
es for
for teach
teachers
ers and
and staff.
staff.
(C) Scho
School
ol fees
fees shou
should
ld not incre
increase
ase becaus
becausee paren
parents ts are burde
burdene
nedd by inflat
inflation
ion..
(D) Gradual
Gradual increa
increases
ses in school
school fees can
can help ease the parents’
parents’ burde
burden.n.
 _____________
___________________
__________________
_________________
_____________
_____________
__________________
__________________
____________
__________ 
____ 

Rizza:
Rizza: All
All the gradu
graduate
atess from
from Schoo
Schooll A go to Colle
College
ge Z.
Gary: I’m not
not sure.
sure. Some
Some of the
the stude
students
nts at Colle
College
ge Z come
come from
from Scho
School
ol B.

11. Gary’s
Gary’s response
response means
means that he has interpret
interpreted
ed Rizza’s
Rizza’s statemen
statementt to mean that 
that  _______.

(A) School
School A is better
better than School
School B
(B) most
most of the students
students from School
School A attend
attend College
College Z
(C) some
some School
School B graduate
graduatess do not attend
attend college
college
(D) only students
students from School
School A attend
attend College
College Z
  ______________________________________________________________________________ 

The deman
demandd that
that the death
death penalt
penalty
y be prove
provenn more
more deter
deterren
rentt than
than altern
alternati
ative
vess cannot
cannot be
satisf
satisfied
ied any
any more
more than
than the deman
demand d that
that six
six years
years in prisonbe
prisonbe prove
proven n to be more
more deterr
deterrent
ent than
than
three.
three. But the uncertain
uncertainty
ty which
which confron
confronts
ts us favors
favors the death
death penalty
penalty as long as, by imposing
imposing
it, we might
might save future
future victims
victims of heino
heinous
us crimes
crimes..

12.
12. Whic
Which
h of the
the foll
follow
owin
ing
g is the
the main
main poin
pointt of the pass
passag
age?
e?

(A) Imposing
Impo sing the
of heinous death
death penalty
crimes. penalty as punishm
punishmentent has been instrumen
instrumental
tal in the reduction
reduction
(B) It is difficult
difficult to
to prove
prove whether
whether or not
not the death
death penalty
penalty is a better
better deterren
deterrentt to
heinous crimes
crimes than other modes of penalty.
penalty.
(C) It has
has been
been satisf
satisfact
actori
orily
ly prove
provenn that
that impo
imposit
sition
ion of the death
death penal
penalty
ty is a bette
better r 
deterrent to heinous crimes than imprisonment.
imprisonment.
(D) There
There is no alternative
alternative to capital
capital punishme
punishment nt as
as a deterrent
deterrent to heinous
heinous crimes.
crimes.
  ______________________________________________________________________________ 

For
For items
items 13
13 and
and 14,
14, refer
refer to the follow
following
ing speec
speech.
h.

My dear
dear gra
gradu
duatates
es,, I impl
implor
oree you
you at thi
thiss mome
moment nt to
to use
use your
your tal
talen
entsts and
and skil
skills
ls in
improv
improving
ing our
our mothe
motherlarland.
nd. I beg
beg of you
you:: if you
you really
really do love
love this
this coun
country
try,, then
then you will
will not
not
work
work or do business
business elsewhere
elsewhere.. Don’t
Don’t you
you know
know that there are millions
millions of
of starvin
starving g mouths
mouths toto
feed
feed in our
our borde
borders
rs?? Take
Take a look
look at Cuba
Cuba.. For
For years
years,, her
her doors
doors were
were clos
closed
ed from
from the
the outs
outsid
idee
worl
world,
d, and
and yet,
yet, she
she is abl
ablee to prov
provid
idee free
free medi
medicacall serv
servic
ices
es to
to her
her citiz
citizen
ens.
s. If Cub
Cubaa made
made it,
then
then so can
can we.

I am
am sure that if you use your
your creati
creativity
vity and passion
passion in our country,
country, things
things will surely
surely get
better.
better. Whether
Whether you
you admit
admit it or not,
not, working
working elsewhere
elsewhere equals
equals abandonmen
abandonmentt of the land that
fed you for the longest
longest time.

PRACTICE BOOKLET
BKL PhiLSAT_AA_0317 GO ON TO
TO THE NEX
NEXT
NEXT
XT PAGE
PA
PAGE
AG
GE
E
16

13. Which
Which of the follow
following
ing statem
statement
entss accurat
accurately
ely depic
depicts
ts the bias
bias of the gradu
graduati
ation
on speake
speaker?
r?

(A) Inves
Investin
ting
g locall
locally
y is the best
best way to help
help the poor
poor of the speake
speaker’s
r’s coun
country
try..
(B) The
The gradu
graduate
atess will have
have to study
study and employ
employ Cuba
Cuba’s ’s examp
examplele for their
their count
country
ry to
improve.
(C) Leavi
Leaving
ng the count
countryry is fine
fine as long
long as one
one return
returnss to rende
renderr serv
service
ice or goods
goods..
(D) Love
Love for countr
countryy canno
cannott be expres
expressed
sed if one
one goes
goes abroa
abroadd to work.
work.

14. Which
Which of the statem
statemen
ents
ts below
below depic
depicts
ts a logica
logicall flaw
flaw in the speech
speech??

(A) The
The ones
ones who
who are gradu
graduati
ating
ng posses
possesss talent
talentss and
and skills
skills..

(B) Cuba
(C) Those
Those
Cu who
who
ba has leave
leaessful
succ ve fully
success thelycountr
country
made
mad eyavail
toailabl
av work
wo rke are
able not
notalpatrio
medic
medical pat riotic
tic.
servic
ser . to her
vices
es her citizens
citizens..
(D)
(D) Ther
Theree are
are seve
severa
rall poor
poor peopl
peoplee in the
the coun
countrtry
y wher
wheree the
the spea
speake
kerr and
graduates belong.
  ______________________________________________________________________________ 

Most philosoph
philosophersers agree that punishmen
punishment, t, in general,
general, is a morally
morally justified
justified social
social practice.
practice.
However
However uneasy
uneasy we might
might feel
feel about
about inflictin
inflictingg harm on another
another person,
person, it is hard to visualize
visualize
a _____
_______
________
____
__ manag
managinging to surviv
survivee witho
withoutut an establ
establish
ished
ed legal
legal syste
systemm of punis
punishm
hment
ent..
On the other
other hand
hand,, dissen
dissenter
terss argue
argue that
that it is possibl
possiblee to struct
structure
ure socie
society
ty in ways
ways that
that would
would
not necess
necessitate
itate commitmen
commitmentt to a legal
legal system
system of punishm
punishment.
ent. For examp
example,le, might
might it not be that
undesirab
undesirable
le social
social behavior
behavior could
could be adequate
adequately ly kept in check
check by therapeut
therapeuticic treatment
treatment rather 
rather 
than by tradition
traditional
al kinds
kinds of punishm
punishmentent such as imprisonm
imprisonment ent or the death
death penalty?
penalty? Such
Such a
system
system would
would certa
certainl
inly
y have
have the advan
advantag
tagee of being
being more
more human
humane. e.

15.
15. Whic
Which
h of the
the foll
follow
owin
ing
g best
best fit
fitss in the
the blan
blank?
k?

(A) despotic
despotic government
government
(B) multiracial population
(C) developing
developing country
(D) complex
complex society
society
  ______________________________________________________________________________ 

If you
you believ
believee that
that all peop
people
le are basica
basically
lly good
good and have
have poten
potentia
tial,
l, you
you can be led down
a lot
lot of path
pathss to the
the orga
organiniza
zati
tion
onal
al aby
abyss
ss.. You
You assu
assume
me that
that a bett
better
er env
envir
iron
onme
ment
nt,, more
more
empowerm
empowerment,ent, and juicier
juicier assig
assignmen
nments ts will
will do
do the trick.
trick. These
These are wonderfu
wonderfull concep
concepts,
ts, but if                 
the perso
personn and the job are misma
mismatchtched,
ed, you
you are desti
destined
ned for disapp
disappoin
ointme
tment.
nt.

16.
16. Whic
Which
h of the foll
follow
owin
ing
g woul
would
d be the
the best
best compl
complet
etio
ion
n of the pass
passag
agee abov
above?
e?

(A) No one can be success


successful
ful unless
unless that person
person is reasona
reasonably
bly matched
matched with what
what
he or she is expec
expectedted to do.
do.
(B) A person
person’s’s total
total dedica
dedication
tion is a sufficien
sufficientt indicati
indication
on that there
there is a match
match between
between
that
that perso
personn and
and the job.
job.
(C) Lack
Lack of intere
interest
st is the best
best indica
indicatio
tion
n of a mismat
mismatch ch betwe
between
en a perso
person
n and
and his
or her job.
(D) Bias in promotio
promotion n is a very sensit
sensitive
ive indicator
indicator of mismat
mismatches
ches between
between employee
employeess
and their jobs.
jobs.

PRACTICE BOOKLET
BKL PhiLSAT_AA_0317 GO ON TO THE NEXT PAGE
17

For
For items 17
17 and
and 18,
18, refer
refer to the follow
following
ing passag
passage.
e.

One of my
my biggest
biggest qualm
qualmss is the futility
futility of conv
convincin
incingg my students
students to read.
read. Today’s
Today’s kids
seem
seem to convu
convulse
lse at the idea
idea of going
going throu
through
gh a book
book until
until its end.
end. What’
What’ss wors
worse,
e, in my thirty
thirty
years
years of
of teach
teaching
ing English,
English, it seems
seems students
students get more and more repulsed
repulsed to the idea of reading
reading
as time goes on.
on.

I now
now appre
apprecia
ciate
te the fact
fact that
that my
my family
family didn’t
didn’t own
own a TV set when
when I was
was young
young.. I had
had to
  be imagin
imaginati
ative
ve to pass
pass time
time   – that’s
that’s why
why I read.
read. Readin
Readingg has
has taug
taught
ht me to imagin
imaginee and think
think..
I have
have always
always marve
marvelle
lled
d at how
how words
words in a page
page can take
take me to place
places,
s, even
even world
worldss I had
had never 
never 
fancie
fanciedd in my wilde
wildest
st drea
dreams
ms.. Too bad
bad many
many kids
kids nowa
nowaday
dayss are
are hook
hookeded into
into digita
digitall devic
devices
es

that
that can
can show
show them
them ever
everyt
ythi
hing
ng in one
one clic
click.
k. Yup,
Yup, no more
more time
time to read
read..
17.
17. Base
Based
d on the
the pass
passag
agee abov
above,
e, whic
which
h of the
the foll
follow
owin
ing
g assu
assump
mptio
tions
ns may
may be consi
conside
dere
red
d as
absolutel
absolutely
y true?

(A) Televisio
Televisionn has not
not yet
yet been
been invented
invented at the time of the narrator’
narrator’ss childho
childhood.
od.
(B)
(B) All
All kids
kids born
born rece
recent
ntly
ly have
have no natu
natura
rall prope
propens
nsity
ity to read
read..
(C) Moder
Modern-n-day
day device
devicess preven
preventt kids from
from readin
reading.
g.
(D) Engli
English
sh has been
been taugh
taughtt as a staple
staple schoo
schooll subje
subject.
ct.

18. Which
Which of the following
following statement
statementss best encapsulate
encapsulatess the author’s
author’s main assumption?

(A) Reading
Reading is the best way to increase
increase a person’s
person’s intelligen
intelligence.
ce.
(B) Modern-day techn
technolo
ologie
giess contri
contribu
bute
te to the decli
decline
ne in a child’
child’ss intere
interest
st in readin
reading.
g.
(C) Informati
Information
on literacy
literacy is necessary
necessary to survive
survive in today’s
today’s world.
world.
(D) Read
Reading
ing habit
habitss are
are diffic
difficult
ult to estab
establis
lish
h once
once a child
child reach
reaches
es ado
adoles
lescen
cence.
ce.

  ______________________________________________________________________________ 
19. Judgin
Judging
g from
from the conte
content
nt of the
the statem
statement
ent and
and the autho
authorit
ritati
ativen
veness
ess of
of the speak
speaker,
er,
which
which of the following
following is the most reasonab
reasonable
le and trustwort
trustworthy?
hy?

(A) City
City Mayo
Mayor: r: The
The peso’s
peso’s recov
recovery
ery in the past
past few days shou
should
ld be a boom
boom to
indus
industry
try,, becau
because se a stron
strongg peso
peso makes
makes the Philip
Philippin
pineses an attrac
attractiv
tivee destin
destinati
ation
on for 
international investors.
(B) Policema
Policeman: n: Sex
Sex educatio
education n is the best means
means of addressi
addressingng sexual
sexual ignoranc
ignorancee and
will greatly
greatly help minimize,
minimize, if not totally
totally eradicate
eradicate,, sex offenses,
offenses, such as child
molestati
molestation,on, rape,
rape, and incest.
incest.
(C) Sociolo
Sociologist:
gist: Religion
Religion cuts across
across all
all segments
segments of society.
society. True believers
believers and those
those
merel
merelyy supers
superstit titiou
iouss seek
seek sanctu
sanctuary
ary in it. The
The masses
masses and the powerf
powerful,ul, simila
similarly
rly
fearful of external damnation, commingle in this fraternity.
(D) Lawyer:
Lawyer: One of the the most serious
serious signs
signs of the increase
increase in moral
moral corruptio
corruption n is
the bound
boundleslesss exaltat
exaltation
ion of sex.
sex. Sex
Sex has even
even invade
invaded d the field
field of educat
education
ion with
with
the help of mass
mass media
media..

PRACTICE BOOKLET
BKL PhiLSAT_AA_0317 GO ON TO
TO THE NEX
NEXT
NEXT
XT PAGE
PA
PAGE
AG
GE
E
18

20. Only Golden


Golden Harvest
Harvest Beer
Beer has a biting
biting efferves
effervescent
cent taste that gives
gives you that all-night
all-night
relaxed
relaxed feeling.
feeling. If this advertisi
advertising
ng claim is true, which
which of the following
following propositi
propositions
ons
CANN
CANNOT OT be true?
true?

I. Maha
Maharli
rlika
ka Beer
Beer is a mor
more popu
popular
lar beer
beer than
than Golde
Golden n Harve
Harvestst Beer.
Beer.
II. Sulta
Sultana
na Beer
Beer give
givess you that
that all-nig
all-night
ht relax
relaxed
ed feelin
feeling.
g.
III.
III. Golde
Goldenn Har
Harvest
vest Cola has that
that biting
biting efferv
effervesc
escent
ent taste
taste of the Golde
Goldenn
Harvest Beer.

(A) I and II only


(B) I and III only

(C)
(D) II II,
(D) I, and
II anIII
, and only
d III
III
  ______________________________________________________________________________ 

For
For items
items 21
21 and
and 22,
22, refer
refer to the follow
following
ing speec
speech.
h.

A privilege
privileged
d speech
speech by a lawmaker
lawmaker against
against the distributi
distribution
on of contrace
contraceptive
ptivess in schools:
schools:

Greeti
Greeting
ngs,
s, fellow
fellow citize
citizens
ns!! Today,
Today, I stand
stand before
before you
you to expres
expresss my disap
disappro
proval
val of the
the
  propose
proposedd distribut
distribution
ion of contra
contracept
ceptives
ives in schools
schools to prevent
prevent teenage
teenage pregnan
pregnancy.
cy. First
First of all,
schools
schools are built not
not just
just to train our kids
kids in academic
academicss but also
also to instil
instil positive
positive values
values among
among
the youth
youth.. Hence
Hence,, distri
distribu
butin
tingg contra
contracep
ceptiv
tives
es is tantam
tantamou
ount
nt to sugges
suggestin
ting
g to our
our youn
young g ones
ones
that promiscu
promiscuity
ity is permissib
permissible.le. Furtherm
Furthermore,
ore, these
these kids are sent to school
school by their parents
parents who
who
dream
dream of givi
giving
ng them
them the best
best life possi
possib
ble. Wouldn’t
Wouldn’t parents
parents also be dismayed
dismayed by this move?
move?
They
They sent
sent their
their kids
kids to schoo
schooll to study
study and not
not to engag
engagee in immora
immorall acts.
acts.

21. What form of argument


argumentation
ation did the lawmaker
lawmaker use in his speech?
speech?
(A) Post hoc   –      Two
          things
things happen
happening
ing one after another,
another, and it is assumed
assumed that these
two are connecte
connected. d.
(B) Straw man  –  Refuting Refuting an argumentargument by raising
raising an irrelevan
irrelevantt issue.
issue.
(C)
(C) Swee
Sweepi
ping
ng gene
genera raliz lizat
atio
ionn –  Comi
Coming
ng up with
with a conc
conclu
lusi
sion
on base
based d on a limit
limited
ed
number of examples.
(D)   Ad hominem 
hominem   –        Attack
Att
      acking
ing the perso
personn who
who propo
proposed
sed the argum
argument
ent instead
instead of the
argument itself.

22. Which
Which of the
the follow
following
ing statem
statement
entss is most
most logica
logically
lly soun
sound?
d?

(A) Schools
Schools are built not just for academic
academicss but
but also to nurture
nurture values.
values.
(B) Distribu
Distributing
ting contrace
contraceptive
ptivess promote
promote sexual
sexual promiscu
promiscuity.
ity.
(C) The
The gove
governm
rnmenentt becom
becomeses insensi
insensitiv
tivee to paren
parents
ts when contr
contrace
acepti
ptives
ves
are distribut
distributed.
ed.
(D) Parents
Parents will get angry
angry if contrace
contraceptiv
ptives
es are distribut
distributed
ed in schools.
schools.

PRACTICE BOOKLET
BKL PhiLSAT_AA_0317 GO ON TO THE NEXT PAGE
19

Civilian:
Civilian: All policeme
policemen n are corrupt.
corrupt.
Policeman
Policeman:: That is not true. I know
know some civilians
civilians who are corrupt,
corrupt, too.

23. The policeman’s response shows his interpretation


interpretation of the
the civilian’s statement to
mean
mean that
that

(A) some
some policeme
policemenn are corrupt
corrupt
(B) civilians
civilians are more corrupt
corrupt than policemen
policemen
(C) policeme
policemen n are more corrupt
corrupt than civilians
civilians
(D) only policemen
policemen are corrupt
corrupt
  ______________________________________________________________________________ 

One
One way
way to determ
determine
ine wheth
whether er or not
not a socia
sociall practic
practicee is moral
morally
ly corre
correct
ct is to see if it is
eith
either
er perm
permit
itte
ted
d or requ
requir
ired
ed by the
the prin
princi
cipl
plee of equa
equalit
lity.
y. To say
say that
that a soci
social
al prac
practi
tice
ce is
permit
permitted
ted is to say that
that its
its inst
institu
itutio
tionn will
will not
not violat
violatee the moral
moral princ
principl
iple.
e. To say that
that itit is
requ
requir
ired
ed is to say
say that
that we must
must inst
institu
itute
te such
such a prac
practitice
ce if we are
are to act
act in a mora
morally
lly corr
correcectt
way.
way. Accord
According
ing to that
that princi
principle
ple,, equal
equalss must
must be treate
treatedd as equal
equals,
s, while
while unequ
unequals
als must
must be be
treated
treated unequally
unequally,, in proportio
proportionn to their differenc
differences.
es.

24. The author


author of this passage
passage would
would most likely agree
agree that ______
__________
_______
______
______
______
_____.
__.

(A) race is relevant


relevant in the hiring
hiring of teachers
teachers in a predomin
predominantly
antly Chinese
Chinese
exclusiv
exclusivee school
school
(B) women
women cannot
cannot be as good as or better
better than men in politics
politics or diplomacy
diplomacy
(C) it is in the best
best interest
interest of society that social institutions be nonegalitarian
in order
order to avoid
avoid direc
directt competi
competitio
tion
n betwe
betweenen men and wome
women n
(D) young
young children
children cannot
cannot be given
given the same rights
rights and responsi
responsibiliti
bilities
es as adults
adults

______________________________________________________________________________
The area
area of value
valuess educat
education
ion has been
been fraug
fraught
ht with
with confu
confusio
sionn and diffic
difficult
ulty.
y. Not
Not long
long
ago,
ago, some
some educat
educators
ors insisted
insisted on values
values-fr
-free
ee educa
educatio
tion
n and pride
pridedd themse
themselvelvess on dealin
dealingg only
only
with
with fact
facts,
s, and
and not
not valu
values
es,, whic
whichh they
they cont
conten
ende
ded
d were
were the
the prov
provin
ince
ce of the
the ____
______ ____
____
____
___.
_.
The very act of of teaching
teaching,, however
however,, presupp
presupposes
oses its own
own set ofof values,
values, including
including for examp
example,
le,
achi
achiev
evem
emen
ent,t, nurt
nurtur
uran
ance
ce,, and
and the
the valu
valuee of educ
educatatio
ion
n itsel
itself.
f. To the
the exte
extent
nt that
that the
the teac
teache
her r 
interacts
interacts with students,
students, these
these values
values are being
being modeled.
modeled.

25.
25. Whic
Which
h of the
the foll
follow
owin
ing
g best
best fit
fitss in the
the blan
blank?
k?

(A) students themselves


(B) church
church and home
(C) parents only
(D) parish
parish priest
priest

PRACTICE BOOKLET
BKL PhiLSAT_AA_0317 GO ON TO
TO THE NEX
NEXT
NEXT
XT PAGE
PA
PAGE
AG
GE
E
20

Section
Section 2. Analytica
Analy
Analytical
ticall Reasoning
Reasonin
Reasoning
g

Direct
Direction
ions:
s: In this
this sectio
section,
n, some
some sets
sets of condit
condition
ionss or propo
proposit
sition
ionss are given.
given. Each
Each set is
follow
followed
ed by quest
question
ionss based
based on the cond
conditio
itions
ns or propo
proposit
sition
ions.
s. Draw
Drawing
ing a diagra
diagramm to visua
visually
lly
estab
establis
lish
h linka
linkages
ges or relati
relations
onship
hipss stat
stated
ed in the condit
condition
ionss may help
help in answe
answerin
ring
g some
some of the
question
questions.
s. Select
Select the best answer
answer to each question
question from the choices
choices given.
given.

For
For items
items 26 to 29, refer
refer to the follow
following
ing condit
condition
ions.
s.

I. Mr. Cruz
Cruz bough
boughtt a small
small printi
printing
ng busin
business
ess of his friend
friend who
who was
was migrat
migrating
ing to Canad
Canada.
a.

He neede
andne eded
hire
hir e adteam
to m
tea familia
famofiliariz
onrize
ly efour,
only himsel
him
fou r, self
allfof
owith
wi th the
f whom
who m business
busin ess,
are former
for , soemp
mer he decid
de
employcided
eesed
loyee s oftohis
start
stafrie
frtriend
on
ond.
n a. small
small scale
sca
He neede
ne le
ededd
at leas
leastt two
two to operoperat
atee the
the prin
printi
ting
ng pres
presss mach
machinine,
e, with
with the
the othe
otherr two
two as bind
binder 
er 
and collator.
collator.
II. The applic
applican ants
ts for operat
operator
or are Simon
Simon,, Josh,
Josh, and Mike.
Mike. The
The appli
applicacants
nts for binder
binder and
collat
collator
or are Eric,
Eric, Charl
Charlie,
ie, Fred,
Fred, and
and Harry.
Harry.
III.
III. Duri
During
ng a talk
talk with
with his
his frien
friend,d, Mr.
Mr. Cruz
Cruz foun
found d out
out that
that Fred
Fred and
and Harry
arry do notnot work
ork well
ell
toge
togeth
ther
er,, Josh
Josh disl
dislik
ikes
es Eric
Eric,, and
and Simo
Simon n has
has a long
long-s-sta
tand
ndin
ing
g feud
feud with
with Fred
Fred.. He must
must
therefore
therefore choose
choose the right
right combina
combinationtion of four people
people to avoid
avoid future
future frictions
frictions..

26. If Harry
Harry is chosen
chosen,, which
which of the follow
following
ing combi
combinat
nation
ionss CANN
CANNOT
OT be chosen
chosen to form
form
the team?
team?

(A)
(A) Simon
imon,, Josh
Josh,, and
and Char
Charli
liee
(B)
(B) Simo
imon, Mike
ike, and
and Eric
ric
(C)
(C) Simon
imon,, Mike
Mike,, and
and Char
Charli
liee

(D) Josh,
Josh, Mike,
Mike, and Eric
Eric
27. If Fred
Fred is chose
chosen,
n, who
who among
among the follow
following
ing groups
groups of appli
applica
cants
nts shou
should
ld be employ
employed
ed as
the three other
other member
memberss of the team?

(A)
(A) Josh
Josh,, Mike,
ike, and
and Char
Charli
liee
(B)
(B) Mike,
ike, Eric
Eric,, and
and Char
Charli
liee
(C)
(C) Harry
arry,, Mike
ike, and
and Eric
Eric
(D)
(D) Simon
imon,, Josh
Josh,, and
and Char
Charliliee

28. If Josh
Josh is hired
hired and
and Mike
Mike is not,
not, which
which of the follow
following
ing statem
statement
entss must
must be true?
true?

I. Harry
arry will
ill be a memb
member
er of the
the team
team..
II.
II. Char
Charli
liee will
will bea memb
member
er of the
the team
team..

(A) I only
(B) II only
(C) Both I and II
(D) Neith
Neither
er I nor
nor II

PRACTICE BOOKLET
BKL PhiLSAT_AA_0317 GO ON TO THE NEXT PAGE
21

29.
29. If Eri
Ericc is chos
chosen
en as a bind
binder
er,, whic
which
h of the foll
follow
owin
ing
g coul
could
d be the
the othe
otherr thre
threee memb
member
erss
of the team?
team?

I. Simon
imon,, Mike
Mike,, and
and Char
Charli
liee
II. Simo
imon, Mike
ike, and
and Fred
red
III.
III. Simo
imon, Mike
ike, and
and Harry
rry

(A)
(A) I and II only
(B)
(B) I and III
III only
(C)
(C) II an
and III
III on
only
(D) I, II, and III

  ______________________________________________________________________________ 
For
For items
items 30 to 33, refer
refer to the follow
following
ing condit
condition
ions.
s.

I. Ther
Theree are
are four
four newl
newlyy born
born babi
babies
es in the
the nurs
nurser
ery,
y, thre
threee girl
girlss and
and one
one boy,
boy, all
all chi
child
ldre
ren
n of                 
differ
differen
entt diplom
diplomati
aticc perso
personn
nnel
el work
workinging in the Philip
Philippin
pinee embass
embassies
ies of
of Brazil
Brazil,, Italy,
Italy,
Sing
Singap
apor
ore,
e, and
and Viet
Vietna
nam.
m. The
The crib
cribss are
are arra
arrang
nged
ed alph
alphababet
eticical
ally
ly in a row
row acco
accord
rdin
ing
g to
nation
nationali
ality
ty for easy
easy identi
identific
ficati
ation
on by visiti
visiting
ng relati
relations
ons and
and frien
friends
ds who
who may view
view the
  babies
babies from behind
behind the glass
glass panel
panel of the
the nursery
nursery room.
room.
1 1
II.
II. The
The babi
babies
es have
have vari
variou
ouss wei
weigh
ghts
ts of
of 6 lb,
lb, 6 lb, 7 lb, and 7 lb, not
not neces
necessar
sarily
ily in that
that
2 2
orde
order.
r. The
The thir
third
d baby
baby is a boy
boy and
and is the
the heav
heavie
iest
st,, whil
whilee only
only the
the Sing
Singap
apor
orea
ean
n baby
baby is
exceed
exceeded
ed in weigh
weightt by the Vietna
Vietnames
mesee baby
baby..

30. “There’s
“There’s your
your sister!”
sister!” said the Vietna
Vietnames
mesee who
who had lifted
lifted his son
son and was
was point
pointing
ing from
from
  behind
behind the glass
glass pane
panell at a baby
baby in the ____
______
____
____
_____
___ crib.
crib.

(A) first
(B) second
(C) third
(D) fourth

31.
31. Base
Basedd on the
the give
given
n info
inform
rmat
atio
ion,
n, it can
can be decl
declar
ared
ed that
that the
the baby
baby boy
boy is
a(an) _____________.

(A) Ital
talian
(B)
(B) Brazil
azilia
ian
n
(C)
(C) Vietn
ietnam
amesesee
(D)
(D) Singa
ingapo
pore
rean
an

32. Of the
the four
four newly
newly born
born babie
babies,
s, the lighte
lightest
st in
in weigh
weightt is the ____
______
____
_____
_____
__..

(A)
(A) Vietn
ietnam
ames
esee
(B) Singaporean
(C) Ital
talian
(D)
(D) Brazil
azilia
ian
n

PRACTICE BOOKLET
BKL PhiLSAT_AA_0317 GO ON TO
TO THE NEX
NEXT
NEXT
XT PAGE
PA
PAGE
AG
GE
E
22

33. Which
Which of the follow
following
ing statem
statement
entss is
is true
true??

(A) The
The Brazil
Brazilian
ian and Singa
Singapor
porean
ean babies
babies togeth
together
er weig
weigh
h heavie
heavierr than
than the Italia
Italian
n and
and
Vietnamese babies.
(B) The
The Singap
Singapore
orean
an and
and Vietna
Vietnamemese
se babies
babies tog
togeth
ether
er weig
weigh
h heavi
heavier
er than
than the
Brazilian
Brazilian and Italian
Italian babies.
babies.
(C) The
The Singap
Singapore
orean
an and
and Italia
Italian
n babie
babiess togeth
together
er weigh
weigh the same
same as the Brazil
Brazilian
ian and
and
Vietnamese babies.
(D) The
The Vietna
Vietnames
mesee and Italia
Italian
n babies
babies togeth
together
er weigh
weigh the same
same as the Singap
Singapore
orean
an
and Brazilian
Brazilian babies.
babies.
  ______________________________________________________________________________ 

For
For items
items 34 to 36, refer
refer to the follow
following
ing condit
condition
ions.
s.

Employee
Employeess in a bus compan
company:
y:

I. All
All mech
mechananic
icss are
are also
also driv
driver
ers.
s.
II. Half
Half of the driver
driverss are mecha
mechanicnics.
s.
III.
III. Some
Some driver
driverss and ticket
ticket inspec
inspector
torss started
started out
out as condu
conducto
ctors.
rs.
IV. Drivers
Drivers cannot
cannot be ticket
ticket inspec
inspectors,
tors, but mechanic
mechanicss can.
can.
V. Condu
Conducto
ctors
rs cannot
cannot become
become mechan
mechanics ics..
VI. Some
Some condu
conducto
ctors
rs and
and ticket
ticket inspe
inspecto
ctors
rs do not
not kno
knoww how
how to drive.
drive.

34.
34. Whic
Which
h posi
positi
tion
on can
can be acce
access
ssed
ed by anyo
anyone
ne in the
the comp
compan
any?
y?

(A) Driver 
(B)
(B) Mech
echanic
nic

(C)
(D) Conductor 
(D) Tick
Ticket
et ins
inspe
pect
ctor 
or 

35.
35. Whic
Which
h posi
positio
tion
n is the
the most
most flex
flexib
ible
le??

(A) Driver 
(B)
(B) Mech
echanic
nic
(C)
(C) Conductor 
tor 
(D)
(D) Tick
Ticket
et ins
inspe
pect
ctor 
or 

36.
36. Whic
Which
h posi
positi
tion
on CANN
CANNOT
OT be acqu
acquir
ired
ed at entr
entry
y leve
level?
l?

(A) Driver 
(B)
(B) Mech
echanic
nic
(C)
(C) Conductor 
tor 
(D)
(D) Tick
Ticket
et ins
inspe
pect
ctor 
or 

PRACTICE BOOKLET
BKL PhiLSAT_AA_0317 GO ON TO THE NEXT PAGE
23

For
For items
items 37 to 39, refer
refer to the follow
following
ing condit
condition
ions.
s.

I. Mr. Yu’s newly constructed three-story commercial-residential


commercial-residential building along Amorsolo
St. in Makat
Makatii cons
consist
istss of
of nine
nine equal
equal unit
units.
s. He has allott
allotted
ed the three
three units
units on the groun
groundd
floor for his hardwareand
hardwareand electrical
electrical supply
supply business
business,, while
while he and his wifeplan to occupy
occupy
the middlemo
middlemost st unit on
on the second
second floor asas their residence
residence..
II.
II. The
The coup
couplele gift
gifted
ed thei
theirr new
newly
ly marr
marrie
ied
d son
son with
ith the
the righ
rightm
tmos
ostt uni
unitt on
on the
the thir
thirdd floo
floor.
r.
They
They also
also exec
execut
uted
ed a cont
contra
ract
ct with
with a grou
group p of dent
dentis
ists
ts for
for a two-
two-yeyear
ar leas
leasee on two
two
adjace
adjacent
nt units
units on one
one floor
floor to be conv
convert
erted
ed into
into a dental
dental clinic
clinic..
III.
III. The
They prefe
eferre
rred to ren
rent the
the rem
remain
aining
ing two
two units
its to fam
familie
iliess with
ith no mor
more than
than two
two
children
children each.
each.

To answe
answerr questi
question
onss 37 and 38,
38, refer
refer to the follo
followin
wing
g units
units in the building
building::

w The
The unit
= unit on the secon
secondd floor
floor dire
directl
ctly
y to the left
left of the
the coupl
couple’s
e’s unit
unit
x The leftmo
= leftmost
st unit
unit on the third
third floor 
floor 
y The unit
= unit direc
directly
tly to the left
left of the
the son’s
son’s unit
unit
z The unit directly
= directly below
below the son’s
son’s unit

37.
37. Whic
Which
h two
two unit
unitss CANN
CANNOT
OT be rent
rented
ed to fami
famili
lies
es??

(A) w an
a nd x
(B) w an
a nd y
(C) x an
a nd z
(D) x an
a nd y

38. Which
Which unit
unit is
is diag
diagon
onall
ally
y adjace
adjacent
nt to the couple’s
couple’s unit?

(A) w
(B) x
(C) y
(D) z

39. If the couple


couple decid
decides
es NOT
NOT to occup
occupy
y the middle
middlemos
mostt unit
unit but
but have
have it rented
rented also,
also, from
from
how many possible
possible pair(s)
pair(s) of adjacent
adjacent units
units will
will the
the dentists’
dentists’ group
group be able to choose?
choose?

(A) Only one


(B) Two
(C) Three
(D)
(D) At mos
mostt fou
four r 

PRACTICE BOOKLET
BKL PhiLSAT_AA_0317 GO ON TO
TO THE NEX
NEXT
NEXT
XT PAGE
PA
PAGE
AG
GE
E
24

For
For items
items 40 to 43, refer
refer to the follow
following
ing condit
condition
ions.
s.

Four
Four pre
preme
medd grad
gradua
uate
tes,
s, Stev
Steve,
e, Anne
Anne,, Vict
Victor
or,, and
and Elai
Elaine
ne,, each
each obta
obtain
ined
ed diff
differ
eren
entt passi
passing
ng
sco
scores
res, 83, 86, 90,
90, and
and 95,
95, not neces
ecessa
sari
rily
ly in that
that ord
order,
er, in the
the natio
tional admiss
missio
ion
n tes
test
for medical schools.

I. Steve
Steve and Victor
Victor obtain
obtained
ed lower
lower scores
scores than
than Anne
Anne..
II.
II. Vict
Victor
or did
did not
not get
get 90.
90.
III.
III. Stev
teve obtain
tained90
ed90..
IV.
IV. Elai
Elaine
ne did
did not
not get
get 95.
95.

40. If Elaine
Elaine obtai
obtained
ned the lowest
lowest score
score among
among the four,
four, then
then Victor
Victor must
must have obtain
obtained
ed
(A) 95
(B) 90
(C) 86
(D) 83

41. Which
Which of the
the follow
following
ing is an accur
accurate
ate list
list of the
the scores
scores tha
thatt Victor
Victor cou
could
ld have
have obtai
obtained
ned??

(A) 95, 86
(B) 90, 86
(C) 90, 83
(D) 86, 83

42. If Victor
Victor obtai
obtained
ned the score
score of 83,
83, then
then Elaine
Elaine obtai
obtaine
ned
d _____
_______
____
__..

(A)
(B) 95
90
(C) 86
(D) 83

43. If the
the scores
scores that
that Vict
Victor
or and Anne
Anne obtaine
obtained
d were
were interc
intercha
hang
nged,
ed, which
which of the srcina
srcinall
statement
statementss would
would NO longer
longer be true?

(A)
(A) State
tateme
ment
nt I
(B)
(B) State
tateme
ment
nt II
(C)
(C) State
tateme
ment
nt III
III
(D)
(D) State
tateme
ment
nt IV

PRACTICE BOOKLET
BKL PhiLSAT_AA_0317 GO ON TO THE NEXT PAGE
25

For
For items
items 44 to 47, refer
refer to the follow
following
ing condit
condition
ions.
s.

Joe
Joe Reye
Reyess and Rach
Rachelel Cru
Cruzz will
ill be wed in a week’
eek’ss time.
time. Melis
elissa
sa Cruz
Cruz,, the
the brid
bride’
e’ss
mother
mother and
and the eldest
eldest amon
among g the Cruz
Cruz siblin
siblings,
gs, is plan
plannin
ning
g to invite
invite her
her entire
entire family
family as well
well
as that
that of
of the
the groom
groom in this exclusive
exclusive occasion
occasion..

Meli
Meliss
ssa’
a’ss par
paren
ents
ts,, Fely
ely and
and Arth
Arthurur,, are
are stil
stilll ali
alive
ve and
and can
can make
make it to the the weddi
edding
ng..
Aside
Aside from
from Melis
Melissa,
sa, this
this couple
couple has three
three other
other childr
children:
en: Micha
Michael,
el, Miche
Michelle
lle,, and
and Mann
Manny.y.
The
The youn
younge
gest
st,, Mann
Manny,
y, went
went on to beco
becomeme a Cath
Cathol olic
ic prie
priest
st and
and will
will offi
offici
ciat
atee the
the wedd
weddining
g
cele
celebr
brat
atio
ion.
n. Mich
Michae
aell and Mich
Michel
elle
le are
are both
both marr
marrie ied,
d, with
with Mich
Michaeaell havin
havingg thre
threee child
childre
ren,
n,
and Michellelle haviinng
g two
two. Mich
Michae
ael’
l’ss elde
eldestst is alre
alread
adyy marr
marrie
ied
d withith one
one chil
child,
d, whil
whilee

the other
othe
accide r two
accident.
nt. Micare
Michel
hellestill
le is singl
sstill
ingle.
sti e. Michelle’
Mich
ll living
livi elle’s
ng with
wit s youngest
young
h her est
husb
husban ofMelis
and.
d. twolissa
Me children,
child ren,
sa only
onl unfor
unsfortun
y has
ha tunate
one ately,
ly,; her
child
child; died
diedhusb
insband
hu a and
car 
died of heart
heart attack
attack just a few weeks
weeks after she has given
given birth.
birth.

On the other
other hand
hand,, Joe is alread
already
y orpha
orphaned
ned by his moth
mother.
er. His fath
father
er is still
still alive
alive and
and no
longer
longer remar
remarried
ried.. Joe has two siblings,
siblings, Jennifer
Jennifer and Jack.
Jack. Jennifer
Jennifer is happ
happily
ily married
married with two
chil
childr
dren
en who
who are
are stil
stilll in prim
primar
aryy scho
school
ol,, whil
whilee Jack
Jack is curre
current
ntly
ly deta
detain
ined
ed in the
the city
city jail
jail for a
heinous crime.

44. How
How many
many will
will be attend
attending
ing the Reyes-C
Reyes-Cruz
ruz nuptia
nuptiall includ
including
ing the primar
primary
y partic
participa
ipants
nts
(bride
(bride,, groom
groom,, and
and priest
priest)?
)?

(A) 18
(B) 19
(C) 20
(D) 21

45.
45. How
How are
are Jenn
Jennif
ifer
er and
and Melis
Melissa
sa rela
relate
ted?
d?

(A)
(A) Cousins
ins
(B)
(B) Sibli
iblin
ngs
(C)
(C) Sist
Sister
ers-
s-in
in-l
-law
aw
(D)
(D) Aunt
unt and
and niec
niecee

46.
46. How
How is Fel
Fely
y rela
relate
ted
d to Rach
Rachel
el??

(A)
(A) She is her
her aun
aunt.
t.
(B)
(B) She is her
her cou
cousi
sin.
n.
(C)
(C) She is her
her mot
mothe
her.r.
(D)
(D) She
She is her
her gra
grand
ndmomothther
er..

47. Which
Which of follow
following
ing statem
statement
entss is INCORR
INCORRECT
ECT??

(A) All
All of the Cruz
Cruz siblin
siblingsgs have
have childr
children.
en.
(B) The
The occas
occasion
ion is a religi
religiou
ouss celebr
celebrati
ation
on..
(C)
(C) Rach
Rachel
el is
is not
not the
the firs
firstt to be marr
marrieied
d amon
amongg the
the Cruz
Cruzes
es..
(D)
(D) Ther
Theree are
are some
some fami
familyly memb
member erss who
who coul
could
d not
not make
make it to
to the
the wedd
weddin
ing.
g.

PRACTICE BOOKLET
BKL PhiLSAT_AA_0317 GO ON TO
TO THE NEX
NEXT
NEXT
XT PAGE
PA
PAGE
AG
GE
E
26

For
For items
items 48 to 50, refer
refer to the follow
following
ing condit
condition
ions.
s.

A race of aliens
aliens from
from Planet
Planet X has visited
visited Earth.
Earth. Earth’s linguists
linguists attempted
attempted to decode
decode
their language:

I. Kaka
Kaka me ga shin
shing 
g     means
   “what is your name?”
II.   Ham
Hamee ne gu shin g  means
shing  mea
          ns      “where
“where do youyou live?
live?””
III.
III. They
They intr
introd
oduc
ucee them
themse selv lves es by sayi
saying
ng  Me ga shan
shang
g Krak
Krakov ov .                 
IV.
IV. When
When poin
pointi
ting
ng at thei
theirr home home plan
planet
et,, they
they say
say  Hame   na ra kong 
kong .       
V. Memu
Memu me ga shin g    means
shing          “who
“who is your father?”
father?”

48. What is the


the most likely meaning of the word  Hame ?             
likely meaning
(A) You
(B) Live
(C) Name
(D) Father 

49.
49. How
How do the
the inha
inhabi
bita
tant
ntss of Plan
Planet
et X say
say “I am
am Sam”
Sam”??

(A) Kaka
Kaka me Sam.
Sam.
(B) Sam
Sam me ga shin
shing.
g.
(C)   Me ga shan
shang Sam .           
g Sam 
(D) Memu
Memu me ga shin
shing
g Sam. Sam.

50.
50. In the
the gram
gramma
marr of
of Pla
Plane
nett X,
X, how
how does
does the
the word
word shing   function?

(A)
(B)
(B) It states
It state
poi
pointssatoname.
nts name . tion
a loca
location..
(C)
(C) It ind
indic
icat
ates
es a ques
questi
tion
on..
(D) It conne
connects
cts words
words into
into a sente
sentence
nce..

PRACTICE BOOKLET
BKL PhiLSAT_AA_0317 GO ON TO THE NEXT PAGE
27

TEST
TEST
TEST C. VERBAL
VERBA
VERBAL
L REASON
REASONING
ING

Direction
Directions:
s: This section
section contains
contains reading
reading selectio
selections
ns that
that are followed
followed by a set of
of items.
items. Answer 
Answer 
the items
items accor
accordin
dingg to what
what is stated
stated or implie
implied
d in the select
selection
ion..

Selection
Selection 1

(1) Kno
Knowin
wingg that
that Mrs.
Mrs. Malla
Mallard
rd was
was afflict
afflicted
ed with
with a heart
heart troub
trouble,
le, great
great care
care was
was taken
taken to
  break
break to her as
as gently
gently as possible
possible the news
news of her
her husband’
husband’ss death.
death. It was her
her sister,
sister, Josephin
Josephine,
e,
who
who told
told her,
her, in broken
broken senten
sentences
ces;; veil
veiled
ed hints
hints that
that reveal
revealed
ed in half
half con
concea
cealin
ling.
g. Her husband’s
friend
friend Richar
Richards
ds was
was there,
there, too,
too, near
near her.
her. It was
was he who
who had been
been in the news
newspap
paper
er office
office when
when
intelligen
intelligence
ce of the railroad
railroad disaster
disaster was received
received,, with Brently
Brently Mallard’s
Mallard’s name leading
leading the list of                 
“killed.”

(2)
(2) The
There
re stoo
stood,
d, faci
facing
ng the
the open
open wind
window
ow,, a comf
comfor
orta
tabl
ble,
e, room
roomy
y armc
armcha
hair
ir.. Into
Into this
this she
she
sank,
sank, presse
pressed
d down
down by a physic
physical
al exhau
exhausti
stion
on that
that haunt
haunted
ed her
her body
body and seeme
seemed
d to reach
reach into
into her 
her 
soul.

(3) She could


could see in the open
open squar
squaree befor
beforee her
her house
house the tops
tops of trees
trees that
that were
were all aquiv
aquiver 
er 
with
with the
the new
new spri
spring
ng life.
life. The
The deli
delici
ciou
ouss brea
breath
th of rain
rain was
was in the
the air.
air. In the
the stre
street
et below
below,, a pedd
peddle
ler r 
was crying
crying his wares
wares.. The notes
notes of a distant
distant song
song which
which someone
someone was singing
singing reached
reached her faintly,
faintly,
and countless
countless sparrows
sparrows were twittering
twittering in the eaves.
eaves.

(4)
(4) She
She sat
sat wit
withh her
her hea
head
d thro
thrown
wn back
back upon
upon the
the cush
cushio
ion
n of the
the chai
chair,
r, quit
quitee motio
motionl
nles
ess,
s,
exce
except
pt whe
when n a sob
sob came
came up into
into her
her thro
throat
at and
and shoo
shook
k her,
her, as a child
child who
who has
has cried
cried itse
itself
lf to slee
sleep
p
continue
continuess to sob in its dreams.
dreams.

(5) She
She did not
not stop
stop to ask if it
it were
were or were
were not
not a monst
monstrou
rouss joy
joy that
that held
held her.
her. A clear
clear and
and
exalted
exalted perceptio
perception
n enabled
enabled her to dismiss
dismiss the suggestio
suggestionn as trivial.
trivial.

(6)
(6) She
She knew
knew that
that she
she woul
wouldd weep
weep agai
again
n when
when she
she saw
saw the
the kind
kind,, tend
tender
er hand
handss fol
folde
dedd in
deat
death;
h; the
the face
face that
that had
had neve
neverr look
looked
ed save
save with
with love
love upon
upon her,
her, fixe
fixed
d and
and gray
gray and
and dead
dead.. A kind
kind
inten
intentio
tion
n or a cruel
cruel intent
intention
ion made
made the act seem
seem no less
less a crime
crime as she
she looked
looked upon
upon it in that
that brie
brief f                 
moment of illumination.

(7) What
What cou
could
ld love,
love, the unsol
unsolve
ved
d myster
mystery,y, coun
countt for
for in face
face of this
this posse
possessi
ssion
on of self
self--
assertion
assertion which
which she suddenly
suddenly recogniz
recognized
ed as the
the stronges
strongestt impulse
impulse of her
her being!
being!

(8) “Free!
“Free! Body
Body and soul free!”
free!”   she kept whispering.
whispering.

(9) Josep
Josephin
hinee was
was kneelin
kneeling
g befor
beforee the closed
closed door
door with her
her lips to the keyho
keyhole,
le, implor
imploring
ing
for admission
admission.. “Louise,
“Louise, open the door!
door! For
For heaven’s
heaven’s sake,
sake, open the door.”
door.”

(10) No, Louise


Louise was drinking
drinking in the very elixir
elixir of life through
through that open window.
window. Her fancy
fancy
was runnin
running g riot
riot alon
along
g those
those days
days ahea
aheadd of her.
her. Sprin
Springg days,
days, and
and summe
summerr days
days,, and
and all sorts
sorts of 
of                 
days
days tha
thatt woul
wouldd be her
her own.
own. She
She brea
breath
thed
ed a quic
quick k pray
prayerer tha
thatt life
life migh
mightt be long
long.. It was
was onl
only
y
yester
yesterda
dayy she
she had
had thoug
thought
ht with
with a shud
shudder
der that
that life might
might be long.
long.

PRACTICE BOOKLET
BKL PhiLSAT_AA_0317 GO ON
ON TO THE NEXT
NEXT PAGE
PAGE
28

(11)
(11) She
She aros
arosee at len
lengt
gth
h and
and open
opened
ed the
the door
door toto her
he r sister’s
sister’s importun
importunities.
ities. There
There was a
feveri
feverish
sh triump
triumph h in her eyes,
eyes, and
and she
she carrie
carried
d hersel
herselff unwit
unwittin
tingly
gly like
like a godd
goddess
ess of Victor
Victory.
y. She
She
clasped
clasped her sister’s
sister’s waist,
waist, and together
together they descend
descended
ed the stairs.
stairs.

(12)
(12) Someo
Someone ne was
was open
opening
ing the front
front door
door with
with a latchke
latchkey.y. It was
was Bren
Brently
tly Malla
Mallard
rd who
who
entere
entered,
d, a little
little trave
travel-s
l-stai
taine
ned,
d, compo
composed
sedly
ly carryi
carrying ng his grips
gripsack
ack and
and umbre
umbrella
lla.. When
When the doctor
doctorss
came,
came, they
they said
said she had died
died of heart
heart disea
disease
se  – of
o f the joy that
that kills.
kills.

  An excerpt
excerpt from “The Story
Story of an Hour” by Chopin, 1894.
by Kate Chopin,

1. The
The statem
statement
ent “Louis
“Louisee was
was drinking
drinking in the very
very elixir
elixir of life” is ironic
ironic becaus
becausee she found
found life
sweet knowing she

(A)
(A) gain
gained
ed cari
caring
ng,, true
true frie
friend
ndss
(B)
(B) would
ould be free
free from
from hear
hearta
tach
ches
es
(C)
(C) was
was supp
suppos
osed
ed to be in emot
emotio
iona
nall pain
pain
(D)
(D) would
would be reliev
relieveded of her
her heart
heart proble
problemm

2. The doctors’
doctors’ statement
statementss contrad
contradicted
icted the real cause
cause of Mrs.
Mrs. Mallard’
Mallard’ss death
death which
which was

(A) the appearan


appearancece of Brently
Brently who was surpr
surprised
ised at Richards’
Richards’ sudde
suddenn movement
movement
and unexpec
unexpected
ted reaction
reaction
(B)
(B) the
the disc
discov
overery
y that
that Bren
Brentl
tly
y was
was alive,
alive, a twis
twistt that
that advers
adversel
ely
y affe
affect
cted
ed her 
her 
heart condition
(C) the sad news
news about
about Brently’s
Brently’s tragic
tragic death
death which
which she found
found  unbearable
unbearable and
difficult
difficult to accept
accept
(D) Joseph
Josephine
ine’s
’s shriek
shrieking
ing cry which
which jolted
jolted Mrs.
Mrs. Malla
Mallard,
rd, causin
causing g her
her to scream
scream and
eventually
eventually collapse
collapse

3. What
What figur
figuree of speech
speech is applied
applied in the 
the  phrase “monstrous joy?”

(A) Irony
(B) Allusion
(C) Metonymy
(D) Oxymoron

4. From
From what
what point
point of view
view is the selection
selection written?
written?

(A) First
(B) Second
(C) Limited
(D) Omnisciscient

PRACTICE BOOKLET
BKL PhiLSAT_AA_0317 GO ON
ON TO THE NEXT
NEXT PAGE
PAGE
29

5. Which
Which of the follow
following
ing best
best states
states the theme
theme of the story?
story?

(A)
(A) All
All marr
marria
iage
ges,s, even
even the
the kind
kindes
estt ones
ones,, are
are inher
inheren
ently
tly oppr
oppres
essi
sive
ve..
(B) Women’s
Women’s true
true calling
calling is in their kitchens
kitchens and laundry
laundry rooms.
rooms.
(C)
(C) Wome
Women n are
are expe
expect
cted
ed to bear
bear and
and rear
rear child
childre
ren
n as dict
dictat
ated
ed by the
the soci
societ ety.
y.
(D)
(D) Inde
Indepe
pend
nden
encece can
can be imag
imaginined
ed only
only priv
privat
atel
ely
y by wome
women n beca
becaus
usee of         
dominatio
dominationn of men.

6. What
What devic
devicee did the autho
authorr use to end the story?
story?

(A)
(A) Open
Open-e
-end
nded
edne
ness
ss

(B)
(C) TComic
Comi
ragicc irelief 
relie
ronyf    
(D) Symbolism
  ______________________________________________________________________________ 

Selection
Selection 2

(1) The
The quality
quality of our
our thoughts
thoughts becom
becomes
es visible
visible through
through our words
words,, our behav
behavior,
ior, or even
even
thro
throug
ughh the
the expr
expresessi
sion
on of our
our fac
face.
e. The
The seed
seed of word
wordss and
and acti
action
onss is
is tho
thoug
ught
ht.. By know
knowining
g
and
and unde
unders
rsta
tand
ndin
ing g the
the type
typess of tho
thoug
ught
htss our
our mind
mind can
can crea
create
te,, we will
will bebe able
able to cons
consci
ciou
ousl
sly
y
redire
redirect
ct tho
though
ughts
ts tow
toward
ardss the
the posit
positive
ive.. Hence
Hence we will
will impro
improve
ve the quality
quality of our
our mind
mind,, body
body,,
and relations
relationships
hips..

(2) Necessary thoughts are those


those that
that deal
deal with our
our daily
daily routin
routinee like,
like, “What
“What will
will I eat?”;
eat?”;
“What
“What do I need
need to do today?
today?”;
”; “When
“When do I hav
havee to pay my bills?
bills?”” These
These are practi
practical
cal thoug
thought
htss to
attend
attend to our
our daily
daily needs.
needs.

(3) Wasteful
Wasteful (or superfluou
superfluous)
s) thoughts
thoughts are those
those that are of no use, neither
neither constructi
constructive
ve nor 
  particul
particularl
arly
y negat
negative
ive.. They
They deal
deal with
with things
things of
of the past,
past, like
like “If this
this had not
not happene
happened;”
d;” “If only I
have done
done it this
this way.”
way.”

(4) Negativ
Negativee thoughts
thoughts are
are most harmfu
harmful,l, especially
especially to ourselve
ourselves. s. Negative
Negative thoughts
thoughts which
which
are based
based on materi
materiali
alisti
sticc desire
desires,
s, anger,
anger, jealou
jealousysy,, hatre
hatred,
d, extre
extreme
me lazine
laziness,
ss, and
and posses
possessiv
sivene
eness
ss
cause
cause us great
great loss
loss of inner
inner peac
peacee and
and stren
strength
gth.. Such
Such thoug
thoughts
hts pois
poisonon our mind
mind as well
well as the
the
atmo
atmosp
sphe
here
re.. No matt
matter
er how
how righ
rightt we are,
are, by thin
thinki
king
ng nega
negati
tive
vely
ly,, we lose
lose beca
becaus
usee nega
negati
tive
ve
thoughts
thoughts take away
away our self-re
self-respec
spect,
t, and also,
also, others lose
lose respect
respect for
for us.
us.

(5) Positive
Positive thoughts
thoughts allow us to accumula
accumulate te inner
inner stren
strength
gth and enable
enable us to be creative
creative and
construc
constructive.
tive. To think
think positivel
positively
y doesn’t
doesn’t meanthat we ignore
ignore the reality.
reality. To think positive
positively
ly means
means
to use the proble
problemm and
and to find
find solut
solution
ionss for
for that
that probl
problem.
em.

(6) A person
person who thinks
thinks positiv
positively
ely will be aware of thethe weaknes
weaknesses
ses of
of others,
others, but will still
turn
turn his attent
attention
ion toward
towardss the
the good
good tenden
tendencie
ciess of others
others.. Posit
Positive
ive thoug
thoughts
hts give
give us a feelin
feeling
g of                 
conten
contentme
tment
nt within
within.. When
When we are conten
contentt within
within,, then
then we have
have the stren
strength
gth to accept
accept others
others as
they
they are witho
without
ut want
wanting
ing to chang
changee them
them as we think
think is righ
right.
t.

PRACTICE BOOKLET
BKL PhiLSAT_AA_0317 GO ON
ON TO THE NEXT
NEXT PAGE
PAGE
30

7. The author
author devel
developed
oped his topic by using
using all of
of the
the following
following methods
methods EXCEPT
EXCEPT

(A) narra
rratio
tion
(B) defin
finitio
ition
n
(C)
(C) enum
enumererat
atio
ion
n
(D)
(D) exem
exemplplif
ific
icat
atio
ion
n

8. In which
which parag
paragrap
raph
h is the main
main idea
idea of the essay
essay found
found??

(A) Para
aragrap
raph 1
(B) Paragraph 2

(C) Paragrap
Para
(D) Paragraph
aragraph6
raph 3

9. In parag
paragrap
raph
h 4, the unde
underlin
rlined
ed statemen
statementt “such thoughts
thoughts poison
poison our mind
mind as well as
the atmosph
atmosphere”
ere” implies
implies that negative
negative thoughts
thoughts

(A) leadto
leadto desp
espair 
air 
(B) tarnish one’s character
(C)
(C) affe
affect
ct the
the surr
surrou
ound
ndin
ings
gs
(D)
(D) have
have soci
social
al impl
implic
icat
atio
ions
ns
  ______________________________________________________________________________ 

Selection
Selection 3

Some
Some prima
primary
ry care
care provid
providers
ers may be appre
apprehe
hensi
nsive
ve about
about using
using insuli
insulin
n in patien
patients
ts with
with
type
type 2 diabet
diabetes.
es.

Concerns about hypoglycemia


hypoglycemia (diminished contents of glucose in the blood) and/or ability
to inject
inject insulin
insulin are good
good reasons
reasons why many providers
providers may approach
approach insulin
insulin therapy
therapy with caution.
caution.
Compo
Compoun undin
ding
g this
this relucta
reluctance
nce is the perce
percepti
ption
on that
that insulin
insulin therap
therapy
y is too comple
complexx to manage
manage in a
busyprimary care practic
practice;
e; prescr
prescribinginforma
ibinginformation
tion provided
provided bymanufacturer
bymanufacturerss has been somewha
somewhatt
vague
vague regardin
regardingg initial
initial dosing
dosing and titration.
titration.

Becaus
Becausee of the kind
kind of inform
informati
ation
on provi
provide
ded
d by the manufa
manufactu
cturer
rers,
s, provi
provide
ders
rs may delay
delay in
making
making the necessary
necessary transition
transition from oral agents
agents to insulin.
insulin. Indeed,
Indeed, recent
recent evidence
evidence suggests
suggests that
the
the hemo
hemogl
glob
obin
in A1c result
result that triggers
triggers glucose-lo
glucose-loweri
wering
ng action
action is ≥ 9% (Bro(Brown
wn and
and Nich
Nichol
ols,
s,
2003).
2003). This is unfortu
unfortunate
nate because
because numerou
numerouss studies
studies have shown
shown that excelle
excellent
nt glycemic
glycemic control
control
can be achiev
achieved
ed with
with insuli
insulin
n therap
therapyy in patien
patients
ts with
with type
type 2 diabet
diabetes.
es.

Subjects
Subjects of a cohort
cohort study (Epidemi
(Epidemiolog
ologyy of Diabetes
Diabetes Interve
Interventio
ntions
ns and Complica
Complication
tionss
Study),
Study), who had been intensive
intensively
ly treated
treated during
during the Diabetes
Diabetes Control
Control and Complica
Complications
tions Trial, at
seven
seven years
years show
showeded significan
significantt decrea
decreases
ses in risk for nephropa
nephropathy
thy and retinopat
retinopathy
hy compared
compared with
subjects from the conventional
conventional treatment
treatment arm.
arm.

Malmb
Malmberg
erg,, Norh
Norhamm
ammar,
ar, Wedel,
Wedel, and Ryden
Ryden (1999
(1999)) demo
demonst
nstrat
rated
ed that
that the unfav
unfavora
orable
ble
long-t
long-term
erm progn
prognosi
osiss for
for myocar
myocardia
diall infar
infarcti
ction
on (heart
(heart attack
attack)) cou
could
ld be improv
improveded by insuli
insulin
n
treatment.

PRACTICE BOOKLET
BKL PhiLSAT_AA_0317 GO ON
ON TO THE NEXT
NEXT PAGE
PAGE
31

Recent
Recent data (Ceriello
(Ceriello et al., 2004)
2004) show
show that postpra
postprandia
ndiall hyperglyc
hyperglycemia
emia is accompa
accompanied
nied
by endothe
endothelial
lial dysfunc
dysfunction
tion in patients
patients with type 2 diabetes.
diabetes.

This significan
significantt body
body of evidence
evidence strongly
strongly supports
supports the rationale
rationale for initiating
initiating therapy
therapy to
achi
achiev
evee glyc
glycem
emic
ic cont
controroll in pati
patien
ents
ts with
ith type
type 2 diab
diabet
etes
es much
much earl
earlie
ierr and
and much
much more
more
aggressively.

  An excerpt from an online


online journal article “A Real  -World
journal article                             Approach
      to Insulin             
Therapy
Therapy in Primary
Primary Care Practi
Practice”
ce” by Hirsch, Hirsch, I.B. et al., 2005.

10. The first paragrap


paragraph
h says that primary
primary care providers
providers view the use of insulin
insulin in patients
patients with
type 2 diabetes
diabetes with

(A) anger  
(B) anxiety
(C) approv
roval
(D)
(D) awar
awaren
enesesss

11. Primary
Primary care provider
providerss are reluctant
reluctant to administe
administerr insulin
insulin therapy
therapy because
because of                 
the manufacturers’
manufacturers’

(A)
(A) inabil
inability
ity to provid
providee instru
instructi
ction
onss on
on initia
initiall dos
dosing
ing and
and titrat
titration
ion
(B)
(B) incomp
incomplet
letee proced
procedure
ure regard
regarding
ing initia
initiall dosin
dosingg and
and titrat
titration
ion
(C)
(C) lack
lack of inform
informatiation
on regard
regarding
ing initia
initiall dosin
dosing g and
and titrati
titration
on
(D)
(D) fuzzy
fuzzy expla
explananatio
tionn regard
regarding
ing initia
initiall dos
dosing
ing and
and titrati
titration
on

12. Which
Which of the following
following statemen
statements
ts summarizes
summarizes the medical
medical article?
article?

(A) Insulin
Insulin therapy
therapy is very
very beneficia
beneficial.
l.
(B)
(B) Peop
People
le must
must be be consci
consciou
ouss of their
their diet
diet..
(C)
(C) Endo
Endothe
thelia
liall damag
damagee is actu
actuall
ally
y preven
preventedted by insuli
insulin
n therap
therapy.
y.
(D)
(D) Insuli
Insulin
n is consid
considere
ered
d as the most
most effecti
effectiveve treatm
treatmen
entt for diabete
diabetes.
s.

13. Which
Which of the follow
following
ing concl
conclusi
usion
onss can be made
made from
from the medica
medicall report?
report?

(A)
(A) It is risky
risky for patien
patients
ts with
with type
type 2 diabet
diabetes
es to have
have insuli
insulin
n therap
therapy.
y.
(B)
(B) It is common
common amongamong type
type 2 diabet
diabetes
es patien
patients
ts to have
have insuli
insulin
n therap
therapy.
y.
(C)
(C) It is cost
costly
ly to have
have insu
insuli
lin
n ther
therap
apyy amon
among g pati
patienents
ts with
with type
type 2 diab
diabet
etes
es..
(D)
(D) It is
is bette
betterr for
for patie
patients
nts with
with type
type 2 diabe
diabetes
tes to have
have insuli
insulin
n therap
therapyy earlie
earlier.
r.

PRACTICE BOOKLET
BKL PhiLSAT_AA_0317 GO ON
ON TO THE NEXT
NEXT PAGE
PAGE
32

Selection
Selection 4

(1) Filipi
Filipino
noss offe
offerr egg
eggss to
to Santa
Santa Clara
Clara to assure
assure good
good weath
weather
er on a partic
particula
ularr day
day.. It is
intere
interesti
sting
ng to contra
contrast
st that
that with the rain
rain dance
dance of the
the Nativ
Nativee Ameri
America
cans
ns to induce
induce rain.
rain. Why
Why is this
this
so? To be able
able to answ
answer er this,
this, it is nece
necessa
ssary
ry to unde
underst
rstand
and how
how rains
rains hav
havee shape
shaped
d our
our cultu
culture.
re.

(2) The import


importanance
ce of rains
rains in Philip
Philippin
pinee cultur
culturee is atteste
attested
d by the rich
rich vocab
vocabulaulary
ry relate
related
d
to prec
precip
ipit
itat
atio
ion.
n. In Taga
Tagalolog,
g, ther
theree are
are term
termss such
such as ambon   (rain
(rain show
shower)
er),, ulan   (rain), bagyo  
(typhoon), unos     (squall), sigwa   (tempest),
(tempest), and siyam-siyam   (incessan
(incessantt rains).
rains). Other
Other Philipp
Philippine
ine
langu
language
agess have
have more
more elabo
elaborat
ratee terms.
terms. For
For examp
example,
le, the Ivatan
Ivatan of Batan
Bataneses distin
distingu
guish
ish differ
different
ent
types of
of typhoon,
typhoon, e.g. salawsaw     (wind
(windyy day
day with
with rain),
rain), nisu   (windy
(windy day without
without rain),
rain), adipogpog  

(torna
(tornado
and do
then
the with
wit
n back
ba h again
ck rain),
rain),
aga dipanchi 
in to rain),     (hea
rain), anin ( heavyvy rain
rastrong
  (very in with
wigthwind
stron stro
strong
ng wind
wicauses
that ndses
cau that
thatdestruct
sudd
sudden
dest enly
ly chan
chand
ruction),
ion), ange
gess to du
anin a sunn
sunny
y day
day            
vanveyeh 
(winds
(winds strong
strong enough
enough to destroy
destroy coconutcoconut trees).
trees). This elaborate
elaborate typhoon
typhoon terminolo
terminology
gy is not at all
surprisin
surprising
g since
since 13 out of the
the 20 typhoon
typhoonss that
that hit the Philippin
Philippines
es annua
annually
lly pass through
through Batanes.
Batanes.

(3) Sev
Severa
erall Filipi
Filipino
no cultu
cultural
ral trai
traits
ts have
have develo
developeped
d as an
an adapta
adaptatio
tion
n to the presen presencece of a
long,
long, distin
distinct
ct rainy
ainy season
season.. For
For examp
example,le, the struc
structur
turee of the bahay kubo  (native
(nati
            ve hut) has
has been
been
design
designed
ed to weath
weatherer heavy
heavy rains.
rains. The roof
roof is steep
steeply
ly struct
structure
uredd so as to allow
allow rainrain to fallfall freel
freely
y to
the ground
ground.. There
There are awnin
awning g windo
windowsws that
that prote
protect
ct the
the inner
inner part
part of the
the house
house from from gettin
gettingg wet.
wet.
The
The hous
housee is buil
builtt on stil
stilts
ts to prev
preven
entt flood
flood wate
waters
rs from
from reac
reachi
hing
ng the
the hous
housee floor
floor.. On the the othe
other r 
hand,
hand, the Ivatan
Ivatan house
house  —made
made of ston
stonee walls
walls and
and roofe
roofed d with
with thick 
thick  cogon   thatch 
thatch —is a hous housee that
that
can withstan
withstand d typhoon
typhoons. s. Unfortun
Unfortunately
ately,, modern
modern houses have have lost theserain-adapted
theserain-adapted characterist
characteristics
ics
and this is why they have
have become
become vulnerab
vulnerable le to destructio
destructionn brought
brought about
about by floods
floods and and typhoon
typhoons. s.

(4) Cult
Cultura
urally
lly,, rains
rains are
are also
also percei
perceive
ved
d as being
being laden
laden with
with certai
certain
n symbo
symbolic
lic meanin
meanings
gs.. A
little
little amoun
amountt of rain durin
during g a weddi
weddingng ceremo
ceremony
ny is consid
considere
ered
d as a blessi
blessing
ng from
from the heave
heavens.
ns. On

the contra
contrary,
ry, heavy
heavy rains
rains are frown
frowned
ed upon
upon since
since they
they bring
bring abou
aboutt flood
floodss and destr
destruc
uctio
tion.
n.
(5) Gettin
Getting g wet
wet in the rain
rain is belie
believed
ved to cause
cause certai
certain
n illnes
illnesse
ses.
s. There
There is a common
common belie
belief f                 
that
that one
one catc
catche
hess a cold
cold beca
becaus
usee he or she
she got
got dren
drenchched
ed in the rain ( naulanan )  . An excep
the rain ceptio
tion,
howeve
however,
r, is when
when one
one takes
takes a show
showerer from
from the first
first rain
rain durin
during
g the month
month of May.
May.

(6) Anot
Anothe
herr popula
popularr devoti
devotion
on among
among Filipi
Filipinonoss is the offeri
offeringng of eggs
eggs to Santa
Santa Clara
Clara to
  bring
bring about
about good
good weather.
weather. One possible
possible explanatio
xplanation n for associati
associating
ng the saint
saint with
with good
good weather
weather is
her name,
name, Clara,
Clara, which
which means
means “cle
“clear”
ar” in Spani
Spanishsh.. Becau
Because se of the
the   offering
offering,, the weather
weather becom
becomeses
clear
clear even
even durin
duringg the rainy
rainy season
season.. Moreo
Moreovever,
r, the egg
egg white,
hite, in Spani
Spanish sh is called
called “clara de huevo ”.           
The
The vene
venera
rati
tion
on to Sant
Santaa Clar
Claraa was
was intr
introd
oduc
uced
ed by thethe Span
Spanis
ish
h fria
friarsrs,, she
she bein
being
g one
one of the
the sain
saints
ts
introduc
introduced
ed early to Filipinos
Filipinos..

  Adapted
Adapted from “Ulan,
“Ulan, unos,
unos, atbp.”
atbp.” By N. T. Castro
Castro..
  Retrieved from http://mb.com.ph.

14. To which
which part
part of
of the
the house
house does
does the under
underlin
lined
ed word
word “thatc
“thatch”
h” in parag
paragrap
raph
h 3 refer?
refer?

(A) Roof                  
(B) Walls
(C) Floor  
(D) Wind
indows

PRACTICE BOOKLET
BKL PhiLSAT_AA_0317 GO ON
ON TO THE NEXT
NEXT PAGE
PAGE
33

15. What
What is the chara
characte
cteris
ristic
tic of a house
house on stilts
stilts??

(A)
(A) It is built
built on wate
water.r.
(B)
(B) It con
consi
sist
stss of two
two floo
floors
rs..
(C)
(C) It has
has a conc
concrerete
te base
baseme
ment nt..
(D)
(D) It is rais
raised
ed from
from the
the grou
groundnd..

16. What
What do the
the rain
rain danc
dancee and
and the
the devo
devoti
tion
on to Sant
Santaa Clar
Claraa shar
sharee in comm
common
on??

(A)
(A) Both
Both are
are aski
asking
ng for
for rai
rain.
n.
(B)
(B) Both
Both are pre-co
pre-colon
lonial
ial pract
practice
ices.
s.

(C)
(D) Both
Both
th impl
(D) Both
Bo are
areply
im done
do
y ne
theduri
thedu
bering
ngf athat
belie
lief dry
drtyrain
tha spel
spnell.
rai l. cont
is contro
roll
lled
ed by a high
higher
er bein
being.
g.

17. Which
Which of the following
following is conside
considered
red a welcome
welcome occurren
occurrence?
ce?

(A)
(A) Havi
Having
ng a down
downpopour
ur duri
during
ng a wedd
weddin
ing
g
(B)
(B) Gett
Gettin
ing
g wet
wet in the
the firs
firstt rain
rain of May
May
(C)
(C) Getti
etting
ng dren
drench
ched
ed in a heav
heavy y rain
rain
(D)
(D) Gett
Gettin
ing
g caug
caught
ht in a ligh
lightt dri
drizz
zzle
le

18. What
What does the use of different
different express
expressions
ions for
for rain in the Philippin
Philippinee languag
languages
es suggests?
suggests?

(A)
(A) The
The richn
richness
ess and
and sophis
sophistic
ticati
ation
on of Philip
Philippin
pinee langu
languag
ages
es
(B)
(B) The
The differ
differen
ences
ces among
among the Philip
Philippin
pinee langua
languages
ges
(C)
(C) The
The change
changess in the preva
prevailin
iling
g weath
weather er condit
condition
ionss
(D)
(D) The
The infl
influe
uenc
ncee of forei
foreign
gn lang
langua
uage
gess on ours
ours

  ______________________________________________________________________________ 
Selection
Selection 5

We all kno
knoww that
that the norma
normall human
human daily
daily cycle
cycle of activ
activity
ity is some
some seven
seven to eight
eight hours
hours of 
of                 
sleep alternating
alternating with some sixteen
sixteen to seventeen
seventeen hours
hours of wakefuln
wakefulness
ess and that, broadly
broadly speaking
speaking,,
sleep normally
normally coincide
coincidess with the hours
hours of darkness
darkness.. Our present
present concern
concern is with how easily
easily and
to what
what exten
extentt this
this cycle
cycle can be modified
modified..

The
The ease
ease,, for
for exam
exampl
ple,
e, with
with whic
whichh peop
peoplele can
can chan
changege from
from work
workining
g in the
the day
day to work
workin
ing
g
at nigh
nightt is a questi
questionon of grow
growing
ing import
importanc
ancee in indust
industryry where
where autom
automati
ation
on calls
calls insis
insisten
tently
tly for 
round-
round-the
the-cl
-cloc
ockk worki
working
ng night.
night. Unfor
Unfortun
tunate
ately,
ly, it is often the case
case in indus
industry
try that
that the eight-
eight-ho
hour 
ur 
work
work shifts
shifts are
are chang
changeded every
every week
week.. This
This mean
meanss that
that no soone
soonerr has he
he gotte
gottenn used
used to one routin
routinee
than
than he has
has to chang
changee to anoth
another,
er, so that
that much
much of his his time
time is spe
spent
nt neit
neithe
herr worki
working
ng nor
nor sleep
sleeping
ing
very efficientl
efficiently.
y.

One
One answe
answerr would
would seem
seem to be longe
longerr period
periodss on each
each shift,
shift, a month
month,, or even
even three
three month
months.
s.
Resear
Researchch by Bonje
Bonjerr (1960)
(1960) of the Nethe
Netherla
rland
nds,
s, howe
howevever,
r, has
has shown
shown that
that peop
peoplele on such
such syste
systems
ms
will revert
revert to their normal
normal habits
habits of sleep and wakefuln
wakefulnessess during
during the weekend
weekend and that this is quite
quite
enoug
enough h to destr
destroy
oy any
any adap
adaptat
tation
ion to night
night work
work built
built up durin
duringg the week
week..

PRACTICE BOOKLET
BKL PhiLSAT_AA_0317 GO ON
ON TO THE NEXT
NEXT PAGE
PAGE
34

The
The only
only real
real solu
soluti
tion
on appe
appear
arss to
to be the
the hand
handin
ing
g over
over of the
the nigh
nightt shi
shift
ft to a corp
corpss of 
of                 
permanent night workers
workers whose nocturnal wakefulness
wakefulness may persist through all weekends weekends andand
holida
holidays
ys.. An intere
interesti
sting
ng study
study of the dome
domesti
sticc life
life and healt
healthh of night
night shift
shift worke
workers rs was
was carried
carried
out
out by Brow
Brown n in 1957
1957.. She foun
foundd a high
high inci
incide
denc
ncee of dist
distur
urbe
bed
d slee
sleep,
p, dige
digest
stiv
ivee diso
disord
rder
er,, and
and
domest
domestic
ic disrup
disruptio
tion
n among
among those
those on altern
alternati
ating
ng day andand night
night shifts
shifts,, but
but no abno
abnorma
rmall occur
occurren
rence
ce
of these
these symptoms
symptoms among
among those
those on permanen
permanentt night work.
work.

This latter
latter syste
systemm then appears
appears to be the best long-term
long-term policy,
policy, but meanwhile,
meanwhile, somethin
something g
may
may be done
done to reli
reliev
evee the
the stra
strain
inss of altern
alternat
atee day
day and
and nigh
nightt work
work by sele
select
ctin
ing
g thos
thosee peop
people
le who
who
can adapt
adapt most
most quick
quickly
ly to the chang
changeses of routin
routine.
e. One
One way
way of know
knowing
ing when
when a perso
person
n has
has adapted
adapted
is by measur
measuring
ing his perfo
performa
rmancnce,
e, but
but this
this can be labori
laboriou
ous.
s. An easie
easierr way
way is to take
take a perso
person’s
n’s

  body temperatu
temp
in normal
norm erature
re at
al daytime
daytime intervals
inter
work
wor valshave
k will of
of two hours
hou
high rs through
thro
temperatu
temp ughout
reout
erature durithe
ng period
during period of
rs wakefuln
the hours
hou wak efulness.
ess.
of wakeful
wakefulness
nessPeople
Peopleaengaged
and engaged
low one
at nigh
night.
t. When
When they
they chang
changee to night
night work,
work, the patter
pattern
n will
will only
only gradu
graduall
ally
y revers
reversee to match
match the new
new
routine
routine and the speed
speed with which
which it does
does parallels
parallels the adaptatio
adaptation n of the bodyas a whole,
whole, particula
particularly
rly
in terms
terms of perfor
performan
mancece and gener
general
al alertne
alertness.
ss. Know
Knowledledge
ge of how quick
quicklyly a perso
personn can adapt
adapt to a
revers
reversed
ed routin
routinee could
could be used
used as a basis
basis for select
selection
ion.. So far,
far, howe
however
ver,, such
such a form
form of select
selection
ion
does
does not
not seem
seem to have
have been
been applie
applied
d in practi
practice.
ce.

19. Body
Body tempe
temperat
rature
ure can be used
used as a physi
physiolo
ologic
gical
al measur
measuree of perfo
performa
rmanc
ncee to indica
indicate
te

(A)
(A) effi
effici
cien
ency
cy in work 
work 
(B)
(B) modi
modifified
ed slee
sleepi
ping
ng habi
habits
ts
(C)
(C) adap
adapta
tati
tion
on to work
work sche
schedu
dule
le
(D)
(D) endu
endura
ranc
ncee for
for nigh
nightt
ttim
imee work 
work 

20. The
The autho
authorr sug
sugges
gests
ts that
that the
the soluti
solution
on to proble
problems
ms arisin
arising
g from
from nightt
nighttime
ime work
work sched
schedule
ule

shou
should
ld be adopte
adopted
d by compa
companie
niess on a
(A)
(A) long
long-t
-ter
erm
m basi
basiss
(B)
(B) shor
short-
t-te
termbas
rmbasisis
(C)
(C) trial-
trial-and
and-er
-error
ror basis
basis
(D)
(D) comb
combin ined
ed shor
shortt and
and long
long-t
-ter
erm
m basi
basiss

21. The
The cycle
cycle of sleep
sleep can be modifi
modified
ed extens
extensive
ively
ly and succe
successf
ssfull
ully
y by

(A)
(A) alte
altern
rnat
atin
ing
g shift
shiftss ever
every
y week
week and
and week
weeken
endd
(B)
(B) alte
altern
rnat
atin
ing
g day
day and
and nigh
nightt work
work sche
schedu
dule
less with
withinin the
the week 
week 
(C)
(C) assi
assign
gnin
ingg a mont
month h or mor
moree of eac
eachh work
work shif
shiftt or sche
schedu
dule
le
(D)
(D) chang
changing
ing the work
work schedu
schedules
les of the day
day and
and night
night shift
shift work
workers
ers

PRACTICE BOOKLET
BKL PhiLSAT_AA_0317 GO ON
ON TO THE NEXT
NEXT PAGE
PAGE
35

Selection
Selection 6

Trav
Travel
elin
ing
g to us from
from the
the burn
burnin
ingg hear
heartt of the sun
sun 93 mill
millio
ion
n mile
miless away,
away, light
light not
not only
only
illuminate
illuminatess our
our little planet
planet but
but create
createss on
on its thin surface
surface the essentials
essentials of life itself.
itself. No wonder 
wonder 
men from
from ancien
ancientt times,
times, awed
awed by its glor
gloriou
iouss light,
light, worsh
worshipe
ipedd the sun
sun for cent
centuri
uries.
es. Its wors
worship
hip
spran
sprang g from
from true
true intuit
intuition
ion since
since in the light
light that
that comes
comes fro
from
m this
this nuclea
nuclearr ball
ball of fire the secret
secret of 
of                 
life on this
this plane
planett does,
does, in fact,
fact, dwell.
dwell. All life
life is the
the offspr
offspring
ing of the
the sun,
sun, for the
the light
light pouri
pouring
ng from
from
it is a wonderf
wonderfulul chemi
chemist.
st. Leaves
Leaves are chemical
chemical factories
factories in which
which sunlight
sunlight and water
water are
are turned
turned
into
into food
food and
and from
from which
which life-g
life-givi
iving
ng oxygen
oxygen is poure
poured d into
into the air for
for man
man to breath
breathe.
e.

Light
Light m
moves
oves to us through
through the atmosphe
atmosphere
re in waves
waves of different
different lengths,
lengths, each carrying
carrying its

own
ow
than
than color.
color.
let,, The
n violet
vio Th
noeneeyes
none eyesger
seethan
longer
lon only
onlyred.
than a small
redsma llhtsegme
. Light
Lig seg
fament
lls nt
falls of the
from
from this
thissky
great
great range
rang
e theepetal
like
lik oftals
pe wavel
was veleng
engths
thss;– every
of flower
flo wers; none
non
ev e drop
ery shorter 
shor
dropter 
of                 
dew is a glory
glory in miniat
miniature
ure and we who
who live
live by the blessi
blessing
ng of this
this distan
distantt star
star can learn
learn to rejoic
rejoicee
in it.
it.

Anyo
Anyone ne can
can see
see how
how whit
whitee brea
breaks
ks into
into colo
colorr by hold
holdin
ingg a refr
refrac
acti
ting
ng pris
prismm in a narr
narrow
ow
beam
beam of sunli
sunligh
ghtt in a darke
darkenened
d room
room and
and projec
projectin
ting g the result
result onto
onto a white
white scree
screen.
n. There
There you
you will
will
discover
discover the band of colors
colors that
that we call the solar
solar spectru
spectrum.m. Similarly
Similarly,, rainbows
rainbows are caused
caused by the
refraction
refraction and reflection
reflection of sunlight
sunlight from the raindrops
raindrops of a vanishin
vanishingg storm.
storm. Today
Today we know
know more
about
about light
light than
than our
our ancest
ancestors
ors did.
did. Our
Our scient
scientist
istss know
know enoug
enough h to use the secret
secretss of the
the sun
sun as a
stepp
stepping
ing-st
-ston
onee to the stars
stars or
or as the
the end
end of life on earth
earth..

22. The sun emits life-givin


life-giving
g light which
which travels
travels through
through space
space in

(A)
(A) equal
equal wave
wavelen
length
gthss
(B)
(B) even
evenly
ly colo
colore
red
d wave
wavess

(C)
(D) spurts
(D) vario
va uslike
rious gusts
wavel
wavelen of wind
ength
gths
s and colors
colors

23. The
The autho
authorr seems
seems to consi
consider
der the
the sun
sun as a source
source of                 

(A)
(A) natur
natural
al calam
calamitie
itiess like
like droug
drought
ht
(B)
(B) ener
energy
gy and
and beau
beauty ty that
that man
man can
can enjo
enjoy
y
(C)
(C) threat
threat to life
life becau
because se of its nucle
nuclear
ar powe
power r 
(D)
(D) nucle
nuclear
ar powe
powerr that
that is bein
being
g waste
wasted
d in space
space

24. As a whole
whole,, the purpo
purpose
se of the select
selection
ion is to
to

(A) describe
describe man’s
man’s ancient
ancient worship
worship of the sun
(B)
(B) prai
praise
se the
the beau
beauty
ty of the
the sun
sun and
and its impo
import
rtan
ance
ce to life
life
(C)
(C) state
state the possi
possibil
biliti
ities
es of
of the
the tremen
tremendo
dous
us pow
powerer of
of the
the sun
sun
(D)
(D) discus
discusss the
the effort
effortss of
of man
man to discov
discover
er the secret
secretss of
of the sun
sun

PRACTICE BOOKLET
BKL PhiLSAT_AA_0317 GO ON
ON TO THE NEXT
NEXT PAGE
PAGE
36

Selection
Selection 7

I was
was born
born in a log
log cabi
cabinn on a wint
winterer.. The
The firs
firstt thi
thing
ng I rem
remem
embe berr is
is bei
being
ng grat
gratef
eful
ul for 
for 
wind
window
ows.s. I was thr
three
ee year
yearss old.
old. My moth
mother
er had
had set
set me to play
play on a matt
mattre
ress
ss care
carefu
fully
lly plac
placed
ed in
the one
one ray of sun
sunlig
light
ht stre
stream
aming
ing throu
through
gh the one
one glass
glass wind
window ow of our
our log
log cabin.
cabin. Baby
Baby as I was,
was, I
hadachedin the agonizi
agonizingng cold
cold of a pione
pioneer
er winter.
winter. Lyingthere
Lyingthere,, warme
warmed d by that
that blesse
blessed
d sunsh
sunshine
ine,,
I was suddenly
suddenly aware
aware of wonde
wonderr and joy and gratitude
gratitude.. It was
was gratitud
gratitudee for glass,
glass, which
which could
could keep
out
out the biti
biting
ng cold
cold and
and let
let in the
the warm
warm sun.
sun...
..

My father
father came
came from
from a family
family of schoo
schooll teache
teachers
rs in New
New Englan
England.
d. My mothe
motherr was
was the
daughter
daughter of a hardwork
hardworking
ing Scotch immigrant
immigrant.. Father’s
Father’s family
family set store
store on ancestry.
ancestry. Mother’s
Mother’s side

was more practical.


practical...
..
The year
year befo
before
re my birth,
birth, these
these two youn
young
g peop
people
le had
had starte
started
d West
West in a prairie
prairie schoo
schoone
nerr to
stake
stake a home
homeste
stead
ad claim.
claim...
..

Afte
Afterr moth
mother
er’s
’s eig
eight
hth
h and
and last
last bab
baby,
y, she
she lay
lay ill for
for a year
year.. The
The care
care of the
the chil
childr
dren
en fell
fell
  princip
principall
ally
y on my youn
youngg should
shoulders
ers.. One
One day I found
found her cryi
crying
ng..

“Mary
“Mary,”,” she said,
said, with
with a tende
tenderne
rness
ss that
that was
was rare
rare,, “If I die,
die, you
you must
must take
take care
care of all
all your 
your 
  brothe
brothers
rs and
and sister
sisters.
s. You
You will
will be the only
only woman
woman within
within eighte
eighteenen miles
miles.”
.”

I was ten years old.


old.

That night
night and
and many
many other
other nights
nights I lay awake,
awake, trembling
trembling at the
the possibilit
possibility
y of being
being left the
only
only woma
woman
n within
within eighte
eighteen
en miles.
miles.

But
Butead
his homes t moth
mother
homestead erner
didcould
partner
part cnot
not die.
oulddie. I e,
must
must
spare,
spar have
hav
I kept
kep e been
bee
t that n a esturdy
home
hom sturdygchild
going
goin child,, she
until for,
for, was
with
withstrong
the little
little
strong help
help fathe
again...
agai n... fatherr and
and

Every
Every fall, the shoemake
shoemakerr made his rounds
rounds through
through the country,
country, reaching
reaching our place
place last, for 
  bey
beyon
ond d us lay only
only untame
untamed d forest
forest and wild
wild beast
beasts.
s. His visit
visit thril
thrilled
led us more
more than
than the arriva
arrivall of                 
any
any king
king toda
today.
y. We hadhad been
been cut
cut off from
from the
the worl
world d for
for month
months. s. The
The shoe
shoema
make
kerr broug
brought
ht news
news
from
from neigh
neighbo bors
rs eighte
eighteen,
en, forty,
forty, sixty,
sixty, even
even a hund
hundred
red and fifty
fifty miles
miles away.
away. Usual
Usuallyly he broug
brought
ht a
few newsp
newspapeapersrs too,
too, treasu
treasured
red afterw
afterward
ard for mont
months
hs.. He remain
remaineded a royal
royal guest
guest,, for many
many days,
days,
until all
all the family
family was shod..
shod....

By the time I was


was fourteen
fourteen,, three
three tremendo
tremendous
us events
events had marked
marked my life: sunlight
sunlight through
through
a windo
windowp
wpane
ane,, the log-ro
log-rolli
lling
ng on the river
river when fathe
fatherr added
added tworooms
tworooms to our
our cabin,
cabin, andthe night
night
I thoug
thought
ht mothe
motherr would
would die and leave
leave me the only
only woman
woman within
within eighte
eighteen
en miles.
miles...
..

But there
there was a fourth
fourth event
event that
that was
was the most tremendo
tremendous.us. One night
night father
father handed
handed my
mother
mother a letter
letter.. Our
Our Great
Great-Au
-Aunt nt Mart
Marthaha had
had willed
willed fathe
fatherr her
her house
househol
holdd good
goodss and
and person
personal
al
belo
belong
ngin
ings
gs and
and a mode
modestst sum
sum that
that to
to us was
was a fort
fortun
une.e. Some
Someononee back
back East
East “aw
“awai
aite
ted
d his
his
instru
instructi
ction
ons.”
s.” Many
Many discus
discussio
sions
ns followe
followed,d, but in the end my mothe
motherr gaine
gainedd her way.
way. Great
Great-Aunt
-Aunt
Marth
Martha’s
a’s househ
household old good
goodss were
were sold
sold at aucti
auction
on.. Fathe
Father,
r, howe
however
ver,, insist
insisted
ed that
that her “per
“perso
sonal
nal
  belong
belonging
ings”
s” be shipp
shipped
ed to us...
us...

PRACTICE BOOKLET
BKL PhiLSAT_AA_0317 GO ON
ON TO THE NEXT
NEXT PAGE
PAGE
37

Afte
Afterr a long
long,, long
long wait
wait,, one
one day
day a trun
trunkk and
and two
two pack
packin
ingg case
casess cam
came.
e. It was
was a sole
solemn
mn
moment
moment when the first box was opened.
opened. Then mother
mother gave
gave a cry of delight.
delight. Sheets
Sheets and bedspre
bedspreads
ads
edged
edged with lace!
lace! Real
Real linen
linen pillowcas
pillowcases
es with
with crochete
crocheted
d edgings
edgings.. Soft woolen
woolen blankets
blankets and bright
bright
handmade
handmade quilts.
quilts. Two heavy,
heavy, lustrous
lustrous tablecloth
tableclothss and two dozen
dozen napkins,
napkins, one white
white set hemmed,
hemmed,
and one
one red-an
red-and-w
d-whit
hite,
e, borde
bordered
red with
with a soft
soft fring
fringe.
e.

What
What the
the world
world calls
calls wealt
wealth
h has
has come
come to me in years.
years. Nothi
Nothing
ng ever
ever equal
equaled
ed in my eyes
eyes the
the
  priceless value
value of Great-Aunt
Great-Aunt Martha’s “personal belongings.”

I was
was in a seve
sevent
nth
h heav
heaven
en of delig
delight
ht.. My fath
fatherer pick
picked
ed up the
the book
bookss and
and bega
begann to read
read,,
  paying
paying no attention
attention to our excla
exclamatio
mations
ns over
over dresse
dressess and
and ribbons,
ribbons, the boxful
boxful of laces,
laces, or the little

shell-c
shell-cov
overe
ered
d case
case holdi
holding
ng a few orname
ornaments
nts in gold
gold and silve
silverr and jet.
jet. Then
Then I picked
picked up a napkin
napkin..
“What
“What are these
these for?”
for?” I asked
asked curiously
curiously..

My fathe
fatherr slam
slammed
med his book
book shut.
shut. I had
had never
never seen
seen such
such a look
look on his face.
face.

“How
“How old are you,
you, Mary?
Mary?”” he deman
demanded
ded sudde
suddenly
nly..

I told him that


that I was going
going on fiftee
fifteen.
n.

“And
“And you
you never
never saw a table
table napkin
napkin?”
?”

His tone was bitter


bitter and accusing
accusing.. I didn’t understan
understand—
d—ho
how
w coul
could
d I?

Fathe
Fatherr bega
began
n to talk,
talk, his words
words growin
growingg more
more and more
more bitter
bitter.. Mothe
Motherr defe
defend
nded
ed herse
herself 
lf                 
hotly.
hotly. Today,
Today, I know
know that justice
justice was on
on her side.
side. But in that first
first adoles
adolescent
cent self-c
self-consc
onscious
iousness
ness,,

my sympathie
sympathiess were
were all with
with father.
father.
Mothe
Motherr had neglec
neglected
ted us 
us —she had not
not taugh
taughtt us to use
use table
table napk
napkins
ins!! Peop
People
le in histor
history
y
used
used them.
them.

From
From that
that time
time on,
on, we used
used napki
napkins
ns and a tablec
tableclot
loth
h on Sund
Sundays
ays..

  An excer
excerpt from “ The
pt from                 Log-     
                Cabin Lady: An Anonymous Autobiography”      by
        Marie
                    Mattingly
  Meloney                   

25. What
What was the
the proof
proof that Mary
Mary was a healthy
healthy ten-year
ten-year old?
old?

(A)
(A) She
She lear
learne
nedd to hunt
hunt for
for food
food..
(B)
(B) She
She neve
neverr got
got sic
sick
k in spit
spitee of the
the hard
hardsh
ship
ips.
s.
(C)
(C) She
She helpe
helpedd her father
father cut the logs
logs for
for their
their cab
cabin.
in.
(D)
(D) She
She ran
ran the
the hous
househehol
old
d when
when herher mothe
motherr was
was sick.
sick.

26. Who
Who is tellin
telling
g the story?
story?

(A) The father 


(B) The mother 
(C) The sho
shoemak
maker 
(D)
(D) The
The elde
eldest
st daug
daught
hter 
er 

PRACTICE BOOKLET
BKL PhiLSAT_AA_0317 GO ON
ON TO THE NEXT
NEXT PAGE
PAGE
38

27. What
What experien
experience
ce made the greatest impact
impact on Mary’s life?

(A)
(A) Roll
Rollin
ingg logs
logs on
on the
the river
river wit
withh her
her fath
father 
er 
(B)
(B) The
The fear
fear of
of bein
being g orph
orphananed
ed by her
her moth
mother 
er 
(C)
(C) Inhe
Inheri
ritin
ting
g thei
theirr grea
great-t-aunt’s
aunt’s personal belongings
(D)
(D) The
The warmt
warmth h of sun
sunlig
light
ht stre
stream
aming
ing throu
through
gh the glass
glass wind
window
ow

28. Besides making


making their shoes, why was the shoemake
shoemaker’s
r’s visit eagerly
eagerly awaited?
awaited?

(A)
(A) He help
helped
ed in clea
cleari
ring
ng the
the land
land..
(B)
(B) He broug
brought
ht supp
supplie
liess for
for the family
family..

(C)
(D) He brou
(D) He enter
enough
br tertai
taine
ghttned
d the
news
new child
chm
s from
fro ildren
thren
the with
wit
e outs
ou h ehis
tsid
ide tricks
tri
worl
wo cks.
d. .
rld.

29. What does the statemen


statement,
t, “He
“He remain
remained
ed a royal
royal guest
guest”” mean?

(A)
(A) He was
was a give
giverr of gift
gifts.
s.
(B)
(B) He was
was a loya
loyall sub
subje
ject
ct of the
the king
king..
(C)
(C) He rece
receiv
ived
ed the
the best
best trea
treatm
tmenentt from
from the
the peop
people
le..
(D)
(D) He was
was know
knownn to ever
everyb
ybod
ody y in the
the comm
commununity
ity..

30. How
How many
many siblin
siblings
gs did Mary
Mary have?
have?

(A) Seven
(B) Eight
(C) Nine
(D) Ten

31. What
What triggered
triggered the bitter
bitter argumen
argumentt between
between husband
husband and wife?

(A) The wife’s


wife’s decis
decision
ion to sell great
great-aunt Martha’s household
household goods
(B) The wife’s
wife’s insistenc
insistencee on using
using table napkins
napkins on Sundays
Sundays only
(C)
(C) The
The real
realiz
izat
atio
ion
n that
that the
the child
childre
ren
n did
did not
not have
have tabl
tablee mann
manner
erss
(D) The
The wife’s
wife’s failu
failure
re to teach
teach her child
children
ren ta 
ta ble manners
  ______________________________________________________________________________ 

Selection
Selection 8

During
During the 1970’s,
1970’s, experts
experts predictedthat
predictedthat the world
world populati
population
on of three
three and one-thirdbillion
one-thirdbillion
woul
would
d doub
doublele in the
the next
next thi
thirt
rty
y year
years.
s. Acco
Accord
rdin
ing
g to Dr.
Dr. Paul
aul Erli
Erlich
ch,, a scie
scient
ntis
istt at Sta
Stanf
nfor
ord
d
Unive
Universi
rsity,
ty, popu
populat
lation
ion explos
explosion
ion was
was causing
causing mankin
mankindd “…to
“…to run out
out of our food resou
resource
rces,
s, and
we were destroyi
destroying
ng the ecologica
ecologicall system
system of our planet
planet which
which keeps
keeps us alive.
alive.””

In the PhilipPhilippin
pines
es,, 3,295
3,295 babies
babies were
were being
being added
added to our
our popula
populatio
tion
n every
every day,
day, that
that is, two
  babi
babies
es were
were beinbeing g born
born ever
every
y one-
one-ha
half
lf minut
minute,
e, repo
report
rted
ed Dr. Juan
Juan Salce
alcedo
do Jr.,
Jr., who was
was then
then
Chairm
Chairmanan of      the
  National
National Scien
Science
ce Develop
Development
ment Board.
Board. Our country
country’s
’s yearly
yearly populati
population
on growth
growth
of 3.5
3.5 perce
percent
nt was was con
consid
sidere
eredd to be one
one of the
the highe
highest
st in the world
world..

PRACTICE BOOKLET
BKL PhiLSAT_AA_0317 GO ON
ON TO THE NEXT
NEXT PAGE
PAGE
39

An inte
intere
rest
stin
ing
g aspe
aspect
ct of
of the
the popu
popula
latio
tionn issu
issuee was
was brou
broughghtt out
out by Dr.
Dr. Merc
Mercededes
es B.
B.
Concepci
Concepcion,on, who was then
then Director
Director of the
the Populati
Population
on Institute
Institute of the Universit
Universityy of the Philippin
Philippines.
es.
She reported
reported that almos
almostt fifty percent
percent of
of the Filipinos
Filipinos were below
below fifteen
fifteen years
years of age.
age. Mr. Alfredo
Alfredo
R. Roces,
Roces, columnis
columnistt of the Manila
Manila Times,
Times, predicted
predicted that the vast majority
majority of our society
society would,
would, in
the near
near futur
future,
e, be compr
compriseisedd of teena
teenager
gers.
s. In conne
connecti
ction
on with
with this,
this, one
one aspect
aspect that
that neede
needed d close
close
stud
study,
y, acco
accordrdin
ingg to Dr.
Dr. Conc
Concepepci
cion
on,, was thethe omin
ominou ouss repe
repercrcus
ussi
sion
on whic
whichh may
may upse
upsett our 
our 
establishe
establishedd economi
economicc structure
structure because
because of a very young
young populati
population.
on. Also noted
noted by her was that the
rapid
rapid growth
growth of the popu
populat
lation
ion here
here had
had broug
broughtht about
about a heavy
heavy depen
depende dency
ncy rate
rate among
among Filipi
Filipino
nos.
s.
She
She stated
stated that
that there
there were
were about
about eighty
eighty-ni
-nine
ne child
child depen
dependen
dents
ts for every
every one
one hundr
hundred
ed worke
workersrs in
this
this cou
count
ntry
ry.. Furt
Furthe
herm
rmorore,
e, a reve
revela
latio
tionn of for
forme
merr Supr
Suprem
emee Cour
Courtt Chie
Chieff Just
Justic
icee Robe
Robertrto
o
Concepcion
Concepcion revealed that only seven seven percent
percent of the Filipinos
Filipinos had a “living”
“living” income,
income, while ninety
ninety--

three
three perce
percent
nt lived
lived on a “hand
“hand-to-
-to-mou
mouth”
th” existenc
existence.
e.
Among
Among the myriads
myriads ofof solutio
solutions
ns to
to the populati
population
on problems
problems that have cropped
cropped up was one
that
that adv
advoca
ocated
ted the accele
accelerat
ration
ion of the growth
growth of the econo
economymy to 10.5
10.5 percen
percentt eve
every
ry year
year whic
whichh
meant
meant that our
our econo
economic
mic productio
production n should
should be increase
increased
d three times the
the growth
growth of our
our populati
population;
on;
that we should
should practice
practice austerity
austerity of consump
consumption
tion and austerity
austerity of imports.
imports. Another
Another solution
solution being
being
advocated
advocated was that
that the affluent
affluent section
section of our
our society
society should
should cut their
their standar
standardd of living in half and
and
use
use that
that mon
moneyey to impr
improv
ovee and
and grow
grow more
more food
food.. Anot
Anothe
herr solu
soluti
tion
on,, whic
whichh was
was per
perha
haps
ps bein
beingg
cons
consid
ider
ered
ed,, was
was the
the expo
export
rtat
atio
ion
n of huma
humansns to the
the moon
moon.. One
One solu
soluti
tion
on whic
which h was
was caus
causin
ingg a
confusin
confusing g debacle
debacle of minds
minds was concern
concerning
ing the use of the pill.

32. Dr. Paul


Paul Erlich
Erlich believed
believed that populati
population
on explosio
explosion
n would
would

(A) bring
bring about
about hung
hunger
er througho
throughoutut the world
world
(B) control
control man’s
man’s dream
dream to explore
explore outer
outer space
(C) force
force man to explore
explore the earth’s
earth’s natural
natural resource
resourcess

(D)  bring about stability and bridge the gap between the rich and the poor 
33. Acco
Accordi
rding
ng to the selec
selectio
tion,
n, Dr. Merce
Mercede
dess B. Conc
Concepc
epcion
ion feared
feared that
that the

(A)
(A) Filipi
Filipino
noss might becom
becomee too depen
dependen
dentt on other
other countr
countries
ies
(B)
(B) youn
youngg peop
people
le migh
mightt esta
establ
blis
ish
h an econ
econom
omy y base
based
d on soci
social
alis
ism
m
(C)
(C) influe
influence
nce of a youn
youngg popul
populati
ation
on might
might upset
upset our
our econ
econom
omicic struc
structur
turee
(D)
(D) youn
youngg popu
populat
lation
ion might
might contro
controll the econo
economic
mic reins
reins of the gover
governm
nmenentt

34. Accordin
According
g to popu
populat
lation
ion expert
experts,
s, during
during the 1970
1970’s,
’s, the earth’s
earth’s popu
populat
lation
ion for the next
next thirty
thirty
years
years would
would be

(A) 1.6 billio


lion
(B) 3.3 billio
lion
(C) 3.5 billio
lion
(D) 6.6 billio
lion

PRACTICE BOOKLET
BKL PhiLSAT_AA_0317 GO ON
ON TO THE NEXT
NEXT PAGE
PAGE
40

Selection
Selection 9

(1) In everyd
everyday
ay life,
life, we tend
tend to assign
assign less
less weigh
weightt to oral
oral speech
speech than
than to the writte
writtenn word.
word.
The
The spok
spoken
en is pres
presum
umed
ed to be “eph
“ephememer
eral
al,”
,” liabl
liablee to be mish
mishea
eard
rd,, rei
reinter
nterpr
pret
eted
ed,, or deni
denied
ed.. The
The
writte
written
n word,
word, in contr
contrast
ast,, is thoug
thought
ht to be “inde
“indelib
lible,
le,”” its meanin
meaning g cast
cast in stone.
stone. But,
But, the litera
literary
ry
scholar
scholar Rolan
Rolandd Barthes,
Barthes, in his classi
classicc essay,
essay, “The Death
Death of the Author,”
Author,” reverses
reverses this equation.
equation. He
argue
argues:
s: “[I]t
“[I]t is ephem
ephemera
erall speech
speech which
which is indeli
indelible
ble,, not
not monume
monumenta ntall writing
writing….…. Speec
Speech h is
irreversib
irreversible:
le: a [spoken]
[spoken] word cannot
cannot be retracted…
retracted…””

(2) Nowhere, perhaps, is this insight truer


truer than in international gatherings of heads
heads of state
or foreign
foreign ministers,
ministers, where
where diplomatic
diplomatic language
language is the norm.
norm. Participa
Participants
nts studious
studiously
ly avoid
avoid giving
giving
off-the-c
off-the-cuff
uff remar
remarks
ks in order
order not
not to be misinterpr
misinterpreted
eted.. Most
Most speak
speak from detailed
detailed notes
notes or simply
simply
read
read a prepa
prepared
red speech
speech.. At the
the end of suc
such
h meetin
meetingsgs,, a carefu
carefully
lly crafte
crafted
d commu
communiqniqué
ué is typ
typica
ically
lly
agreed
agreed upon
upon and issue
issued
d to make
make sure
sure there
there is no
no room
room for misun
misunder
dersta
stand
nding
ing..

(3)
(3) In
In such
such gath
gather
erin
ings
gs,, word
wordss do
do matt
matter
er.. Utmo
Utmostst cord
cordia
iali
lity
ty is obse
observ
rved
ed even
even amon
among g
represent
representative
ativess of rival
rival nations.
nations. That is why
why presiden
presidents,
ts, prime ministers,
ministers, and monarch
monarchss bring with
them
them a staff
staff of season
seasoneded diplom
diplomatsats and
and write
writers
rs who prepa
prepare
re the groun
groundd for their
their princ
principa
ipals
ls long
long
before
before the latter
latter actua
actually
lly meet.A
meet.A stro
strong
ng opini
opinionloose
onlooselyly uttere
uttered
d byone head
head of state
state inthe presen
presence
ce
of anothe
anotherr could
could be taken
taken as a slight.
slight. The conseque
consequences
nces that follow
follow could
could be costly,
costly, and unfortun
unfortunate
ate
because unintended.
unintended.

(4) It is one
one thin
thing
g for
for a nati
nation
on’s
’s lead
leaderer to spea
speakk and
and act
act in a cali
calibr
brat
ated
ed way
way in thethe purs
pursui
uitt
of a clea
clearr and
and cohe
cohere
rent
nt for
forei
eign
gn poli
policy
cy.. It is
is qui
quite
te anot
anothe
herr to allo
allow w ones
onesel
elf,
f, espe
especi
cial
ally
ly on the
the
intern
internati
ation
onal
al stage,
stage, to be a captiv
captivee of the
the conti
contingngenc
encies
ies of oral
oral speec
speech. h. “Spee
“Speech
ch can be a stream
stream of                 
cons
consci
ciou
ousn
snes
ess,
s, with
with unfi
unfininish
shed
ed utte
uttera
ranc
nces
es,, half
half-f
-for
orme
medd thou
though
ghts
ts,, and
and a heal
healththy
y smat
smatteteri
ring
ng of         
mess
messyy slan
slang.
g. You
You don’
don’tt have
have to know
know how
how a sentsenten
ence
ce will
will end
end bef
before
ore you
you star
startt it.”
it.” (Chi
(Chi Luu,
Luu, “Is
“Is
writing
writing a technolo
technology
gy or language
language?”)?”) It is from this thatthat extempora
extemporaneo neous
us speech
speech draws
draws its charm
and power,
power, as well as its inherent
inherent risks.
risks. This is partic
particularl
ularly
y so if one
one happens
happens to be a head of state.state.

(5) Under
Under our system
system of gove
governme
rnment,
nt, the Presiden
Presidentt is vested
vested with the power
power to determin
determinee
foreign
foreign policy
policy and to conduct
conduct foreign
foreign relations
relations with other
other states.
states. He alone
alone can negotiate
negotiate treaties
treaties on
behalf
behalf of the country,althoug
country,although, h, under
under the 1987
1987 Constitu
Constitution,
tion, suchtreaties
suchtreaties are subject
subject toto ratificat
ratification
ion
  by the
the Sena
Senatete.. A 1988
1988 Supr
Suprem
emee Cour
Courtt rulin
ruling
g denyi
denying
ng medi
mediaa acce
access
ss to minu
minute
tess taken
taken at a treat
treaty
y
negotiatio
negotiationn (G.R.
(G.R. No. 84642)
84642) made this very clear:
clear: “The
“The Presiden
Presidentt alone
alone has
has the powe
powerr to speak 
speak 
or listen as a represen
representativ
tativee of the Nation.
Nation. Congres
Congresss itself
itself is powe
powerless
rless to invade
invade it. The Presiden
Presidentt
is the sole
sole organ
organ of the Natio
Nationn in its extern
external
al relati
relation
ons,s, and
and its sole
sole repres
represen
entat
tative
ive with
with foreig
foreignn
  Nations.”

(6) This respon


responsibili
sibility
ty makes
makes it imperative
imperative for any presiden
presidentt to always
always speak
speak with caution.
caution.
Whether
Whether it is to communi
communicate cate a substanti
substantiveve shift
shift in
in foreign
foreign policy,
policy, or it is simply
simply to reiterate
reiterate time-
honore
honored
d princi
principle
pless in foreig
foreignn relati
relation
onss —such
such as sove
soverei
reign
gn equali
equality
ty and nonin
noninter
terfer
feren
ence
ce  —it is
important
important for any administr
administratio
ation
n to clearly
clearly state if it is veering
veering toward
toward a radical
radical rethinkin
rethinking g of the
country’s
country’s existing
existing alliances.
alliances. If it is, our people
people deserve
deserve to know.
know.

(7) The Presiden


Presidentt canno
cannott say
say one thing and then,
then, immediatel
immediately y after,
after, his staff scramble
scrambless to
redesc
redescrib
ribee what
what he actua
actually
lly said.
said. If done too often
often,, it can unde
undermi
rmine
ne the Presid
President
ent’s
’s credibi
credibility
lity.. In
an era where
where speech
speeches
es are
are instan
instantly
tly record
recorded,
ed, trans
transcri
cribe
bed,
d, and retrie
retrieve
ved,
d, there
there is hardl
hardly
y any
any room
room

PRACTICE BOOKLET
BKL PhiLSAT_AA_0317 GO ON
ON TO THE NEXT
NEXT PAGE
PAGE
41

for widely differing


differing interpreta
interpretations
tions.. Fortunat
Fortunately,
ely, the rules
rules of diplomacy
diplomacy give parties
parties enoughleeway
to offer
offer or demand
demand clarificat
clarifications
ions of contro
controvers
versial
ial statemen
statementsts  — precisely because words have
contexts
contexts and conseque
consequences
nces.. Parties
Parties at the receiving
receiving end of strong
strong statement
statementss often
often want to know
know if                 
there
there is a hidden
hidden messa
messagege behind
behind these
these uttera
utterance
nces.s.

An   excerpt from “Speech and the written


written word” by Randy David,
http://opinion.inquirer.net/97234/speech-and-the-written-word#ixzz4azunO3IX      

35. Which
Which of the follow
following
ing is highli
highligh
ghted
ted by the write
writerr in his essay
essay??

(A)
(A) The
The impo
import
rtan
ance
ce of dipl
diplom
omac
acyy
(B)
(B) The
The impo
import
rtan
ance
ce of prepa
prepare
red
d spee
speech
ches
es
(C) The mandated
mandated power
power of the
the presiden
presidentt
(D)
(D) The
The diff
differ
eren
ence
ce betw
betwee
een
n oral
oral spee
speech
ch and
and writt
written
en word
word

36. What
What writin
writing
g style
style did the write
writerr use to develo
develop
p his essay
essay??

(A)
(A) Deduc
educti
tive
ve
(B) Indu
nductiv
tive
(C)
(C) Induc
Inductiv
tive-d
e-ded
educt
uctive
ive
(D)
(D) Dedu
Deducti
ctive
ve-in
-indu
ducti
ctive
ve

37. In the first


first three paragr
paragrap
aphs,
hs, the writer
writer implies
implies the import
importan
ance
ce of             

(A) since
inceri
rity
ty
(B)
(B) blun
bluntn
tnes
esss
(C)
(C) tact
tactfu
fuln
lnes
esss
(D)
(D) cunn
cunnin
ingn
gnesesss

38. Accordi
According
ng to the author,
author, communic
communication
ation,, whether
whether written
written or spoken,
spoken,

(A)
(A) shou
should
ld be care
carefu
fully
lly craf
crafte
ted
d
(B)
(B) can
can be misu
misund
nder
erst
stoo
oodd
(C)
(C) shou
should
ld be rehe
rehear
arse
sed
d
(D)
(D) can
can spre
spread
ad quic
quickl
kly
y

39. What is the


the attitude of the writer towards extemporaneous
extemporaneous speech of dignitaries?

(A)
(A) Indi
Indiff
ffer
eren
ence
ce
(B)
(B) Relu
Reluct
ctan
ance
ce
(C) Tolera
lerannce
(D)
(D) Passi
assivi
vity
ty

PRACTICE BOOKLET
BKL PhiLSAT_AA_0317 GO ON
ON TO THE NEXT
NEXT PAGE
PAGE
42

40. Which
Which of the follow
following
ing is an import
important
ant consid
considera
eratio
tion
n when
when speaki
speaking
ng as implied
implied in the
  passage?

(A) Audienience
(B) Autho
thority
rity
(C) Fllu
ueency
(D) Contextext

41. Which
Which of the
the follow
following
ing words
words can
can be used
used synon
synonym
ymou
ously
sly with
with the inten
intended
ded meanin
meaning
g of the
the
writer
writer in paragrap
paragraph h 7?

(A)
(B) Retract
Restate
(C) Rectif
tify
(D) Rephrase
rase

42. The
The essa
essay
y as a whol
wholee is true
true to

(A)
(A) cele
celebr
brit
itie
iess
(B)
(B) head
headss of stat
states
es
(C)
(C) chur
church
ch offi
offici
cial
alss
(D)
(D) peop
people
le from
from all walks
walks of life
life
  ______________________________________________________________________________ 

Selection
Selection 10

When
When a hacker
hacker claime
claimed
d to have
have breach
breached
ed Mitt
Mitt Romney’s personal
personal email account in 2012,

he didn’t
didn’t do
  password
password itking
by
by infect
cracking
crac infecting
ing
k  —his
attack 
attac compu
computer
he did ter with
it with wit
thehword
a data
dat“Seamus.”
a -leeching
-leeching virus
virus or by launchin
launching
g a brute-fo
brute-force
rce

Seam
Seamus us was
was the
the name
name of Rom Romneney’
y’ss dog,
dog, and
and appa
appare
rent
ntly
ly the
the answer
answer to his passwo
passwordrd
reminder
reminder questio
question,
n, “what
“what is your
your favorit
favoritee pet.” Because
Because Romney’
Romney’ss email
email address
address had
had been made
made
publ
public
ic in a news
news sto
story
ry seve
severa
rall days
days earl
earlie
ier,
r, and
and the
the dogg
doggo-
o-in
in-q
-que
uest
stio
ionn was
was the
the subj
subjec
ectt of an
unfort
unfortun
unate
ate media
media scanda
scandall for havin
havingg been
been strapp
strapped
ed to the roof
roof of the family
family car durin
duringg a 1983
1983
road
road trip
trip,, the
the alle
allege
gedd hack
hacker
er had
had ever
everyt
ythi
hingng he need
neededed to expl
exploi
oitt a noto
notoririou
ousl
sly
y weak
weak gate
gatewa
wayy to
  password
password security:
security: the
the password
password recovery
recovery question.
question.

Whil
Whilee sett
settin
ing
g a pass
passwo
wordrd remi
remind
nder
er ques
questio
tion
n is a fine
fine idea
idea in theo
theory
ry (so
(so many
many pass
passwo
wordrds,
s,
so little mental
mental space!),
space!), it has probably
probably encourag
encouraged ed you to make your passworpassword d overly
overly vulnerab
vulnerable.le.
The
The simp
simple
le trut
truth
h is tha
thatt in our
our age
age of soc
socia
iall medi
mediaa over
over-s -sha
hari
ring
ng,, it is far
far too
too easy
easy to suss
suss out
out
anyone’s
anyone’s answers
answers toto the question
question “where
“where did you meet your your spouse
spouse,” ,” or “wha
“whatt is your
your mother’
mother’ss
maid
maidenen name
name.”
.” If you
you have
have a publ
public
ic Face
Facebo
book
ok,, Twitt
Twitter
er,, or Inst
Instag
agraramm acco
accoun
unt,
t, you
you also
also have
have a
dossier
dossier of clues
clues for would-
would-bebe hackers to peruse
peruse at will. Many security
security industry
industry professi
professional
onalss wish
the passwor
password d reminder
reminder questio
questionn would
would be outright
outright abolish
abolisheded from account
account setup,
setup, but until
until that day
comes,
comes, what
what can you
you do to work
work with
with the system
system and keep
keep yours
yourself
elf secure
secure??

For
For one
one thing
thing,, pick
pick a harde
harderr ques
questio
tion.
n. A Micro
Microso
soft
ft and
and Carne
Carnegiegie Mello
Mellonn study
study found
found that
that
the safest
safest passwor
password d reminder
reminder quest
question
ion may be “What’s
“What’s your
your father
father’s
’s middle
middle name,”
name,” as it’s
it’s easy to
reme
rememb
mberer,, hard
hard to gues
guess,s, and
and unlik
unlikel
ely
y to be publ
public
ic know
knowleledg
dgee on the
the Inte
Intern
rnet
et.. (Oth
(Other
er safe
safe-i
-ish
sh

PRACTICE BOOKLET
BKL PhiLSAT_AA_0317 GO ON
ON TO THE NEXT
NEXT PAGE
PAGE
43

question
questionss were,
were, “What
“What was
was your
your first
first phone
phone number?
number?,”
,” “Who
“Who was your
your favor
favorite
ite teacher,”
teacher,” and
“Who is your favorite
favorite singer?”)
singer?”)

Some
Some experts
experts recommen
recommend d answering
answering the question
question with a non sequitur (What is your mom’s
maid
maiden
en name
name?? Plat
Platyp
ypus
us).
). But
But even
even a rand
random
om,, one-
one-wo
word
rd answ
answerer is vuln
vulner
erab
able
le to a brut
brute-
e-fo
forc
rcee
hacking
hacking attack
attack designed
designed to rapidly
rapidly guess
guess every
every combinati
combination
on of letters
letters and numbers
numbers in sequence
sequence..

No matt
matter
er whic
which h secu
securi
rity
ty ques
questi
tion
on you
you go with,ith, your
your best
best bet
bet isis to
to trea
treatt it
it lik
likee
another password 
password —a long
long string
string of letter
letters,
s, numbe
numbers,
rs, and specia
speciall charac
character
terss that
that could
could not
not be
guesse
guessed
d or divine
divined
d from
from a cursor
cursoryy glance
glance at Facebo
Facebook
ok.. Use
Use a full
full phras
phrasee instea
instead d of a single
single word
word
(mayb
(maybee even
even one
one that
that inspir
inspires
es you
you to be a better
better perso
person
n every
every time
time you
you type
type it).
it). Need
Need an examp
example?le?

If you
you pick
pick the
the ques
questi
tion
on,, “wha
“H3NeverC@meWhen1C@lled!”. “whatt was
was your
your fir
first
st dog
dog’s
’s nam
namee ,” a strong answ
answer
er woul
would
d be

Make
Make sure
sure you
you use a uniqu
uniquee answe
answerr for every
every accou
accountnt that
that require
requiress a remind
reminderer quest
question
ion,,
and log them
them in a passw
password
ord manag
manager er along
alongsid
sidee your
your other
other passw
password
ords.
s. Of cours
course,
e, this
this is all moot
moot
for
for the
the esti
estima
mate
ted
d 17%
17% of Amer
Americican
anss who
who secu
secure
re thei
theirr data
data with
with the
the pass
passwo
word
rd “123
“12345456.
6.”” But
But
  progress happens inbaby steps (just maybe
maybe don’t use your baby’s name
name as your passwordreminder 
answer).

  An excerpt from “Your Password Recovery Questions Are Insanely Easy                     


To Hack—and
Hack—and You Might Blame”     b    y    Brandon
Might Be to Blame”                        Specktor 
http://www.rd.com/advice/work-career/password-recovery-questions/

43. In the first


first two paragr
paragrap
aphs,
hs, the writer
writer implie
implied
d that
that

(A)
(A) what
what we
we thou
though
ghtt to be priv
privat
atee is act
actua
ually
lly publ
public
ic
(B)
(B) what
what a public
public figure
figure does
does excite
excitess hacke
hackers
rs
(C) the answ
answer
er to the securi
security
ty quest
question
ion is weak 
weak 
(D)
(D) the
the hack
hacker
er is a geni
genius
us

44. The purpose


purpose of the writer
writer is to

(A) entert
tertaain
(B)
(B) pers
persua
uade
de
(C) anally
yzze
(D) inform

45. Which
Which of the follow
following
ing is most
most likely
likely the reaso
reason
n why securi
security
ty indus
industry
try profes
professio
sional
nalss
would
would like to abolish
abolish the password
password reminder
reminder question
question??

(A)
(A) Answ
Answer
erss are easi
easily
ly deco
decodeded.
d.
(B)
(B) Answ
Answers
ers are identi
identical
cal to others
others..
(C)
(C) Answ
Answers
ers are diffic
difficult
ult to const
construc
ruct.
t.
(D) Answers
Answers are
are most
most of the time forgotten
forgotten..

PRACTICE BOOKLET
BKL PhiLSAT_AA_0317 GO ON
ON TO THE NEXT
NEXT PAGE
PAGE
44

46. The
The articl
articlee as a whole
whole explor
explores
es the
the

(A)
(A) flaw
flawss of
of set
setti
ting
ng up a pass
passwoword
rd
(B)
(B) ways
ays to crea
create
te a pass
passwo
word
rd
(C)
(C) poss
posses
essi
sion
on of an
an acco
accoun
untt
(D)
(D) hack
hackin
ing
g of anan acco
accoun
untt

47. The last paragrap


paragraph
h is an example
example of a(an)
a(an)

(A)
(A) inst
instru
ruct
ctio
ion
n
(B)
(B) sugg
sugges
esti
tion
on

(C) command
(D) demand

48. Which
Which of the
the following
following statemen
statements
ts is most
most likely
likely NOT true?

I. Acco
Accoun
untt holde
holders
rs inten
intentio
tional
nally
ly leave
leave clues
clues upo
upon n passw
password
ord set up.
up.
II.
II. Hack
ackers
ers only attack famous personalities’
personalities’ account
account..
III.
III. Ever
Everyo
yone
ne can
can be a hack
hackin
ing
g vict
victim
im..
IV.
IV. Identi
Identity
ty theft
theft is possi
possible
ble when
when the accoun
accountt is hacked.
hacked.

(A) I and
and II only
(B) I and III only
(C) II an
and III
III onl
only
y
(D) II an
and IV onllyy

49. Which
Which quote
quote can be direct
directly
ly assoc
associat
iated
ed with
with the articl
article?
e?

(A) “It’s more than what


what meets
meets the
the eye.”
eye.”
(B) “The
“The world
world is watch
watching
ing us.”
us.”
(C) “Be
“Be the first
first to
to know.
know.””
(D) “It’s a small world.”
world.”

50. Based
Based on the article,
article, account
account holders
holders should
should

(A) answer
answer remind
reminderer questi
questions
ons discreetl
discreetly
y
(B)
(B) deact
deactiva
ivate
te their
their accou
accounts
nts every
every two
two week
weekss
(C)
(C) aboli
abolish
sh remind
reminder er quest
question
ionss
(D)
(D) skip
skip remi
remind
nder
er quest
questio
ions
ns

PRACTICE BOOKLET
BKL PhiLSAT_AA_0317 GO ON
ON TO THE NEXT
NEXT PAGE
PAGE
45

TEST
TEST D. QUANTITATIVE REASONING

Section
Section 1. Pattern
Pattern Recogniti
Recognitio
Recognition
on
n

Direct
Direction
ions:
s: In this
this test,
test, the stem
stem consis
consiststs of a serie
seriess of numb
numbers
ers.. In each
each series
series,, discov
discover
er the
logica
logicall rule.
rule. Selec
Selectt from the five
five choice
choicess the
the numbe
numberr that shou
should
ld come
come next
next in the series
series..

1. 61 54 47 41 34 29 22

(A)
(B) 16
15 ((E
D)) 1
28
0
(C) 17

2. 16 32 8 48 6 60 5

(A) 50 (D) 70
(B) 55 (E) 75
(C) 80

3. 1 2 4 7 28 33 198

(A) 1,386 (D) 1,188


(B) 206 (E) 191
(C) 205

4. 5 30 25 5 35 30 6
(A) 48 (D) 36
(B) 1 (E) 25
(C) 54

5. 18 20 10 8 10 5

(A) 3 (D) 10
(B) 7 (E) 15
(C) 0

6. 88 102 99 111 105 115

(A) 123 (D) 124


(B) 106 (E) 103

(C) 104

PRACTICE BOOKLET
BKL PhiLSAT_AA_0317 GO ON TO THE
HE NEXT PAGE
46

7. 54 162 62 186 86

(A) 89 (D) 248


(B) 172 (E) 258
(C) 264

8. 69 72 24 27 9 12

(A) 3 (D) 8
(B) 4 (E) 15
(C) 6

9. 16 8 24 16 48 40

(A) 20 (D) 120


(B) 32 (E) 160
(C) 80

10. 25 15 15 6 12 4 12

(A) 8 (D) 48
(B) 6 (E) 3
(C) 5

11. 10 12 17 20 24 28

(A) 33 (D) 30
(B) 32 (E) 29
(C) 31

12. 3 8 18 23 32 37 45

(A) 48 (D) 49
(B) 50 (E) 51
(C) 52

13. 81 9 64 8 49 7 36

(A) 25 (D) 5
(B) 16 (E) 4
(C) 6

14. 107 102 93 86 79 70 65

(A) 62 (D) 55
(B) 59 (E) 54
(C) 58

PRACTICE BOOKLET
BKL PhiLSAT_AA_0317 GO ON TO THE
HE NEXT PAGE
47

15. 126 138 134 144 136 144 132

(A) 140 (D) 119


(B) 139 (E) 118
(C) 138

Sectio
Section
n 2. Data
Datta
Da a Suffic
Sufficien
iency
cy

Dire
Direct
ctio
ionsns:: The
The datadata suff
suffic
icie
ienc
ncyy prob
proble
lemm cons
consis
ists
ts of a quesquesti
tion
on and
and two statstatem
emen
ents
ts,,
labele
labeled
d (I) and (II),
(II), in whichcerta
whichcertain in data
data aregiven.
aregiven. Decid
Decidee wheth
whetherer the data
data given
given in thestateme
thestatements
nts
are suffic
sufficien
ientt for answeri
answeringng the quest
question
ion.. Using
Using the data
data given
given in the statem
statement
ents,
s, plus
plus knowled
knowledgege
of mathem
mathematicsatics and everyday
everyday facts (such
(such as the
the number
number of of days
days in July or the
the meaning
meaning of thethe word
counterclockwise),
counterclockwise), determine whether:
 Statement (I) ALONE
ALONE is sufficient,
sufficient, but statement
statement (II) alone is not sufficient to answer the
question
question asked.
asked.
 Statement (II) ALONE
ALONE is sufficient,
sufficient, but statement
statement (I) alone is not sufficient to answer the
question
question asked.
asked.
 BOTH
BOTH statement
statementss (I) and
and (II) TOGE
TOGETHE THER R are sufficien
sufficientt to answer
answer the question
question asked,
asked,
  but NEITHER
NEITHER statement
statement ALONE
ALONE is sufficien
sufficientt to answer
answer the question
question asked.
asked.
 EACH
EACH statement
statement ALON
ALONE E is sufficien
sufficientt to answer
answer the question
question asked.
asked.
 Statement
Statementss (I)(I) and
and (II) TOGETHE
TOGETHER R are NOT sufficien
sufficientt to answer
answer the
the question
question asked,
asked, and
additio
additionanall data
data speci
specific
fic to the proble
problem m are neede
needed. d.

16. What
What is the large
largest
st numb
number
er among
among the three
three numbe
numbers
rs A, B, and
and C?

I. A + B = 3 and B –  C = 5
II. 2A + 3C = 11 and
and A, B, and
and C can
can be nega
negativ
tivee or posi
positi
tive
ve numb
number
ers.
s.

(A) Statemen
Statementt (I) ALON
ALONE E is sufficie
sufficient,
nt, but statem
statement
ent (II)
(II) alone
alone is not
not sufficien
sufficient.
t.
(B) Statemen
Statementt (II) ALONE
ALONE is sufficie
sufficient,
nt, but statem
statement
ent (I)
(I) alone
alone is not
not sufficie
sufficient.
nt.
(C)
(C) BOTH
BOTH statem
statemenents
ts TOGETH
TOGETHER ER are suffi
sufficie
cient,
nt, but
but NEITH
NEITHER ER statem
statement
ent ALONE
ALONE
is sufficient.
(D)
(D) EACH
EACH statem
statemenentt ALO
ALONE NE is suff
suffici
icien
ent.
t.
(E) Statem
Statemenents
ts (I) and (II)
(II) TOG
TOGETETHE
HER R are NOTNOT suffi
sufficie
cient.
nt.

17. In how many


many minute
minutess can Raj
Raj wash
wash the car?
car?

I. Raj
Raj and
and Russ
Russel
elll toge
togeth
ther
er can
can wash
wash the
the car
car in 30 minu
minute
tes.
s.
II.
II. Russ
Russel
elll alo
alone
ne can
can fini
finish
sh washi
ashing
ng the
the car
car in
in 1 hour
hour..

(A) Statemen
Statementt (I) ALON
ALONE E is sufficie
sufficient,
nt, but statem
statement
ent (II)
(II) alone
alone is not
not sufficien
sufficient.
t.
(B) Statemen
Statementt (II) ALONE
ALONE is sufficie
sufficient,
nt, but statem
statement
ent (I)
(I) alone
alone is not
not sufficie
sufficient.
nt.
(C) BOTH
BOTH statement
statementss TOGETHER
TOGETHER are sufficien
sufficient,
t, but NEITHER
NEITHER statement
statement ALONE
ALONE
is sufficient.
(D)
(D) EACH
EACH statem
statemenentt ALO
ALONE NE is suff
suffici
icien
ent.
t.
(E) Statem
Statemenents
ts (I) and (II)
(II) TOG
TOGETETHE
HER R are NOT
NOT suffi
sufficie
cient.
nt.

PRACTICE BOOKLET
BKL PhiLSAT_AA_0317 GO ON TO THE
HE NEXT PAGE
48

18. How much does


does a pharmace
pharmaceutica
uticall sales represen
representativ
tativee earn in a year?
year?

I. A pharmace
pharmaceutica
uticall sales
sales represen
representativ
tativee averages
averages ₱42,000
₱42,000 a month
month including bonuses.
II. A pharmace
pharmaceutica
uticall sales representati
representative
ve averages
averages ₱40,000
₱40,000 a month
month and
and ₱2,5
₱2,500
00 in
commissions per month.

(A) Statemen
Statementt (I) ALON
ALONE E is sufficie
sufficient,
nt, but statem
statement
ent (II)
(II) alone
alone is not
not sufficien
sufficient.
t.
(B) Statemen
Statementt (II) ALONE
ALONE is sufficie
sufficient,
nt, but statem
statement
ent (I)
(I) alone
alone is not
not sufficie
sufficient.
nt.
(C)
(C) BOTH
BOTH statem
statemenents
ts TOGETH
TOGETHER ER are suffi
sufficie
cient,
nt, but
but NEITH
NEITHER ER statem
statement
ent ALONE
ALONE
is sufficient.
(D)
(D) EACH
EACH statem
statemenentt ALO
ALONE NE is suff
suffici
icien
ent.
t.
(E) Statem
Statemenents
ts (I) and (II)
(II) TOG
TOGETETHE
HER R are NOTNOT suffi
sufficie
cient.
nt.

19. Blooms
Blooms & Petals
Petals Flower
Flower Shop put on sale bouquet
bouquetss of flowers
flowers on Valentine
Valentine’s
’s Day.
Each
Each bouq
bouquet
uet was sold
sold at ₱450
₱450.. How
How much
much profit
profit did
did the store
store make
make from
from selling
100 bouquet
bouquets?
s?

I. The estimated
estimated cost of one
one bouquet
bouquet is ₱200
₱200..
II. The store
store charged
charged an additional
additional ₱50 for delivery
delivery orders.
orders.

(A) Statemen
Statementt (I) ALON
ALONE E is sufficie
sufficient,
nt, but statem
statement
ent (II)
(II) alone
alone is not
not sufficien
sufficient.
t.
(B) Statemen
Statementt (II) ALONE
ALONE is sufficie
sufficient,
nt, but statem
statement
ent (I)
(I) alone
alone is not
not sufficie
sufficient.
nt.
(C)
(C) BOTH
BOTH statem
statemenents
ts TOGETH
TOGETHER ER are suffi
sufficie
cient,
nt, but
but NEITH
NEITHER ER statem
statement
ent ALONE
ALONE
is sufficient.
(D)
(D) EACH
EACH statem
statemenentt ALO
ALONE NE is suff
suffici
icien
ent.
t.
(E) Statem
Statemenents
ts (I) and (II)
(II) TOG
TOGETETHE
HER R are NOTNOT suffi
sufficie
cient.
nt.

20. Which
Which of the two fast-food
fast-food chains
chains in the universit
university
y belt is more popular
popular among
among
the students
students of Universit
University
y M? The total populati
population
on of studen
students
ts in the universit
university
y is
approximately
approximately 45,000.
1
I. The
The prop
propor
orti
tion
on of stu
stude
dent
ntss that
that frequ
frequen
ents
ts Fas
Fastt Food
Food A is   of the
the total
5
student population.
II.
II. An aver
averag
agee of 6,50
6,500
0 stud
studen
ents
ts goes
goes to Fast
Fast Food
Food B per
per day
day..

(A) Statemen
Statementt (I) ALON
ALONE E is sufficie
sufficient,
nt, but statem
statement
ent (II)
(II) alone
alone is not
not sufficien
sufficient.
t.
(B) Statemen
Statementt (II) ALONE
ALONE is sufficie
sufficient,
nt, but statem
statement
ent (I)
(I) alone
alone is not
not sufficie
sufficient.
nt.
(C)
(C) BOTH
BOTH statem
statemenents
ts TOGETH
TOGETHER ER are suffi
sufficie
cient,
nt, but
but NEITH
NEITHER ER statem
statement
ent ALONE
ALONE
is sufficient.
(D)
(D) EACH
EACH statem
statemenentt ALO
ALONE NE is suff
suffici
icien
ent.
t.
(E) Statem
Statemenents
ts (I) and (II)
(II) TOG
TOGETETHE
HER R are NOTNOT suffi
sufficie
cient.
nt.

PRACTICE BOOKLET
BKL PhiLSAT_AA_0317 GO ON TO THE
HE NEXT PAGE
49

21. How
How much
much tax will
will Man
Mang
g Tomas’
Tomas’ mini
mini grocery
grocery store must
must pay
pay at the
the end of the fiscal
fiscal year 
year 
if the tax rate
rate is 32%?
32%?

I. His taxable
taxable income
income amounted
amounted to ₱75,000
₱75,000..
II. His total sales
sales amounte
amounted
d to ₱250,00
₱250,000
0 and the cost of goods
goods sold is 80%
80% of his sales.

(A) Statemen
Statementt (I) ALON
ALONE E is sufficie
sufficient,
nt, but statem
statement
ent (II)
(II) alone
alone is not
not sufficien
sufficient.
t.
(B) Statemen
Statementt (II) ALONE
ALONE is sufficie
sufficient,
nt, but statem
statement
ent (I)
(I) alone
alone is not
not sufficie
sufficient.
nt.
(C)
(C) BOTH
BOTH statem
statemenents
ts TOGETH
TOGETHER ER are suffi
sufficie
cient,
nt, but
but NEITH
NEITHER ER statem
statement
ent ALONE
ALONE
is sufficient.
(D)
(D) EACH
EACH statem
statemenentt ALO
ALONE NE is suff
suffici
icien
ent.
t.
(E) Statem
Statemenents
ts (I) and (II)
(II) TOG
TOGETETHE
HER R are NOTNOT suffi
sufficie
cient.
nt.

22. How
How old is Rosa
Rosa today
today??

I. Rosa
Rosa is 5 year
yearss youn
younge
gerr than
than her
her brot
brothe
herr Jose
Joseph
ph..
II.
II. In thre
threee year
years,
s, Jose
Joseph
ph will
ill be
be twic
twicee as old
old as Rosa
Rosa..

(A) Statemen
Statementt (I) ALON
ALONE E is sufficie
sufficient,
nt, but statem
statement
ent (II)
(II) alone
alone is not
not sufficien
sufficient.
t.
(B) Statemen
Statementt (II) ALONE
ALONE is sufficie
sufficient,
nt, but statem
statement
ent (I)
(I) alone
alone is not
not sufficie
sufficient.
nt.
(C)
(C) BOTH
BOTH statem
statemenents
ts TOGETH
TOGETHER ER are suffi
sufficie
cient,
nt, but
but NEITH
NEITHER ER statem
statement
ent ALONE
ALONE
is sufficient.
(D)
(D) EACH
EACH statem
statemenentt ALO
ALONE NE is suff
suffici
icien
ent.
t.
(E) Statem
Statemenents
ts (I) and (II)
(II) TOG
TOGETETHE
HER R are NOTNOT suffi
sufficie
cient.
nt.

23. How
How much
much simple
simple intere
interest
st is earn
earned
ed at the
the end of one
one year?
year?

I. Lucy deposited ₱35,000


₱35,000 in a bank
bank that gives
gives 0.1%
0.1% interest
interest per year.
year.
II. Lucy
Lucy invested
invested her money
money amountin
amounting g to ₱35,000
₱35,000 in a financial
financial market
market that offers
offers
3% interest
interest rate
rate per annum
annum..

(A) Statemen
Statementt (I) ALON
ALONE E is sufficie
sufficient,
nt, but statem
statement
ent (II)
(II) alone
alone is not
not sufficien
sufficient.
t.
(B) Statemen
Statementt (II) ALONE
ALONE is sufficie
sufficient,
nt, but statem
statement
ent (I)
(I) alone
alone is not
not sufficie
sufficient.
nt.
(C)
(C) BOTH
BOTH statem
statemenents
ts TOGETH
TOGETHER ER are suffi
sufficie
cient,
nt, but
but NEITH
NEITHER ER statem
statement
ent ALONE
ALONE
is sufficient.
(D)
(D) EACH
EACH statem
statemenentt ALO
ALONE NE is suff
suffici
icien
ent.
t.
(E) Statem
Statemenents
ts (I) and (II)
(II) TOG
TOGETETHE
HER R are NOTNOT suffi
sufficie
cient.
nt.

PRACTICE BOOKLET
BKL PhiLSAT_AA_0317 GO ON TO THE
HE NEXT PAGE
50

24. Which
Which of the
the rectangu
rectangular
lar and
and square
square lots has
has a bigger
bigger area?
area?

I. The
The perim
perimete
eterr of
of the
the rectan
rectangu
gular
lar lot is twice
twice the perime
perimeter
ter of the square
square lot.
lot.
II.
II. The
The widt
width
h of the
the rect
rectan
angu
gula
larr lot
lot is equal
equal to the
the leng
length
th of one
one side
side of the
the squa
square
re lot.
lot.

(A) Statemen
Statementt (I) ALON
ALONE E is sufficie
sufficient,
nt, but statem
statement
ent (II)
(II) alone
alone is not
not sufficien
sufficient.
t.
(B) Statemen
Statementt (II) ALONE
ALONE is sufficie
sufficient,
nt, but statem
statement
ent (I)
(I) alone
alone is not
not sufficie
sufficient.
nt.
(C)
(C) BOTH
BOTH statem
statemenents
ts TOGETH
TOGETHER ER are suffi
sufficie
cient,
nt, but
but NEITH
NEITHER ER statem
statement
ent ALONE
ALONE
is sufficient.
(D)
(D) EACH
EACH statem
statemenentt ALO
ALONE NE is suff
suffici
icien
ent.
t.
(E) Statem
Statemenents
ts (I) and (II)
(II) TOG
TOGETETHE
HER R are NOTNOT suffi
sufficie
cient.
nt.

25. How
How much
much money
money does
does Mrs.
Mrs. Gome
Gomezz save
save from
from her sala
salary
ry per mon
month?
th?
2
I. Her mo
monthlysa
thlysala
laryis
ryis just
just of her husband’s
husband’s monthly
monthly salary.
3
II.
II. She spen
spends
ds 50%
50% of her
her sal
salar
ary
y on food
food..

(A) Statemen
Statementt (I) ALON
ALONE E is sufficie
sufficient,
nt, but statem
statement
ent (II)
(II) alone
alone is not
not sufficien
sufficient.
t.
(B) Statemen
Statementt (II) ALONE
ALONE is sufficie
sufficient,
nt, but statem
statement
ent (I)
(I) alone
alone is not
not sufficie
sufficient.
nt.
(C)
(C) BOTH
BOTH statem
statemenents
ts TOGETH
TOGETHER ER are suffi
sufficie
cient,
nt, but
but NEITH
NEITHER ER statem
statement
ent ALONE
ALONE
is sufficient.
(D)
(D) EACH
EACH statem
statemenentt ALO
ALONE NE is suff
suffici
icien
ent.
t.
(E) Statem
Statemenents
ts (I) and (II)
(II) TOG
TOGETETHE
HER R are NOTNOT suffi
sufficie
cient.
nt.

26. A publis
publishe
herr rece
recentl
ntly
y releas
released
ed a new
new scienc
sciencee fictio
fiction
n nove
novell in book
booksto
stores
res.. How
How much
much is
the sellin
selling
g price
price of one
one book
book??

I.
II. A
Thmarku
mar
The kupp of
e materi
mat als125%
erials12
an5%
and isnting
adde
added
d printi
pri ng dcost
coto
stthe srcina
srcinal
of one l cost
book
boo ckost.
. ounted
amoun
am ted to ₱250
₱250..

(A) Statemen
Statementt (I) ALON
ALONE E is sufficie
sufficient,
nt, but statem
statement
ent (II)
(II) alone
alone is not
not sufficien
sufficient.
t.
(B) Statemen
Statementt (II) ALONE
ALONE is sufficie
sufficient,
nt, but statem
statement
ent (I)
(I) alone
alone is not
not sufficie
sufficient.
nt.
(C)
(C) BOTH
BOTH statem
statemenents
ts TOGETH
TOGETHER ER are suffi
sufficie
cient,
nt, but
but NEITH
NEITHER ER statem
statement
ent ALONE
ALONE
is sufficient.
(D)
(D) EACH
EACH statem
statemenentt ALO
ALONE NE is suff
suffici
icien
ent.
t.
(E) Statem
Statemenents
ts (I) and (II)
(II) TOG
TOGETETHE
HER R are NOTNOT suffi
sufficie
cient.
nt.

PRACTICE BOOKLET
BKL PhiLSAT_AA_0317 GO ON TO THE
HE NEXT PAGE
51

27. Susan
Susan won
won ₱300
₱300,00
,0000 from
from the lotter
lottery.
y. She
She plans
plans to inves
investt a part
part of her winnin
winningsgs and
the rest
rest she will
will set aside
aside as emerg
emergen
ency
cy fund.
fund. How
How much
much did she set set aside for emerg
emergen
encie
cies?
s?

I. Sixty
Sixty perce
percent
nt of her
her winn
winning
ingss was
was invest
invested
ed to a finan
financia
ciall marke
markett that
that offers
offers
7% annual
annual interest
interest rate compoun
compounded
ded annually
annually for 5 years.
years.
II. She
She used
used 50% of her winnin
winnings
gs to start
start a buy-a
buy-and-
nd-sel
selll busin
business
ess..

(A) Statemen
Statementt (I) ALON
ALONE E is sufficie
sufficient,
nt, but statem
statement
ent (II)
(II) alone
alone is not
not sufficien
sufficient.
t.
(B) Statemen
Statementt (II) ALONE
ALONE is sufficie
sufficient,
nt, but statem
statement
ent (I)
(I) alone
alone is not
not sufficie
sufficient.
nt.
(C)
(C) BOTH
BOTH statem
statemenents
ts TOGETH
TOGETHER ER are suffi
sufficie
cient,
nt, but
but NEITH
NEITHER ER statem
statement
ent ALONE
ALONE
is sufficient.
(D)
(D) EACH
EACH statem
statemenentt ALO
ALONE NE is suff
suffici
icien
ent.
t.
(E) Statem
Statemenents
ts (I) and (II)
(II) TOG
TOGETETHE
HER R are NOTNOT suffi
sufficie
cient.
nt.

28.
28. A gradu
graduate
ate was
was calcul
calculati
ating
ng the average
average increa
increase
se in his daily
daily allow
allowanc
ancee durin
during
g his
college
college years.
years.

I. His
His daily allow
allowan
ance
ce durin
during
g his four
four years
years in colle
college
ge were
were ₱150
₱150,, ₱200
₱200,, ₱300
₱300 and
₱350, respectively.
II. His
His paren
parents
ts then
then increa
increased
sed his allow
allowan
ance
ce based
based on his Gene
General
ral Weigh
Weighted
ted Avera
Average
ge
every
every semester.
semester.

(A) Statemen
Statementt (I) ALON
ALONE E is sufficie
sufficient,
nt, but statem
statement
ent (II)
(II) alone
alone is not
not sufficien
sufficient.
t.
(B) Statemen
Statementt (II) ALONE
ALONE is sufficie
sufficient,
nt, but statem
statement
ent (I)
(I) alone
alone is not
not sufficie
sufficient.
nt.
(C)
(C) BOTH
BOTH statem
statemenents
ts TOGETH
TOGETHER ER are suffi
sufficie
cient,
nt, but
but NEITH
NEITHER ER statem
statement
ent ALONE
ALONE
is sufficient.
(D)
(D) EACH
EACH statem
statemenentt ALO
ALONE NE is suff
suffici
icien
ent.
t.
(E) Statem
Statemenents
ts (I) and (II)
(II) TOG
TOGETETHE
HER R are NOTNOT suffi
sufficie
cient.
nt.

29. Benjie’s
Benjie’s Bed and Breakfas
Breakfastt located
located in Baguio City is offering
offering fully furnishe
furnished
d transien
transientt
rooms
rooms at a discounte
discountedd rate of 20%
20% off its
its srcinal
srcinal rent per day
day during
during off-peak
off-peak season.
season.
How
How much
much will
will a tour
touris
istt have
have to pay for
for a day
day for
for two room
rooms?
s?

I. The
The rent
rent for
for thre
threee room
roomss with
with one
one extr
extraa bed
bed is ₱2,500.
II.
II. Durin
uring
g peak
peak seas
season
on,, one
one room
room is rent
rented
ed at ₱850
₱850 per day.
day.

(A) Statemen
Statementt (I) ALON
ALONE E is sufficie
sufficient,
nt, but statem
statement
ent (II)
(II) alone
alone is not
not sufficien
sufficient.
t.
(B) Statemen
Statementt (II) ALONE
ALONE is sufficie
sufficient,
nt, but statem
statement
ent (I)
(I) alone
alone is not
not sufficie
sufficient.
nt.
(C)
(C) BOTH
BOTH statem
statemenents
ts TOGETH
TOGETHER ER are suffi
sufficie
cient,
nt, but
but NEITH
NEITHER ER statem
statement
ent ALONE
ALONE
is sufficient.
(D)
(D) EACH
EACH statem
statemenentt ALO
ALONE NE is suff
suffici
icien
ent.
t.
(E) Statem
Statemenents
ts (I) and (II)
(II) TOG
TOGETETHE
HER R are NOTNOT suffi
sufficie
cient.
nt.

PRACTICE BOOKLET
BKL PhiLSAT_AA_0317 GO ON TO THE
HE NEXT PAGE
52

30. A senior
senior high school
school is offering
offering four strand
strandss in the academic
academic track:
track: General
General Academi
Academicc
Strand (GAS); Humanities and Social Sciences strand (HUMSS);(HUMSS); Science,
Science, Technology,
Technology,
Enginee
Engineering
ring,, and Mathemati
Mathematics cs strand
strand (STEM);
(STEM); and Accoun
Accountancy
tancy,, Business
Business,, and
Mana
Managem
gemenentt strand
strand (ABM)
(ABM).. How
How many
many enroll
enrolled
ed in the STEM
STEM strand
strand out
out of 5,400
5,400 enrol
enrollee
leess
in Grad
Gradee 11?
11?

I. Thir
Thirty
ty-t
-two
wo perc
percen
entt of the
the enro
enrollllee
eess chos
chosee GAS,
GAS, 25% chos chosee ABM,
ABM, and
and
11% chose
chose HUMSS.
HUMSS.
II.
II. The
The rati
ratio
o of STEM
STEM enro
enroll
llee
eess to ABM
ABM enro
enroll
llee
eess is 5 : 3.

(A) Statemen
Statementt (I) ALON
ALONE E is sufficie
sufficient,
nt, but statem
statement
ent (II)
(II) alone
alone is not
not sufficien
sufficient.
t.
(B) Statemen
Statementt (II) ALONE
ALONE is sufficie
sufficient,
nt, but statem
statement
ent (I)
(I) alone
alone is not
not sufficie
sufficient.
nt.
(C)
(C) BOTH
BOTH statem
statemenents
ts TOGETH
TOGETHER ER are suffi
sufficie
cient,
nt, but
but NEITH
NEITHER ER statem
statement
ent ALONE
ALONE
is sufficient.
(D)
(D) EACH
EACH statem
statemenentt ALO
ALONE NE is suff
suffici
icien
ent.
t.
(E) Statem
Statemenents
ts (I) and (II)
(II) TOG
TOGETETHE
HER R are NOTNOT suffi
sufficie
cient.
nt.

PRACTICE BOOKLET
BKL PhiLSAT_AA_0317 GO ON
ON TO THE
THE NEXT
NEXT PAGE
PAGE
53

Section
Section 3. Data Interpret
Interpretatio
pretation
ation
n

Direct
Direction
ions:
s: In this
this sectio
section,
n, some
some sets
sets of data
data are given
given.. Inter
Interpr
pret
et or analyz
analyzee the given
given data
data to
answer
answer the question
questionss that refer to the data presente
presented.
d. Select
Select the correct
correct answer
answer from the options
options
given.

For
For item
itemss 31
31 and 32,
32, refer
refer to the graph
graph below
below..

DISTRIBU
DISTRIBUTIO
TION
N OF SHARES
SHARES OF STOCKS
STOCKS OF DIFFER
DIFFERENT
ENT CORPOR
CORPORATIO
ATIONS
NS

40,000

s
e
r 30,000
a
h
S
f
o 20,000
r
e
b
m
u
N10,000

0
ABCDE
Corporation

31. The
The numbe
numberr of share
sharess owne
owned
d by A exceed
exceedss that
that of D by how many
many shares
shares??

(A) 10,000
(B) 20,000
(C) 25,000
(D) 37,500

32. What
What percent
percent of
of the number
number of
of shares
shares owne
owned
d by E is the number
number of
of shares
shares owne
owned
d by B?

(A) 42%
(B) 30%
(C) 25%
(D) 18%

PRACTICE BOOKLET
BKL PhiLSAT_AA_0317 GO ON TO THE
HE NEXT PAGE
54

For
For items
items 33
33 to 35, refer
refer to the chart
chart belo
below.
w.

PERCEN
PERCENTAG
TAGE
E DISTRIBU
DISTRIBUTIO
TIONN OF STUDEN
STUDENTS
TS ENROLL
ENROLLEDED IN THE DIFFER
DIFFERENT
ENT
COUR
COURSE
SES
S IN A CERTA
CERTAIN
IN UNIVE
UNIVERS
RSITY
ITY (N = 28,00
28,000)
0)

Dentistry
20%

  Nursing
8% Management
55%
Political Science
4%

Engineering
13%

33. Which
Which course(s
course(s)) comprise(
comprise(s)
s) one
one fifth of the total enrol
enrollmen
lmentt in the universit
university?
y?

(A) Denti
entist
stry
ry
(B)
(B) Engi
Engine
neer
erin
ing
g
(C)
(C) Nurs
Nursin
ing
g and
and Engi
Engine
neer
erin
ing
g
(D)
(D) Nursi
Nursing
ng,, Engine
Engineeri
ering
ng,, and Politi
Political
cal Scien
Science
ce

34. The differenc


differencee between
between the enrollmen
enrollments
ts in Managem
Management
ent and in Engineer
Engineering
ing is

(A) 14,280
(B) 13,160
(C) 11,760
(D) 9,800

35. What
What is the ratio
ratio of studen
students
ts enrol
enrolled
led in Mana
Managem
gement
ent to those
those who
who are enrol
enrolled
led in
Dentistry?

(A) 4 : 11
(B) 11 : 4
(C) 20 : 55
55
(D) 55 : 20

PRACTICE BOOKLET
BKL PhiLSAT_AA_0317 GO ON TO THE
HE NEXT PAGE
55

For
For item
itemss 36
36 to 38, refer
refer to the table
table below
below..

DISTRIB
DISTRIBUTI
UTION
ON OF DEGREES
DEGREES GRANTED
GRANTED
IN PHILIPP
PHILIPPINE
INE COLLEG
COLLEGES
ES AND UNIVER
UNIVERSITI
SITIES
ES

Degree N %
Education 33,911,000 39.24
Business 34
34,193,000 39.57
Law 1,105,000 1.28
Architecture 279,000 0.32

Engineering 4,462,000 5.16


Social Science 5,086,000 5.88
  Na
Natural Science 2,364,000 2.74
Medical Science 5,019,000 5.81
Medicine (36.5)
Dentistry (7.7)
  Nursing (41.8)
Optometry (3.8)
Pharmacy (10.2)

36. Approxim
Approximately
ately,, what
what part of the populatio
population
n of graduate
graduatess came from all other
other courses
courses
besides
besides Educatio
Education n and Business
Business??

(A) One third


ird
(B) One fourth
(C)
(C) Less
Less than
than one
one four
fourth
th
(D) Morethantw
rethantwo o thir
third
ds

37. How many percent


percent of the
the total populatio
population
n finished
finished Medicine
Medicine??

(A) 2.12%
(B) 6.29%
(C) 21.22%
(D) 36.54%

38. Which
Which of the following
following statemen
statements
ts does
does NOT correctly
correctly describe
describe the distributi
distribution
on
of degrees?

(A)
(A) The
The numb
numberer of degre
degrees
es in Medic
Medical
al Scien
Science
ce differ
differed
ed from
from that
that in Socia
Sociall Science
Science
  by 67,000.
67,000.
(B)
(B) The
The larges
largestt number
number of
of degree
degreess was gran
granted
ted in Busin
Business
ess..
(C)
(C) Ther
Theree were
were more
more degr
degree
eess giv
given
en in Nurs
Nursin
ing
g than
than in Educ
Educat atio
ion.
n.
(D)
(D) There
There were
were 386,4
386,463
63 degre
degreeses grante
granted
d in Denti
Dentistr
stry.
y.

PRACTICE BOOKLET
BKL PhiLSAT_AA_0317 GO ON TO THE
HE NEXT PAGE
56

For
For items
items 39
39 to 41, refer
refer to the chart
chart belo
below.
w.

PERCEN
PERCENT
T DISTRIB
DISTRIBUTIO
UTION
N OF OVERSE
OVERSEAS
AS FILIPIN
FILIPINOO WORKER
WORKERS
S (OFWs)
(OFWs)
BY PLAC
PLACEE OF WORK
WORK:: 2012
2012

SaudiArabia (20.6%)
(20.6%)

UnitedArab Emirates
Emirates
Europe (8.7%)
(14.9%)

  North and South America


America Singapore (7.5%)
Asia (79.7%)
(7.6%) Qatar (6.5%)
(6.5%)
Australia
Australia (2.2%)
(2.2%) Hongkong (5.5%)
Kuwait
Kuwait (4.2%)
(4.2%)
Africa
Africa (1.7%)
(1.7%)
Japan (3.4%)
(3.4%)

Other countries (17.1%)

Source: PhilippineS tatistics Authority, 2012 Survey on OverseasFilipinos


URL: https://psa.gov.ph/content/201
https://psa.gov.ph/content/2012-survey
2-survey-overseas-filipinos
-overseas-filipinos

39. Based
Based on the chart
chart,, which
which of the follow
following
ing gener
generali
alizat
zation
ionss are true?
true?

I. There
There are more
more OFWs
OFWs in Asian
Asian coun
countrie
triess than
than in other
other places
places combin
combined
ed..
II.
II. Ther
Theree are
are more
more OFWs
OFWs in Japa
Japan
n than
than in Aust
Austraralia
lia and
and Afri
Africa
ca comb
combin
ined
ed..
III.
III. Abou
Aboutt one
one fift
fifth
h of the
the OFWs
OFWs in Asia
Asia are
are in Saud
Saudii Ara
Arabi
bia.
a.
IV. Abou
Aboutt one
one fourth
fourth of the OFWs
OFWs are in North
North and South
South Ameri
America
ca and
and Europe
Europe..

(A) I and
and II only
(B) I and III only
(C) II and IV onllyy
(D) I, III, and IV only

40. Which
Which place(s)
place(s) of
of work
work comprise(
comprise(s)
s) approx
approximate
imately
ly one tenth of the
the total OFWs in 2012?
2012?

(A)
(A) Nort
North
h and
and Sout
South
h Amer
Americicaa and
and Aust
Austra
rali
liaa
(B)
(B) Nort
North
h and
and Sout
South
h Amer
Americicaa
(C)
(C) Euro
Europe
pe and
and Aust
Austra
rali
liaa
(D) Europe

41. What
What will
will happe
happen n to the ranking
ranking of coun
countries
tries according
according to the percentag
percentagee of OFWs
OFWs if the
the
frequen
frequency
cy distributi
distribution
on is reported
reported instead
instead of percent distribution
distribution??

(A)
(A) The
The rankin
ranking g canno
cannott be determ
determineined
d from
from the frequ
frequenc
ency
y distrib
distributi
ution
on..
(B)
(B) The
The rank
rankin
ingg will
will be reve
revers
rsed
ed..
(C)
(C) The
The rank
rankin
ingg will
will be the
the same
same..
(D)
(D) Ther
Theree will
ill be
be tied
tied rank
ranks.
s.

PRACTICE BOOKLET
BKL PhiLSAT_AA_0317 GO ON TO THE
HE NEXT PAGE
57

For
For items
items 42 to 46, refer
refer to the follow
following
ing situat
situation
ion..

In 2015
2015,, Toyo
Toyotata Motor
Motor Philip
Philippin
pines
es was
was able
able to sell
sell a total
total of
of 124,4
124,426
26 units
units.. This
This show
showss a
growth
growth of 17.8%
17.8% from
from its 2014
2014 sales
sales perfor
performan
mance.
ce. Below
Below is a table
table showin
showing
g the top 5 model
modelss
that have
have the largest
largest numbe
numberr of units sold during
during the year.
year.

Model Vios 1.3 Hiace Innova Di


Diesel Fortuner 4×2 Avanza
  No.
No. of Units
Units
30,754 16,675 16,079 15,148 8 ,646
Sold
Source: http://www.topgear.com.ph/n
http://www.topgear.com.ph/news
ews

42. How
How many
many units
units were
were sold
sold in 2014
2014??
(A) 71,762
(B) 87,302
(C) 102,278
(D) 105,62
,625

43. Supp
Suppose
ose that,
that, in the follow
following
ing year,
year, the numb
number
er of units
units sold
sold for the top 5 models
models
doubled
doubled except
except that for
for Vios
Vios 1.3. Which
Which model(s)
model(s) would
would have more number
number of units
sold
sold than
than Vios
Vios 1.3?

(A) Hiace only


(B)
(B) Inno
Innova
va Dies
Diesel
el and Hiac
Hiacee only
only
(C) Fortun
rtuneer 4×2, Inno
Innova
va Dies
Diesel
el,, and
and Hiac
Hiacee only
only
(D)
(D) All
All fou
fourr mod
model
elss

44. Which
Which of the follow
following
ing can be conclu
conclude
ded
d based
based on the table?
table?
(A)
(A) More
More peop
people
le buy
buy Vios
Vios 1.3
1.3 than
than any
any othe
otherr Toy
Toyot
otaa mode
model.l.
(B)
(B) Avan
Avanzaza is the
the most
most expe
expens
nsivivee Toyo
Toyota
ta mode
model.
l.
(C)
(C) Peop
People
le do not
not buy
buy a vehi
vehicl
clee base
basedd on the
the type
type of fuel
fuel it uses
uses..
(D)
(D) All of thes
thesee

45. If a 5% growt
growth
h sale
sale of Inno
Innova
va Diesel
Diesel mod
model
el is forec
forecast
asted
ed in 2018
2018,, how
how many
many units
units of
of this
model
model will be sold?
sold?

(A)
(A) Exac
Exactl
tly
y 16,8
16,800
00
(B) More tha
than 16,88
,880
(C)
(C) Less
Less than
than 16,8
16,880
80
(D)
(D) Exac
Exactl
tly
y 16,8
16,880
80

46. Which
Which of the following
following statemen
statements
ts abou
aboutt the
the units
units sold
sold in 2015 is FALSE?
FALSE?

(A)
(A) Abou
Aboutt one
one fourth
fourth of the total
total unit
unitss sold
sold are Vios
Vios 1.3.
1.3.
(B)
(B) Seve
Sevent
nty
y perc
percen
entt of the
the tota
totall units
units sol
sold
d belo
belong
ng to the
the top
top 5 mode
modelsls..
(C)
(C) Ther
Theree were
were more
more Vios
Vios 1.3
1.3 sold
sold than
than Fort
Fortun
uner
er 4×2 and Avan
Avanzaza combi
combine
ned.
d.
(D)
(D) Each
Each of the top 5 models
models is at least
least 10%
10% of the total
total units sold.
sold.

PRACTICE BOOKLET
BKL PhiLSAT_AA_0317 GO ON TO THE
HE NEXT PAGE
58

For
For item
itemss 47
47 to 50, refer
refer to the graph
graph below
below..

PERCEN
PERCENTAG
TAGE
E OF SMARTP
SMARTPHON
HONE E USAGE
USAGE IN 2014
2014
ACRO
ACROSS
SS AGE
AGE GROU
GROUPS
PS IN THE
THE U.S.
U.S.

18-29 30-49 50+

100
Text messaging 98
92

97
Internet use 90
80

Voice/video 93
91
calls 94

91
E-mail 87
87

Source: Pew Research Center


Center American Trends Panel, 2014.
URL: http://www.pewinternet.org/2
http://www.pewinternet.org/2015/04/0
015/04/01/chapter-three-a-wee
1/chapter-three-a-week-in-the-life-analysis-of-
k-in-the-life-analysis-of-
smartphone-users

47. The Pareto


Pareto chart
chart shown
shown below
below is constru
constructe
cted
d for the 30 –  49 age
age grou
group
p base
based
d on
the bar graph.
graph. Which
Which phone
phone feature
feature is misrepres
misrepresente
ented?
d?

100
90
sr
e 80
n
w 70
o
e
n 60
o
h
tpr
50
a 40
sm
f 30
         o
% 20
10
0
T ex
ex tm es
es sa
sagi ng
ng I nt
nte rn
rn et
et us
use V oi
oi ce
ce/ Vi
Vid eo
eo E-mail
calls
Phonefeatures

(A) Text messaging (C) Voice/Video ca


c alls
(B) E-mail
E-mail (D)
(D) Intern
Internet
et use
use

PRACTICE BOOKLET
BKL PhiLSAT_AA_0317 GO ON TO THE
HE NEXT PAGE
59

48.
48. Which
Which of the follow
following
ing statem
statemen
ents
ts abo
about
ut the patter
pattern
n of smartp
smartpho
hone
ne usage
usage among
among
Americans is true?

(A) The youn


younges
gestt age
age group
group was
was high
highly
ly depen
dependen
dentt on a smartp
smartpho
hone
ne for intern
internet
et use
use
compared
compared to text messagin
messaging.g.
(B) The rates
rates of voice/
voice/vid
video
eo callin
calling
g for the
the oldes
oldestt and young
youngest
est smart
smartpho
phonene owner
ownerss were
nearly
nearly identical.
identical.
(C) Using
Using e-mail
e-mail was
was the
the most
most vari
variabl
ablee behav
behavior
ior among
among age
age group
groups.
s.
(D) Text
Text messag
messaging
ing was
was equal
equally
ly popu
popular
lar amon
amongg age
age group
groups.
s.

49.
49. Which
Which smart
smartph
phon
onee featur
featuree showe
showed
d the highe
highest
st varia
variabil
bility
ity in usage
usage among
among owne
owners?
rs?

(A)
(A) Voic
Voice/
e/Vi
Vide
deo
o calls
calls
(B)
(B) Text
Text mess
messagagin
ing
g
(C)
(C) Inte
Intern
rnet
et use
use
(D) E-mail

50.
50. Whic
Which
h smar
smartp
tpho
hone
ne feat
featur
uree has
has a simi
simila
larr rate
rate of usage
usage betw
betwee
een
n two
two age
age grou
groups
ps??

(A)
(A) Text
Text mess
messagagin
ing
g
(B)
(B) Inte
Intern
rnet
et use
use
(C)
(C) Voic
Voice/
e/Vi
Vide
deo
o call
callss
(D) E-mail

STOP!
WAIT
WAIT FOR FURTHE
FURTHER
R
INSTRUCTIONS.
INSTRUCTIONS

PRACTICE BOOKLET
BKL PhiLSAT_AA_0317

You might also like